Вы находитесь на странице: 1из 104

CLASS

ASSIGNMENTS
2016

INDEX
Assignments

Pages

1.

Quantitative Aptitude Assignments 1- 27

2.

Logical Reasoning Assignments. 28-47

3.

Verbal Ability Assignments48-76

4.

General Awareness Assignments...77-100

QUANTITATIVE APTITUDE
CLASS ASSIGNMENT -1
Average, Percentage, Profit/Loss
(c) 20,000

AVERAGE
Complete the following exercise:
Observations
1,2,3,4,5,6,7,8,9,10
21, 24, 27, 30, 33, 36
10, 20, ___ , 35, 45, 65

Complete the given exercise:


No.
Avg.
No.
Avg.
of
weight
of
weight
girls
of girls
boys of boys
10
20 kg
20
30 kg
10
30
45 kg
15 kg
25
40 kg
1.

Average

Karna drove his chariot from Kurukshetra to


Indraprastha at the speed of 40 km/hr and then
he traveled the same distance on his foot at 10
km/hr towards Mathura. He then returned from
Mathura to Kurukshetra via Indraprastha at 24
km/hr riding on a horse.
The average speed of the whole trip is:
(a) 20 km/hr
(b) 25 km/hr
(c) 19.2 km/hr
(d) 18.5 km/hr

6.

The average of nine numbers is M and the


average of three of these is P. If the average of
remaining numbers is N , then
(a) M = N + P
(b) 2M = N + P
(c) 3M = 2N + P
(d) 3M = 2P + N

38.75
27.5

What is the average of the following


observations: 1, 2, 2, 3, 3, 3, 4, 4, 4, 4, 5, 5, 5, 5, 5,
6, 6, 6, 6, 6, 6, 7, 7, 7, 7, 7, 7, 7
(a) 4
(b) 5
(c) 6
(d) None of these

2.

The average age of A and B is 20 years, that of B


and C is 19 years and that of A and C is 21 years.
What is the age (in years) of B?
(a) 39
(b) 21
(c) 20
(d) 18

3.

The average weight of 8 people increases by 2.5


kg when a new person comes in place of one of
them weighing 65 kg. What might be the weight
of the new person?
(a) 84 kg
(b) 85 kg
(c) 86 kg
(d) 87 kg

4.

5.

40

overall
average
weight

The average salary of 12 employees of STAR PLUS


is Rs. 18,000 and that of 15 employees of NDTV is
Rs. 25,000 per month. The average monthly of all
employees in rupees is:
(a) 17,000
(b) 16,500

(d) None of these

PERCENTAGE
Complete the given exercise:
Fraction
Corresponding percentage
1/2
1/4
1/_
12.5%
33.33%
16.66%
11.11%
1.

A number is increased by 37.5% and it gives 99 as


the final answer. The number is:
(a) 140
(b) 61.5
(c) 72
(d) 48

2.

Two candidates fought an election. One got 65%


of the votes, and won by a margin of 3,600 votes.
The total number of votes polled is:
(a) 12,000
(b) 10,000
(c) 2,000
(d) 8,000

QUANTITATIVE APTITUDE CLASS ASSIGNMENT - 1

Page 1

3.

4.

5.

The price of an item is increased by 20% and then


decreased by 20%. The final price of the item as
compared to its original price is:
(a) 4% more
(b) 4% loss
(c) No profit No loss
(d) Cannot be found
The average weight of a class of students is 67.5
kg. The weight of the class teacher is 25% more
than the average weight of the class. The average
weight of the class is less than the class teacher
by x%. The value of x is:
(a) 33.33%
(b) 25%
(c) 20%
(d) Cannot be found

4.

If the cost price of 15 apples is equal to the selling


price of 20 apples. What is the gain or loss
percentage?
_________

5.

By selling 8 bananas, a fruit seller gains the selling


price of 1 banana. Calculate his gain percentage.
_________
Anna sold his muffler to Arvind at a profit of 20%
and Arvind sold it to Kiran at a profit to 100%.
Kiran sold it to Narendra at a loss of 9.09%.
Narendra then sold the same muffler to Anna at
a profit of 10%. What is Annas overall profit/loss
percentage?
_________

6.

Each side of a cube is increased by 20%. The total


percentage increase in the surface area of the
cube is:
(a) 144%
(b) 40%
(c) 44%
(d) 72.8%

PROFIT & LOSS


Complete the following exercise:
CP
100
50

Profit/Loss

SP
80

+25
-70

200

100
300

1.

Profit/Loss
(%)

+25%
+75%

Given are some questions without any option.


Solve these questions and write your answer in
the space provided
Find the cost price of an article which is sold for
Rs. 220 at a loss of 12%
_________

2.

A man sells two wrist watches. One at a profit of


10% and another at a loss of 10%, but the selling
price of each watch is Rs. 200. Find the:
(1) Net percentage profit or loss.
_________
(2) Net amount of profit or loss.
_________

3.

A man sells a book at a profit of 20%. If he had


bought it at 20% less and sold it for Rs. 18 less, he
would have gained 25%. The cost price of book is
_________.

QUANTITATIVE APTITUDE CLASS ASSIGNMENT - 1

Page 2

QUANTITATIVE APTITUDE
CLASS ASSIGNMENT - 2
Ratio / Proportion, Mixtures, Partnership
Complete the given table
Ratio
Surajs Salary
3:4
300
3:4
3:4
2:_
800
600

Dheerajs Salary
800
2800
1200
900

Find the ratio of 90 cm to 1.5 m

2.

The ratio of A: B is 1: 3, B: C is 2: 5, C: D is 2: 3.
Find the value of A: B: C: D.
_________

3.

The sum of two natural numbers is 64. Which of


the following cannot be the ratio of these two
numbers?
(a) 3 : 5
(b) 1 : 3
(c) 7 : 9
(d) 3 : 4

4.

If A, B & C have some sweets with them all


together weighing 110 kg. If A gives half of his
share to B then he now has twice as much as A.
but if C gives half of his share to B, then B now has
four times as much as that of C. What amount of
sweets does B has initially?
(a) 60 kg
(b) 30 kg
(c) 20 kg
(d) None of these

_________

5.

A cat takes 5 steps for every 7 steps of a dog, but


5 steps of a dog are equal to 6 steps of a cat. What
is the ratio of speed of cat to that of dog?
(a) 24 : 25
(b) 42 : 25
(c) 24 : 19
(d) 25 : 42

6.

A camel pursue an elephant and takes 5 leaps for


every 7 leaps of the elephant, but 5 leaps of

1800
1200

elephant are equal to 3 leaps of camel. What is


the ratio of the speeds of camel and elephant?
(a) 5 : 4
(b) 4 : 5
(c) 25 : 21
(d) 21 : 25

RATIO
1.

Total Salary

MIXTURES
7.

In the 75 litres of mixture of milk and water, the


ratio of milk and water is 4: 1. The quantity of
water required to make the ratio of milk and
water 3 : 1 is :
(a) 1 litre
(b) 3 litres
(c) 4 litres
(d) 5 litres

8.

450 litres of a mixture of milk and water contains


the milk and water in the ratio 9: 1. How much
water must be added to make a new mixture
containing milk and water in the ratio 3: 1?
(a) 54 litres
(b) 90 litres
(c) 45 litres
(d) 63 litres

9.

The ratio of petrol and kerosene in a container is


3: 2. When 10 liters of mixture is taken out, and
replaced by the kerosene, the ratio becomes 2: 3.
The total quantity of the mixture in the container
is:
(a) 25 litres
(b) 30 litres
(c) 45 litres
(d) Cannot be found

QUANTITATIVE APTITUDE CLASS ASSIGNMENT - 2

Page 3

PARTNERSHIP
10. A company makes a profit of Rs. 9,00,000. Twenty
percent of which is paid as taxes. If the rest is
divided among the partners, P, Q & R in the ratio
3

of 1 2, then the shares of P, Q & R are


2

respectably:
(a) Rs. 2,40,000, Rs. 3,20,000, Rs. 1,60,000
(b) Rs. 3,20,000, Rs. 2,40,000, Rs. 1,60,000
(c) Rs. 1,60,000, Rs. 3,20,000, Rs. 2,40,000
(d) Rs. 1,60,000, Rs. 2,40,000, Rs. 3,20,000
11. We have to divide a sum of Rs 13,950 among
three persons A, B and C. B must get the double
of As share and C must get Rs 50 less than the
double of Bs shares. The share of A will be:
(a) Rs 1,950
(b) Rs. 1,981.25
(c) Rs. 2,000
(d) Rs. 2007.75
12. A and B are partners in a business. They invest in
the ratio 5 : 6, at the end of 8 months A
withdraws. If they receive profits in the ratio of
5:9, find how long Bs investment was used?
(a) 12 months
(b) 10 months
(c) 15 months
(d) 14 months

QUANTITATIVE APTITUDE CLASS ASSIGNMENT - 2

Page 4

QUANTITATIVE APTITUDE
CLASS ASSIGNMENT - 3
Speed, Distance, Time, Work, Pipes

SPEED, DISTANCE, TIME


Convert the given units.
___ m = 5 km
___ m = 2.5 km
1400 m = ____ km
8000 m = ____ km

150 m = ____ cm
189 m = ____ cm
____ Km = 400 cm
____ Km = 2400 cm

Complete the given table


Sl. Distance
Time
No
1
100 km
20 hr
2
____ km
2 hr
3
20 km
_____ min
4
100 m
20 hr
5
___ m
240 sec
6
20 m
_____ hr

1 hr
0.5 hr
2.5 hr
4 hr

= _____ mins
= _____ mins
= _____ mins
= _____ mins

60 min = _____ hr
480 min = _____ hr

Convert the given speeds


Speed

Km/hr
32

___ km/hr
30 km/hr
4 km/hr
___ m/sec
30 km/min
4 km/hr

m/sec
15

60
100
2.5

Revise Your Basics


1.

A car travels 15 meters in 2 seconds. Find its


speed in m/s.

4.

The rocket used to launch Mangalyaan covers


200 km in 6 minutes. Find its speed in km/hr.

2.

Usain Bolt runs 100 metres in 9 seconds. Find his


speed in km/hr.

5.

A skateboarder rolls at 8 m/s. how long would it


take him to roll 60 m at this speed?

3.

Boing 747 travels 15 kms in 3 minutes. Find its


speed in m/s.

SPEED, DISTANCE, TIME


1.

A bus travels at an average speed of 100 km/hr,


stopping for 3 minutes after every 75 km. How
long did it take to reach its destination 600 km
from the starting point?
(a) 6 hr 24 min
(b) 6 hr 21 min
(c) 6 hr 18 min
(d) 6 hr 15 min

2.

A car goes from a Delhi to Panipat at a speed of


32 km/hour but returns to Delhi at a slower
speed. If its average speed for the trip is 28
km/hour, find the return speed of the train.
(a) 48 km/hour (b) 25 km/hour
(c) 52 km/hour
(d) 47.4 km/hour

QUANTITATIVE APTITUDE CLASS ASSIGNMENT - 3

Page 5

3.

A train crosses a pole in 10 seconds. If the speed


of the train is 10 m/s, then what is the length of
the train?
(a) 1 m
(b) 10 m
(c) 100 m
(d) Cannot be found

6.

The speed of a boat is 10 kmph and that of a


stream is 5 kmph. What will be the time taken by
the boat to travel
(a) Upstream 10 km in the stream?
(b) Downstream 10 km in the stream?

4.

A train, 800 meter long is running with a speed of


78 km/hr. It crosses a tunnel in 1 minute. What is
the length of the tunnel?
(a) 650 meter
(b) 555 meter
(c) 500 meter
(d) 458 meter

7.

By increasing its speed by 1km per hour, a person


was able to reach his office early by 1 hour. If the
distance between his office and his house is 20
km, then what was his original speed was?
(a) 3 km/hr
(b) 4 km/hr
(c) 5 km/hr
(d) 6 km/hr

5.

How many seconds will a 500 meter long train


take to cross a man walking with a speed of 3
km/hr in the direction of the moving train if the
speed of the train is 63 km/hr
(a) 25 seconds
(b) 28 Seconds
(c) 30 seconds
(d) 35 Seconds

PIPES & CISTERNS


Complete the Given Table
Sl. No

Capacity of Tank

Time taken to fill up


tank

120 L

1 hr

240 L

2 hr

360 L

Fraction of Tank
Filled in one minute

300 ml
3 min

1.

A pipe is known to fill up a tank in 1 hour. What


portion of the tank is filled up by the pipe in 1
minute?

2.

In continuation to the previous question, if it is


known that another pipe fills up the same tank in
30 minutes, then what portion of the tank is filled
up by the pipe in 1 minute?

3.

Capacity of tank
filled in 1 min

Two pipes A and B can fill a tank in 6 hours and


4 hours respectively. If they are opened on
alternate hours and if pipe A is opened first, in
how many hours, the tank will be full?

400 ml
4.

In a house, the water supply provided by MCD


guarantees to fill up a standard water tank in 3
hours. And a tap in the kitchen is known to drain
the water tank in 5 hours. If the tank is initially
empty and both of them are opened
simultaneously, then in how much time will the
tank be full?

5.

Two taps can fill a tub at 10 L/min. and 15 L/min.


respectively. A pipe can empty it in at the rate of
8L/min. If all the three are kept open The tank
gets filled in 24 hours. What is the capacity of the
tank?

QUANTITATIVE APTITUDE CLASS ASSIGNMENT - 3

Page 6

WORK
1.

2.

3.

A can do a piece of work in 8 days, B can


do a piece of work in 12 days. How many
days will they take to complete the piece of
work working together?
A and B can together complete a piece of
work in 4 days. If A alone can complete the
same work in 12 days, in how many days can
B alone complete that work?
(a) 4 days
(b) 5 days
(c) 6 days
(d) 7 days
A is twice as good as workman as B and
together they finish a piece of work in 18
days. In how many days will B alone finish
the work?
(a) 27 days
(b) 54 days

(c) 56 days

(d) 68 days

4. 20 workers can finish a piece of work in 30


days. After how many days should 5 workers
leave the job so that the work is completed in 35
days?
(a) 5 days
(b) 10 days
(c) 15 days
(d) 20 days
5. Tom can copy 50 pages in 10 hours: tom and
brad together can copy 300 pages in 40 hours. In
how much time can brad copy 30 pages?
(a) 27 days
(b) 54 days
(c) 56 days
(d) 68 days

QUANTITATIVE APTITUDE CLASS ASSIGNMENT - 3

Page 7

QUANTITATIVE APTITUDE
CLASS ASSIGNMENT 4
Speed, Distance, Time, Work, Pipes

SPEED, DISTANCE, TIME


1.

2.

Two trains of equal length are running on


parallel lines in the same direction at 46 km/hr
and 36 km/hr. The faster train passes the
slower train in 36 seconds. The length of each
train is:
(a) 40 meter
(b) 45 meter
(c) 50 meter
(d) 55 meter
A 270 metres long train running at the speed of
120 kmph crosses another train running in
opposite direction at the speed of 80 kmph in
9 seconds. What is the length of the other
train?
(a) 220 meter
(b) 225 meter
(c) 230 meter
(d) 235 meter

3.

Two cyclists start on a circular track from a


given point but in opposite directions with
speeds of 7 m/s and 8 m/s respectively. If the
circumference of the circle is 300m, after what
time will they meet at the starting point?
(a) 20
(b) 100
(c) 300
(d) 200

4.

If Rahul rows 15 km upstream in 3 hours and


21 km downstream in 3 hours, then the speed
of the stream is
(a) 5 km/hr
(b) 4 km/hr
(c) 2 km/hr
(d) 1 km/hr

5.

A man can row at 5 kmph in still water. If the


velocity of the current is 1 kmph and it takes
him 1 hour to row to a place and come back.
How far is that place.
(a) 0.4 km
(b) 1.4 km

(c) 2.4 km

(d) 3.4 km

6.

Buses start from a bus terminal with a speed of


20 km/hr at intervals of 10 mins. What is the
speed of a man coming from the opposite
direction towards the bus terminal if he meets
the buses at intervals of 8 mins?
(a) 3 km/hr
(b) 4 km/hr
(c) 5 km/hr
(d) 7 km/hr

7.

In a race of 600 meters, Ajay beats Vijay by 60


meters and in a race of 500 meters Vijay beats
Anjay by 25 meters. By how many meters will
Ajay beat Anjay in a 400 meter race?
(a) 48 m
(b) 52 m
(c) 56 m
(d) 58 m

QUANTITATIVE APTITUDE CLASS ASSIGNMENT - 4

Page 8

WORK, PIPES & CISTERNS


1.

Ram can do a piece of work in 8 days which


Shyam can finish in 12 days. If they work at it
on alternate days with Ram beginning, in how
many days, the work will be finished?
(a) 9 and 1/3
(b) 9 and 1/24
(c) 9and 1/2
(d) 10 and 1/3

2.

A is twice as good a workman as B and


together they complete a work in 15 days. In
how many days can the work be completed by
B alone.
(a) 22
(c) 30

3.

4.

(b) 37

7.

Two candles of the same height are lighted at


the same time. The first is consumed in 4 hr
and the second in 3 hr. Assuming that each
candle burns at a constant rate, in how many
hrs after being lighted was the first candle
twice the height of the second?
(a) 0.45 hr
(b) 1.5 hr
(c) 2.6 hr
(d) 2.4 hr

8.

40 men build a wall 4 meters high in 15 days.


The number of men required to build a similar
wall of 5 meters height in 6 days would be:
(a) 115
(b) 105
(c) 135
(d) 125
(e) None of these

9.

If 12 men or 6 women can bake 48 cakes in one


day, then how many cakes can 8 men and 8
women bake in one day?

1
2

(d) 45

4 men and 6 women finish a job in 8 days, while


3 men and 7 women finish it in 10 days. In how
many days will 10 women working together
finish it?
(a) 30 days
(b) 40 days
(c) 50 days
(d) 60 days
5 men and 2 boys working together can do four
times as much work as a man and a boy.
Working capacity of man and boy is in the ratio
(a) 1:2
(b) 1:3
(c) 2:1
(d)2:3

5.

Sakshi can do a piece of work in 20 days. Tanya


is 25% more efficient than Sakshi. The number
of days taken by Tanya to do the same piece of
work is:
(a) 15
(b) 16
(c) 18
(d) 25

6.

Twenty women can do a work in sixteen days.


Sixteen men can complete the same work in
fifteen days. What is the ratio between the
capacity of a man and a woman?
(a) 3 : 4
(b) 4 : 3
(c) 5 : 3
(d) Data inadequate

10. X is 60% less efficient than Y. If X does the job


in 30 days, Y would take how many days to
finish the job?
(a) 12 days
(b) 18 days
(c) 75 days

(d) 18 days
4

(e) None of these

QUANTITATIVE APTITUDE CLASS ASSIGNMENT - 4

Page 9

QUANTITATIVE APTITUDE
CLASS ASSIGNMENT - 5
Number Systems
1.

Which of the following is not correct :


(a) 2 is a prime number
(True / False)
(b) -2 is a prime number
1

2.

(True / False)

(d) -12 is a composite number

(True / False)

If x is an odd integer, all of the following are


odd except:
(a) x 2
(b) 6x + x
2
(c) x + 2x
(d) x2 + x

3.

What number(s) can be put in place of c in


the number 38c to make it divisible by :
(i) 2
(ii) 3
(iii) 4
(iv) 5
(v) 6
(vi) 8
(vii) 9
(viii) 10
(ix) 11

4.

If n is a positive integer, which one of the


following numbers must have a remainder of
3 when divided by any of the numbers 4, 5,
and 6?
(a) 12n + 3
(b) 24n + 3
(c) 90n + 2
(d) 120n + 3

5.

6.

The largest possible length of measuring


tape which can measure 525cm, 1050cm
and 1155cm length of cloth in a minimum
number of attempts (measuring it in cms
only) is:
(a) 25
(b) 10
(c) 75
(d) None of these

8.

Find the largest no. which divides 5, 7, 9, 12


& 15 and leaves a remainder 1 in each case:

9.

In a bangle shop, if the shopkeeper displays


the bangles in the form of a square then he
is left with 38 bangles with him. If he wanted
to increase the size of the square by one unit
on each side, then he notices that he is short
of 25 bangles. The actual number of bangles
that he has with him is:
(a) 1690
(b) 999
(c) 538
(d) CBD

(True / False)

(c) is an even number


2

7.

A hundred and twenty digit number is


formed by writing the first x natural numbers
in
front
of
each
other
as
12345678910111213.. Find the remainder
when this number is divided by 8.
(a) 6
(b) 7
(c) 2
(d) 0
On a road three traffic lights change after 48,
72, 108 seconds. If all the three lights
changes simultaneously on 8:20:00 pm, then
at which of the following time the lights will
again change simultaneously?
(a) 8 : 27 : 12
(b) 8 : 25 : 14
(c) 8 : 30 : 00
(d) 8 : 29 : 30

10. What will be the remainder of


11. What will be the remainder of

(16)5
2

(9)5
2

12. What will be the remainder when 1623 is


divided by 5?
13. If it is known that: 7! = 7X6X5X4X3X2X1
5! = 5X4X3X2X1 and so on. What is the value
at the unit place of: 1! + 2! + 3! + 4! +
5! + + 10!
14. What is the remainder when
1! + 2! +
3! + 4! + 5! + + 15! is divided by 5?
15. What is the digit at the units place of:
(a) 623
(b) 156

QUANTITATIVE APTITUDE CLASS ASSIGNMENT - 5

Page 10

QUANTITATIVE APTITUDE
CLASS ASSIGNMENT - 6
Equations and Logarithm

Equations
Identify the given equations
Equation

Degree

Type of Equation
(Put a check in the correct box)
Linear Quadratic
Cubic

Number of
Variables

Value of
Variables
(If possible)

3 + 4 = 0
3 + 4 = 2
2 + 2 + 1 = 0
3 + 3 2 + 1 = 0
2 + 3 + 22 = 9
Complete the following table:
Equations

2
3

2
6

6
12

1.

+
+ 7

+
+ 3

+ 3
+ 6

= 9
= 15

= 9
= 15

= 15
= 30

Value of
Constants
1 =
1 =

Unique
Solutions

Intersecting [ ]
Parallel
[ ]
Coincident [ ]

Unique
[ ]
No solution [ ]
Infinite
[ ]

Intersecting [ ]
Parallel
[ ]
Coincident [ ]

Unique
[ ]
No solution [ ]
Infinite
[ ]

Intersecting [ ]
Parallel
[ ]
Coincident [ ]

Unique
[ ]
No solution [ ]
Infinite
[ ]

1 =
1

2 =
2 =

2 =

1 =
1 =

1 =

2 =
2 =

2 =

1 =
1 =

1 =

2 =
2 =

Type of Lines

Ratio of Constants

2 =

Solve the following questions:


In an entrance test comprising of 100 questions,
a student scores 6 marks for every correct answer
and loses 2 marks for every wrong answer. If a
student attempted 90 questions and scores 356
marks, then the number of questions answered
correctly are:

2.

1
2

1
2

1
2

(a) 67 (b) 68 (c) 70


(d) 71
The number of values of for which the system
of equations:
( + 1) +
8 = 4
+ ( + 3) = 3 1
has infinitely many solutions, is:

QUANTITATIVE APTITUDE CLASS ASSIGNMENT - 6

Page 11

(a) 0
(b) 1
(c) 2
(d) Infinite
Solving Quadratic Equations and Relation between roots of quadratic equation:
Equations
Root 1
Root 2
Sum of roots
2 + 3 + 2 = 0
2 + 4 + 4 = 0
1
4 2 + 4 __ = 0
2
1.

2.

Kunal told Kanika that he has either or


marbles. Further he stated that , satisfy the
equation 2 ( + 1) + = 0, then ( +
1)( + 1) =
(a) 1
(b) 1
(c) 1 +
(d)

Product of roots

3.

The positive value of m for which the roots of


equation 2 + 4( 2) + 27 = 0 are in ratio
1:3, is:
(a) 7
(b) 5
(c) 3
(d) 1

4.

If , are the roots of the equation 3 2 + 4 +


1

7 = 0, then the value of + is

If one root of the equation ax 2 + bx + c = 0,


a 0, be reciprocal of the other, then:
(a) b = c
(b) a = c
(c) a = 0
(b) b = 0

(a)
(c)

3
7
4
3

(b)
(d)

7
7
4

Logarithm
Fill in the blank Spaces
23 = _____

33 = _____

54 = _____

2(___ ) = 8

3(___ ) = 27

5(___ ) = 625

log 2 8 =

log 3 27 =

log 5 625 =

Using the given properties of log, solve the


following:
1. log 4 434 =
4. log 8 16 =
2. log10 6 =
5. log 2 6 =
3. log10 0.01 =
6. log 1 =

1.

Solve the following:


log10 10000 =
log 8 512 =
log 3 9 =

2.

If log b (5) = c, then b4c will be:


(a) 4
(b) 5
(c) 25
(d) 625

3.

The number of real solutions of the equation


log () = 2 log ( + 1)
(a) 1
(b) 2
(c) 3
(d) 4

4.

If log 2 log 4 ( + 4 + ) = 0, then what is the


value of?
(a) 1/4
(b) 3/4
(c) 9/4
(d) Cannot be solved

Properties of log

(a) log ab = b log(a)


(b) log(ab) = log(a) + log(b)
a

(c) log ( ) = log(a) log(b)


b

(d) log c a =

logk a
logk c

Important log values:


log10 2 = 0.3010

log10 3 = 0.4771

Solve the following questions:


If log 4 64 + log 3 9 = log10 , then is equal to
(a) 105
(b) 107
12
(c) 10
(d) 1015

QUANTITATIVE APTITUDE CLASS ASSIGNMENT - 6

Page 12

QUANTITATIVE APTITUDE
CLASS ASSIGNMENT - 7
Permutation & Combination , Probability

1.

How many numbers of following digits can


be formed by using 1, 2, 3, 4, 5 (with
repetition)?
(a) 5 digit
(b) 4 digit
(c) 3 digit

2.

How many six digit numbers can be formed


with 0, 1,2,3,4 and 5? (Without repetitions)
Now play with restrictions:
How many of these numbers are
i. Even numbers
ii. Between 2000 and 4000
iii. Divisible by 5

3.

4.

Find the number of ways in which 6 boys &


6 girls be seated in a row so that:
I. All the boys sit together
II. All the girls are always together
III. All the girls are never together
IV. Boys & girls sit alternatively
V. No two girls may sit together

V.

If Sachin is selected to play, Kambli must be


selected.
VI. If Sachin is selected, Kambli must be selected
and vice versa.
VII. If Sachin is selected to play, then Kambli
must not be in the team.
VIII. In these questions how do you find the
batting orders
6.

There are 3 players of each state, Haryana


Delhi & Mumbai respectively. In how many
ways 3 people can be awarded such that at
least 1 person from each state is awarded.

7.

If P(n, r) = 720 and C(n, r) = 120, find r?

8.

(i) If a dice is rolled. Then answer the


following:
(a) Probability of getting an even
number?
(b) Probability of getting number
greater than 2 and less than 5?
(ii) If two dices are rolled simultaneously,
Find probability of :
(a) Getting sum greater than 6.
(b) Less than or equal to 6.

9.

Find:
i. Probability of getting the card with 9
written on it from a pack of 52 cards?
ii. Probability of getting a jack of spade or
queen of hearts from a pack of 52 cards?

Find if (, + ): (, ) = .
Concept Puzzle: In how many ways can 9
identical balls be arranged?

5.
I.
II.
III.

IV.

In how many ways can a team of 11


cricketers be selected from 15 players?
Total number of ways (unrestricted)
In the team a player named Sachin must
always be there
If out of 15 players, 7 are batsmen, 6 are
bowlers and 2 are wicketkeepers. In how
many ways the team can be formed
comprising of 5 batsmen, 5 bowlers and 1
wicketkeeper?
If out of 15 players, 7 are batsmen, 6 are
bowlers and 2 are wicketkeepers. In how
many ways the team can be formed
comprising of at least 3 batsmen, at least 3
bowlers and 1 wicketkeeper?

10. If we have to pick 3 balls out of a box


containing 3 yellow and 4 black balls then
find probability that we will get all the balls
of same colour.
11. A dice is thrown twice. What is the
probability that at least one of the two
throws comes up with the number 4?

QUANTITATIVE APTITUDE CLASS ASSIGNMENT - 7

Page 13

QUANTITATIVE APTITUDE
CLASS ASSIGNMENT - 8
Probability
1. An urn contains three white and two
blackballs. Balls are drawn one by one with
replacement. What is the probability that a black
ball appears at the fourth draw?
2
24
(a)
(b)
5
625
(c)

54

(d)

625

81

7. The probability of A and B occurring together is


0.45. The probability of A occurring is 0.6, the n
the probability that B will occur, given that A has
already occurred is
(a) 0.75
(b) 0.27
(c) 0.5
(d) cannot be found.

625

2. If A and B are two arbitrary events and we


denote by 1=[]+[],
2=[], then the probability that exactly one
of the events will occur is
(a) 12
(b) 122
(c) 1+2
(d) 1

8. An urn has four coins. Three coins are fair and


one coin is biased. The probability of head for the
3
biased coin is . A coin is drawn at random from
4
the urn and tossed.What is the probability that a
head is observed?
9
7
(a)
(b)
(c)

3. Two coins with probability of heads p and q,


respectively are tossed independently. If P (both
coins show up tails) = P (both coins show up
heads), then p +q equals
1
1
(a)
(b)
(c)

4
3

16
7

32

(d)

16
3

16

(d) 1

4. Let A and B be two events such that P (B)>0 and


P (A/B) > P ( /B), then
1
1
(a) P (A/B) >
(b) P (A/B) =
(c) P (A/B) <

2
1
2

(d) P (A/B) =

2
1
3

5. The minimum number of tosses of an unbiased


coin, necessary to ensure a probability of getting
at least one head, greater than 0.5 is
(a) 1
(b) 2
(c) 3
(d) 4
6. Event S and T are independent with P(S) <P (T),
6
P (ST) = , and P (S|T) +P (T|S) = 1, then P(S) is
(a)
(c)

1
5
3
5

25

(b)
(d)

2
5
1
25

QUANTITATIVE APTITUDE CLASS ASSIGNMENT - 8

Page 14

QUANTITATIVE APTITUDE
CLASS ASSIGNMENT - 9
Sequences and Series, Complex Numbers and Advance Number Systems

Sequences and Series


Concept Puzzle:
Is number series 1, 2, 3, 4, 5 .. an arithmetic
progression? Why or Why not?
1.

IMPORTANT QUESTION:

Given a series as = + ,and


and are and , then find
Find the sum to terms of the series
+ + + +

9.

st

If on 1 January, 20115 you got Rs. 1 , Rs.


2 on the second day, Rs. 3 on the third day
and so on, what will be the amount that
you have on 31st December 2015?

10. Find the sum of the series:


+ + + +

2.

If the pth term of an A.P. is q and the q th


term is p, then the rth term is

11. Insert three geometric means between 2


and 32

3.

Find four numbers in A.P. whose sum is


20 and the sum of whose squares is 120.

4.

th

th

12. What is the sum of series

th

If the p , q and r terms of an A.P. are


a, b, c respectively, find the value of a (qr) + b(r-p) + c (p-q)

Find the value of:

( ) ( )
( )
5.

The first term of a G.P. is 1. The sum of


the third and fifth terms is 90. Find the
common ratio of the G.P.

6.

Sum the series to infinity

8.

For all 3 digit numbers that leave a


remainder of 2 when divided by 3
(a) Find the number of terms,
(b) Find the sum of number of terms.
The sum of first three terms of a G.P. is 16
and the sum of the next three terms is
128. Find the sum of terms of the G.P.

up to terms

13. Find the value of

= +
14. Which of the following is Harmonic
Progression?
(a) 2,3,4,5
1 1 1 1
(b) . , ,
2 4 6 8
(c) 2, 4, 5, 8
(d) 2, 4, 8, 16, 32

Complex Numbers

3 5
3 5
3 5
- 2 + 3 - 4 + 5 - 6 +
4 4
4 4
4 4
7.

IMPORTANT CONCEPTS
What is the cube root of 1?
What is ?

1.

Find the least positive value of , if


(

2.

) =1

Find real values of x and y for which the


following equalities hold.
( + ) + ( + ) = ( + )

QUANTITATIVE APTITUDE CLASS ASSIGNMENT 9

Page 15

(a) 5, 7 or 6, 7
(b) 8, 9 or 9, 8
(c) 5, 2 or 5, 2
(d) 7, 10 or 7, 10
1.

Convert + into polar form.

2.

If + + = , then find the value


[ +




] + [ + ] + + [ + ]

Advance Number System

1.

Find the remainder of

2.

Find the remainder of

3.

What is the remainder of

4.

What is the reminder of :

5.

How many zeros are there at the end of


100!

QUANTITATIVE APTITUDE CLASS ASSIGNMENT 9

Page 16

QUANTITATIVE APTITUDE
CLASS ASSIGNMENT - 10
Sets, Relations & Functions (Part I)
1.

Which of the following is correct?


(a) {} {{}, }
(b) {, } {, {, }}
(c) {, } {, {, }}
(d) None of these

2.

Which of the following collection is a set?


(a) The collection of all girls in your class
(b) The collection of intelligent girls in your
class
(c) The collection of beautiful girls in your
class
(d) The collection of tall girls in your class

3.

The set of all subsets of a set A is called


the power set of A and is denoted by
(). If = {, , }, then () ?

4.

20 teachers of a school either teach


Mathematics or Physics. 12 of them teach
Mathematics while 4 teach both the
subjects. Find the number of teachers
teaching Physics only.

5. Which of the following is the empty set?


(a) { 2 1 = 0}
(b) { 2 + 1 = 0}
(c) { 2 9 = 0}
(d) { 2 = + 2}
6.

Given a relation A from


= {(, ) = + , }
What is the domain, codomain and range
of A?

7.

Let = {, , }, then the relation =


{(, ), (, ), (, )} on A is:
(a) Reflexive
(b) Transitive
(c) Symmetric
(d) None of these.

8.

Identify the correct match for a function


: :
Type
of Definition
functions
1. Injective
(a) Every element of is the
image of some element
of under.
(b)
Every element of x
2. Many-one
should have a distinct
image in and should
cover every element of
3. Surjective
.
(c) Different element of
have different images in
under f.
4. Bijective
(d) If two or more elements
of set have the same
image in .

9.

If a set A contains 4 elements and a set B


contains 8 elements, the maximum
number of elements in is
(a) 4
(b) 12
(c) 8
(d) None of these

10. Let =
{ (, )(, ), (, )(, ), (, ),
(, ) (, ), (, ) } be a relation on
the set = { , , , }. The relation
is
(a) Reflexive and transitive
(b) Reflexive only
(c) An equivalence relation
(d) Reflexive and symmetric only

QUANTITATIVE APTITUDE CLASS ASSIGNMENT 10

Page 17

11. What is a function?


12. What is Domain and codomain for a
function?
13. Is the given graph a continuous function?

14. Identify the domain and codomain of the


given graph:

IMPORTANT CONCEPTS:
What is the difference between
codomain and range?
What are one-one, onto and into
functions?
What are the other names of one-one,
onto and into functions?
15. What is the range and codomain of the
function () = .
16. Draw the given functions:
||

QUANTITATIVE APTITUDE CLASS ASSIGNMENT 10

Page 18

QUANTITATIVE APTITUDE
CLASS ASSIGNMENT 11
Sets, Relations & Function (Part II) , Height and Distances
1.

Is || continuous, at ?

2.

Given: () = + , () = +
Find and . Also find () and
().

3.

Given:

||

() = {

(d) 6

, 0

The domain of the function () =


+

6.

Which of the following pair of functions is


identical?
(a) () = , () = ()
(b) () =

1
2

, () =

11. Rohan found that for a function , () =


and (()) = , then the
function () =
(a) 2

(c)

is:
(a)
(b)
(c)
(d)

)+ (

)+ (

) + + (

(d) 2

= {(, )| = , x 0, x } and

= {(, )| = , }, then
(a) A B = A
(b) A B = B
(c) A B =
(d) None of these

Height and Distances


1.

, , find the value of

((((()))))
(a) 2
(b) 3
(c) 4

(b) + 3

998
1
499
None of these

If () =

2+5

12. If two sets A and B are defined as

If () = ( ), then the value of


(

8.

10. In a class test consisting of Maths and


Physics, 80% passed in Maths and 50%
passed in Physics 15% failed in both
subjects. If 180 students passed in both the
subjects how many students failed in both
the subjects?
(a) 80
(b) 60
(c) 90
(d) 50

(c) () = log( 1) + log( 2),


() = log( 1)( 2)
(d) None of these
7.

Let = {, $}, = {, , , } then the


number of distinct relations from A to B is
(a) 8
(b) 9
(c) 2
(d) 6

0 , = 0

The point of discontinuity for the above given


function is:
______________
4. The domain of the function () =
5.

9.

A radio transmitter antenna of height 100 m


stands at the top of a tall building. At a
point on the ground, the angle of elevation
of bottom of the antenna is 45 and that of
top of antenna is 60. What is the height of
the building?
(a) 100 m
(b) 50 m

QUANTITATIVE APTITUDE CLASS ASSIGNMENT - 11

Page 19

(c) 50(3+1) m

(d) 50(3-1) m

2.

The angle of elevation of the top of an


unfinished pillar at a point 150 m from its
base is 30. If the angle of elevation at the
same point is to be 45, then the pillar has
to be raised to a height of how many
meters?
(a) 59.4 m
(b) 61.4 m
(c) 62.4 m
(d) 63.4 m

3.

A ladder 25 m long is leaning against a wall


which is perpendicular to the level ground.
The bottom of the ladder is 7 m from the
base of the wall. If the top of the ladder slips
down 4 m, how much will the bottom of the
ladder slip?
(a) 7 m
(b) 8 m
(c) 10 m
(d) 15 m

4.

The shadow of a pole 6 metre high is 15


metre long and at the same time the
shadow of a tree is 25 metre long. What is
the height of the tree?
(a) 21 m
(b) 10 m
(c) 35 m
(d) None of the above

QUANTITATIVE APTITUDE CLASS ASSIGNMENT - 11

Page 20

QUANTITATIVE APTITUDE
CLASS ASSIGNMENT 12
Limits, Continuity and Differentiation
CONCEPT:
What do you understand by the concept of
limits?
1.

Find lim( 2 + 4)2

2.

Find lim

3.

lim

3 2

x0 4 9
1

8.

is :

(c) 1

(b)

||

(a)
(b)
(c)
(d)

3 2 5+6

The points of discontinuity for () =


(

(a)

4.

7.

) are

0. 1
1, 1
0, 1
0, 1

The graph of a function () is shown


below. Which of the following
statements are true of ()?

(d) -1

Find the exact value of

(a) 3
(b) 0
1
(c)

23

(d) The limit does not exist


(i)

CONCEPT:
What is the requirement for a function to be
continuous?

() = 2

(ii) () = 1
1

(iii) () does not exist


1

(iv) () = 1
5.

(a)
(b)
(c)
(d)
6.

The given function:


+ ,
() = {

, =
Is continuous at = 0
Not continuous at = 0
Continuous but not differentiable at = 0
Both continuous and differentiable at = 0
For what value of k is the given function
continuous?

() = {

, =

_________

(v) () does not exist


1

(a)
(b)
(c)
(d)
9.

Only (i) and (iv) are true


Only I and V are true
Only II and IV are true
Only II and V are true

Consider again the function g(x) whose


graph is shown in problem 8. Which of
the following statements are true of g(x)?
I. g is continuous at x = -1
II. g is continuous at x = 1
III. g is differentiable at x = -1
IV. g is differentiable at x = 0
V. g is differentiable at x = 1
(a) only I and IV are true

QUANTITATIVE APTITUDE CLASS ASSIGNMENT - 12

Page 21

(b) only II and IV are true


(c) only I and V are true
(d) only II and V are true
10. What is the derivative of:
(a) 2
(c)
(e)

(b)

(d) cos + 4

+4

(f) et+4 w. r. t t

log

11. If = In(
(A)
(C)

10

10

), then

(B)

(a) zero
(c) Negative

(b) positive
(d) None of these

(D) 0

10e

12. If a function has the equation

( +

+ ), then what is its value at


= ?
13. What will be the slope of the line parallel
to the curve + at =
14. What will be the slope of the line
perpendicular to the curve +
at =
15. Given a curve + . Fid its
(a) Minimum Value (b)
Maximum
Value
16. The Profit of a company is given by
() = + .
The
maximum profit that the company can
make is:
17. For the graph of f(x) depicted, what is the
value of

()

at the point a?

QUANTITATIVE APTITUDE CLASS ASSIGNMENT - 12

Page 22

QUANTITATIVE APTITUDE
CLASS ASSIGNMENT 13
Co-ordinate Geometry, Integration

Co-ordinate Geometry
1.

The equation of the line through the point of


intersection of 3x-y-1=0 and x-3y+5=0,
passing through the point (1,5) is
(a) 2x-y+5=0
(b) 2x+y+5=0
(c) x+y=0
(d) x=1

2.

The points (0,8/3), (1,3) and (82,30) are the


vertices of
(a) An obtuse-angled triangle
(b) A right-angled triangle
(c) An isosceles triangle
(d) None of these

3.

The line segment joining (-3,-4) and (1,-2) is


divided by y-axis in the ratio
(a) 1: 3
(b) 2: 3
(c) 3: 1
(d) 3: 2

4.

The locus of the point, the sum of whose


distances from the coordinate axes is 9 is
(a) 2 2 = 9
(b) 2 2 = 9
(c) 2 2 = 9
(d) None of these

5.

The equation of the line passing through the


point (1,1) and perpendicular to the 3x+4y5=0 is
(a) 3x+4y-7=0
(b) 3x+4y+k=0
(c) 4x-3y+1=0
(d) 4x-3y-1=0

6.

The centroid of a triangle formed by (7,p),(q,6),(9,10) is (6,3), then (p,q)=


(a) (4, 5)
(b) (5, 4)
(c) (-5,-2)
(d) (5, 2)

7.

The base vertices of a right angled isosceles


triangle are (2,4) and (4,2) then its third vertex
is
(a) (1, 1) or (2, 2)
(b) (2, 2) or (4, 4)
(c) (1, 10) or (3, 3)
(d) (2, 2) or (3, 3)

QUANTITATIVE APTITUDE CLASS ASSIGNMENT - 13

Page 23

Integration
1.

2.

( + ) =
(a) +
(c) +

2(1)
1

2(1)
1

(a) (1)2 +

(b) + +
(d) log( ) +

(b) (1)2
(c) (1)2 +

dx is equal to.
(a) cotx-tanx+c
(c) cotx+tanx+c

2(1)

+1

4
1

1
+1

4
1

1
+1

+
+
+

(d) None of these


(b) cotx+tanx+c
(d) cotx-tanx+c

8.

+ dx is equal to
1

3.

If

(a) x+logx+ log( 2 + 1)-tan1 +

(b) x-logx+ log( 2 + 1)-tan1 +

2
1

= ( + ) + ,then

2
1

(a) a = , =arbitrary constant

(c) x-logx- log( 2 + 1)-tan1 +

(b) a = , =arbitrary constant

(d) None of these

(c) a = , =arbitrary constant


2

(d) None of these


4.

9.

is equal to

(a)

(a) tan1 3 +
1
(b) (tan1 3 )2 +
6

(d)
5.

2
1

10.
=
+

dx is equal to:
(a) +A

(a)

(b) +
2

(b)

(c) 2( 1) +

6.

sin () 1 2 +
2

(d) sin1 () 1 2 +
2
2

(d)2( 1) + ,
constant

2
1

2 )3

dx is equal to:

(c) [3 sin1 () 1 2 ] +

(tan1

3
1

(b) sin1 () + 1 2 +
2
2

(c) (tan1 3 )2 +
1

I=

is

an

arbitrary

(c)
(d)

3
3
3
3
3

4
2
2
2
2

2(+1)2

+
+ log( + 1) +
+ + log( + 1) +
+

+dx is equal to
(a)
(b)
(c)

3
3
3
3
3

+
+ +
+

x1

+C

+1
x1

+1
x+1
1

+C

+C

(d) None of these


7.

()(+)dx is equal to

QUANTITATIVE APTITUDE CLASS ASSIGNMENT - 13

Page 24

QUANTITATIVE APTITUDE
CLASS ASSIGNMENT 14
Mensuration / Matrices / Determinants

1.

2.

3.
4.

5.
6.

7.

+ +
If [

]=


[
+ ]
+

Find the values of a, b, c, x, y and z


State True / False
(a) The value of determinant is changed if
rows & column and interchanged
(b) If any two rows & column are
interchanged, then the value of
determinant changes by minus sign
only.
(c) If any two rows & Column are identical
then value of determinant is equal to 1.
(d) If each element of row (column) are
zero, the determinant is also equal to 0.
Construct a matrix = [ ] whose
elements are given by +
If A & B are two matrices then which of the
following is true?
(a) ( ) =
(b) ( + ) = + , A and B being of
same order.
(c) () = , K be any scalar
(d) () = .
(e) All of these
Show that the points (, + ), (, + ) and
(, + ) are collinear.
State true or false:
(a) All zero matrix are equal.
(b) All diagonal matrix are square matrix
(c) All identity matrix are scalar matrix
(d) If product of two matrix is zero matrix
then one of the matrix must be 0.
If A, B and C are matrices with orders 3x3, 2x3
and 4x2 respectively, how many of the following
matrices are defined?
(i) 6B
(ii) A +B
(iii) 3BT+ C (iv) AB
T
T
(v) B A
(vi) (CB)
(vii) CBA

8.

Find area of shaded region. (Take

= 22/7)

r
14

9.

The length of a room is double the breadth. The


cost of colouring the ceiling at Rs. 25 per sq.m is
Rs 5,000 and the cost of painting the four walls at
Rs. 240 per sq. m is Rs. 64,800. Find the height of
the room.
10. A well of 11.2 m diameter is dug 8m deep. The
earth taken out has been spread all around it to a
width of 7 m to form a circular embankment. Find
the height of this embankment?
11. What is the value of:
(i) Interior angle and
(ii) Exterior angle of a regular decagon?
12. Find the value of AB in the following figure:
Here, = , = , =

A
6

D
4

13. What is the value of x in the following figure?


Given, = KL is the diameter.

K
60
N
O

14. In a triangle ABC, O is the center of incircle PQR,


BCA = 75 and BAC = 65, find ROQ:

QUANTITATIVE APTITUDE CLASS ASSIGNMENT - 14

Page 25

QUANTITATIVE APTITUDE
CLASS ASSIGNMENT 15
Vector & 3-D

Vector
1.

Vectors (p,q) and (5,1) are parallel if


(a) pq=5
(b) q=5p
(c) p+q=5
(d) p=5q

2.

If a,b,c are three vectors such that a+b=c, then


b is called.
(a) a projection of c
(b) a complement of c
(c) a component of c
(d) None of these

3.

4.

5.

6.

If a=(1,-1) and b=(-2,m) are two collinear


vectors, then m=
(a) 4
(b) 3
(c) 2
(d) 0
If a and b are two non-zero and non-colliners
vectors, then a+b and a-b are
(a) linearly dependent vectors
(b) linearly independent vectors
(c) linearly dependent and independent vector
(d) None of these
If i+2j+3k is parallel to the sum of vectors
3i+j+2k and -2i+3j+k, then is equal to:
(a) 1
(b) -1
(c) 2
(d) -2
If the point A,B,C and D have position vectors
a,2a+b,4a+2b and 5a+4b respectively. Then
the three collinear points are
(a) A, C and D (b) B, C and D
(c) A,B and C
(d) A,B and D

7.

is to be written as
= +4
The vector
= i+j
parallel to
the sum of a vector
perpendicular to
. Then
and a vector

=
(a)
(b)
(c)
(d)

8.

2
2
3
1
1
1
3

( + )
( + )
( + )
( + )

and (2b-c) +(3a+c)


If vectors a+2b+a

+b are parallel, then unit vector in their


direction is
1
(a)
(2 + 3 + 2)
(b)
(c)
(d)

9.

17
1
6
1

( + 2 + )

11
1
3

( + 3 + )

(2 + + 2)

and +
If the version
are orthogonal to each other
+
then the locus of the point (x,y) is
(a) A circle
(b) an ellipse
(c) A parabola
(d) a straight line

10. The projection of the vector +

on the vector + + is
(a)
(c)

5
2
6
16

QUANTITATIVE APTITUDE CLASS ASSIGNMENT - 15

(b) 2
(d)

1
9

9
16

Page 26

3-D
1.

2.

If ,, are the angles which a half ray makes


with the positive directions of the axes, then
(a) 1
(b) 2
(c) 0
(d) -1

5.

6.

7.

(b) ( , 1, )

(c) ( , 1, )

4.

8.

(d) none of these

The distance of the point (1, 2, 3) from the


coordinate axes are A, B and C respectively.
Now consider the following equations.
1. 2 = 2 + 2
2.2 = 2 2
3. 22 2 = 132
(a) 1 only
(b) 1 & 3
(c) 1 & 2
(d) 2 & 3
The ratio in which the join of (2,1,5) and
(3,4,3) is divided by the plane (x+y-z)= 1/2 is
(a) 3:5
(b) 5:7
(c) 1:3
(d) 4:5
The direction ratio of the line OP are equal
and the length OP=. Then the coordinates
of the point P are
(a) (-1,-1,-1)
(b) (3, 3, 3)
(c) (2, 2, 2)
(d) (2, 2, 2)
If a line makes angles and with x-axis
and y-axis respectively, then the angle which
this line subtends with z-axis is
(a) 35
(b) 45
(c) 55
(d) 90
If A=(1,2,3), B=(-1,0,2), C=(1,-1,2) and D is the
foot of the perpendicular from A on BC. Then
BD:
(a) 3:2
(b) 2:3

(d) 1:3

The circumcentre of the triangle formed by


the points (1,0,0), (0,1,0) and (0,0,1) is
(a) the origin
(b) (1,1,1)
1 1 1

The point equidistant from the points


(0,0,0),(1,0,0),(0,2,0) and (0,0,3) is
(a) (1, 2, 3)

3.

(c) 3:1

(c) ( , , )

(d) (-1,-1,-1)

3 3 3

9.

The angle between the lines (x=1,y=2) and


(y=-1,z=0) is
(a) 90
(b) 30

10. The angle between the lines whose direction


ratios are 1,1,2 and -1,--1,4 is
(a) 45
(b) 30

(c) 60
(d) 90
11. The angle between the straight lines
+

and

(a) 45
(c) 60

is

(b) 30
(d) 90

12. The coordinates of A,B,C,D are (3,5,-3), (2,3,1), (1,2,3) and (3,5,7) respectively. Then
(a) AB is perpendicular to CD
(b) AB is parallel to CD
(c) The angle between AB and CD is /3
(d) The angle between AB and CD is 2/3
13. The angle between a line with direction
ratios 2:2:1 and a line joining (3,1,4) to
(7,2,12) is
2

(a) cos 1 ( )

(b) cos 1 ( )

(c)

(d) none of these

3
2
1 ( )
3

14. The angle between the lines whose direction


cosines satisfy the equations
L+m+n=0 and 2lm+2nl-mn=0 is
(a) 60
(b) 90

(c) 120
(d) 150

QUANTITATIVE APTITUDE CLASS ASSIGNMENT - 15

Page 27

LOGICAL REASONING CLASS ASSIGNMENT - 1


Logical Venn Diagrams, Venn Diagrams, Seating Arrangement

Logical Venn Diagrams


These questions are based on the following
figures.
Study the figures carefully and then answer the
questions given below it.

(a)

(b)

(c)

(d)

1. Mothers, Women, Divorcees


2. Historian, Scientist, Economist
3. Travelers, Train travellers, Bus travellers
4. Donkey, Pet, Horse

(e)

1.

2.

Venn Diagrams

In a certain region, the number of children who


have been vaccinated against rubella is twice
the number who have been vaccinated against
mumps. The number who have been
vaccinated against both is twice the number
who have been vaccinated only against mumps.
If 5,000 have been vaccinated against both,
how many have been vaccinated only against
rubella?
(a) 2,500
(b) 7,500
(c) 10,000
(d) 15,000
(e) 17,500

Passed in all the subjects


Failed in all the subjects
Failed in Chemistry
Failed in Physics
Failed in Mathematics
Passed in Chemistry only
Passed in physics only
Passed in Mathematics only

197
70
170
210
192
64
51
46

3.

How many failed in Chemistry only?


(a) 84
(b) 49
(c) 3
(d) 100

In a class test consisting of mathematics and


physics, 80% passed in mathematics and 50%
passed in physics. 15% failed in both the
subjects. If 180 students passed in both the
subjects, how many failed in both the subjects?
(a) 80
(b) 60
(c) 90
(d) 50

4.

How many failed in one subject only?


(a) 40
(b) 161
(c) 237
(d) 70

5.

How many passed in at least one subject?


(a) 197
(b) 398
(c) 70
(d) 271

Direction for Questions 3 to 5: Answer the


questions based on the following information.
Data on the 468 students, who took an
examination
Chemistry,
Physics
and
Mathematics, is as follows:

6.

30% people of Saharanpur like cold weather,


40% like summer, 55% like autumn. 18% like
cold and summer, 20% like summer and

LOGICAL REASONING CLASS ASSIGNMENT - 1

Page 28

of the employers in Sun Metals are general


graduates and 75% of the engineers earn more
than Rs 5 lakhs per year. If 50% of the
organizations employees earn more than 5
lakhs per year, what proportion of general
graduates employed by the organization earn
Rs 5 lakhs or less?
(a) 3/5
(b) 3/4
(c) 1/2
(d) none of these

autumn, 12% like cold and autumn. 10% like all


three weathers.
(a) How many like none of the weathers
(b) How many like only cold
(c). How many like only cold and summer.
(d). How many like cold and autumn.
(e). How many like atleast one
(f). How many like only one
(g). How many like at least two.
7.

One hundred and twenty-five aliens descended


on a set of film on extra-terrestrial beings. Of
these, 40 had two noses, 30 had three legs, 20
had four ears, 10 had two noses and three legs,
12 had three legs and four ears, 5 had two
noses and four ears, and 3 had all the three
unusual features. How many were there
without any of these unusual features?
(a) 5
(b) 35
(c) 80
(d) None of these

8.

There are two types of employees in Sun


Metals, general graduates and engineers. 40%

9.

Of 30 students who took the GMAT, 14 had


been out of school for at least 3 years, 18 had
business degree, 3 had been out of school for
less than 3 years and did not have business
degrees. How many students had been out of
school for at least 3 years and had degrees in
business
(a) 14
(b) 13
(c) 9
(d) 7
(e) 5

Seating Arrangement
Linear Arrangement
Direction for questions 10 to 12: Read the
following information carefully and answer the
questions given below:
(i) A, B, C, D, E, F, G and H are standing in a row
facing North.
(ii) B is not adjacent to G.
(iii) F is to the immediate right of G and is also
adjacent to E.
(iv) G is not at either extreme.
(v) A is sixth to the left of E.
(vi) H is sixth to the right of C.
10. Who among the following are not adjacent to
each other?
(a) AC
(b) DG
(c) FH
(d) CB
11. Which one among the following defines the
position of D?
(a) Fifth to the left of H
(b) Third to the right of C
(c) Neighbour of B and E

(d) To the immediate right of B.


12. Which of the following is true?
(a) C is to the immediate left of A
(b) G is to the immediate Right of D
(c) Both (a) and (b)
(d) None of these
There are ten lights in a row on a control board.
The lights are numbered1 to 10in sequence
from left to right. Each light is either on or off.
The following is also known:
No two consecutively numbered lights are on.
No three consecutively numbered lights are off.
Exactly four of the ten lights are on.
Light 3 is off.
13. If light 4 is off, which of the following must be
true?
(a) Light 1 is on
(b) Light 2 is off
(c) Light 5 is off
(d) Light 6 is off
(e) Light 7 is on

LOGICAL REASONING CLASS ASSIGNMENT - 1

Page 29

14. If lights 9 and 10 are off, which of the following


must be true?
(a) Light 1 is on
(b) Light 2 is off
(c) Light 4 is on
(d) Light 5 is on
(e) Light 6 is off

15. If light 6 is on and light 8 is off, for how many of


the lights can on-off status be determined?
(a) 4
(b) 5
(c) 6
(d) 7
(e) 8

Circular Arrangement
Direction for Q.16 to Q.18: Six friends Ajay,
Rohit, Sony, Donald, Mickey and Tom are
sitting around a circular table facing the center?
1. Tom is sitting in between Mickey and Ajay.
2. Sony is sitting in between Ajay and Rohit.
3. Mickey is on left of Donald.
16. Who is sitting to the left of Rohit?
(a) Ajay
(b) Sony
(c) Donald
(d) Mickey
(e) None of these
17. Which person is sitting to the right of Sony?
(a) Ajay
(b) Rohit
(c) Donald
(d) Mickey
(e) Tom
18. Which of the condition if removed will doesnt
effect to solve earlier questions?
(a) 1
(b) 2
(c) 3
(d) All of these
(e) None of these
Direction for Q.19 to Q.22: Six men are sitting
around a round table facing its centre. They are

A, B, C, D, F and G. F is not between B and D but


some other one. A is next to the left of G, C is
fourth to the right of A
19. Which of the following is NOT true?
(a) D is just next to the right of C
(b) F is just next to the right of G
(c) C is second to the left of F
(d) A is second to the right of C
20. If A and C interchange their positions then
which of the following pair will sit together?
(a) CF
(b) AG
(c) GC
(d) BG
21. What is the position of F?
(a) Just next to the right of B
(b) Second to the left of A
(c) Between B and C
(d) To the immediate right of G
22. Which one is sitting diagonally opposite to G?
(a) B or D
(b) C
(c) F
(d) A

LOGICAL REASONING CLASS ASSIGNMENT - 1

Page 30

LOGICAL REASONING CLASS ASSIGNMENT - 2


Syllogism, Series, Data Interpretation - I

Syllogism
1.

2.

Treat the statement(s) to be true and verify


the conclusions for the following:
Statement 1: All jets are fast.
Statement 2: All fast are exciting.
Conclusion 1: All jets are exciting.
Conclusion 2: All exciting are jets.
Conclusion 3: Some exciting are jets.
Statement 1: All jets are fast.
Statement 2: All jets are exciting.
Conclusion 1: All fast are exciting.
Conclusion 2: All exciting are fast.
Conclusion 3: Some fast is exciting.

3.

Statement 1: All jets are fast.


Statement 2: Some fast is exciting
Conclusion 1: Some exciting are fast.
Conclusion 2: Some exciting are jets.
Conclusion 3: All exciting are fast.
Conclusion 4: Some fast is jet.

4.

Statement 1: Some jets are fast.


Statement 2: Some fast is exciting.
Conclusion 1: Some exciting are fast.
Conclusion 2: Some exciting are jets.
Conclusion 3: All exciting can be fast.

5.

Statement 1: Some jets are fast.


Statement 2: No fast is exciting.
Conclusion 1: No jet is exciting.
Conclusion 2: Some jets are not exciting.
Conclusion 3: Some exciting are not jets.

6.

Statement 1: All jets are fast.


Statement 2: No fast is exciting.
Conclusion 1: No jet is exciting.
Conclusion 2: No exciting is jet.

7.

Statement 1: All jets are fast.


Statement 2: No jet is exciting.
Conclusion 1: No exciting is a fast.

Conclusion 2: No exciting is jet.


Conclusion 3: Some fast are not exciting.
Conclusion 4: Some exciting are not fast.
8.

Statement 1: No jet is fast.


Statement 2: No fast is exciting.
Conclusion 1: No jet is emotion.
Conclusion 2: Some jets are not exciting.
Conclusion 3: Some jets are exciting.

Practice Exercise
9.

Statements:
(i) All knives are hammers.
(ii) No hammer is sword.
(iii) Some swords are nails.
Conclusions:
I. Some nails are hammers.
II. Some swords are knives
III. No nail is hammer.
Chose the correct option:
(a) None follow
(b) Only either I or III follow
(c) Only II follow
(d) Only III follow
(e) Only I follow

10. Statements:
(i) All hills are roads.
(ii) All roads are stones.
(iii) All stones are jungles.
(iv) All jungles are rivers.
Conclusions:
I. Some rivers are stones.
II. Some Jungles are hills.
III. Some stones are hills.
IV. All rivers are jungles.
Chose the correct option:
(a) Only I and II follow
(b) Only II and III follow
(c) Only I, II and III follow

LOGICAL REASONING CLASS ASSIGNMENT - 2

Page 31

(d) Only II, III and IV follow


(e) All follow

13. Statements:
Some elephants are rats. All rats are flowers.
Some flowers are not elephants.
Conclusions:
(I) Some flowers are elephants.
(II) All elephant are flowers.
(III) All rats are elephants
(IV) No flower is an elephant
Chose the correct option
(a) Only I follows
(b) Only I and II follow
(c) Only II and III follow
(d) None follows

11. Statements:
(i) Some apple is bat.
(ii) No bat is cap.
Conclusions:
(i) Some apple is cap.
(ii) No apple is cap.
Chose the correct option:
(a) Only (i) follows
(b) Only (ii) follows
(c) Either (i) or (ii) follows
(d) None follows
12. Statements:
(i) All drivers are mechanics.
(ii) All mechanics are cleaners
Conclusions:
(i) All cleaners are drivers.
(ii) Some drivers are mechanics.
(iii) All drivers are cleaners.
(iv) Some cleaners are mechanics.
Chose the correct option:
(a) All conclusions follow
(b) Only (ii) (iii) (iv) follow
(c) Only (iii) (iv) follow
(d) Only (ii) (iii) follow

14. Statements:
All pens are papers. All papers are aeroplanes.
All aeroplanes are skies.
Conclusions:
(I) All pens are skies
(II) All skies are papers
(III) All aeroplanes are pens
(IV) All papers are skies
Chose the correct option
(a) Only I follow
(b) Only I and III follow
(c) Only I and IV follow
(d) Only III and IV

SERIES
Numeric Series
1.
2.
3.
4.
5.
6.
7.
8.
9.
10.

2, 4, 7, 11, 16, 22, __


720, 504, 336, 210, __
86, 78, 87, 77, 88, ___ , ____
21, 15, 6, 25, 11, 3, 29, 7, 0, __
1, 5, 17, 37, 65, ___
2, 6, 12, 20, 30, 42, ____
0, 6, 24, 60, 120,
11, 41, 161, 641 _____
5, 15, 60, 300, ____
3505, 700, 135, ___

Alphabet and Alphanumeric series


1.
2.

A, Z, B, Y, C, X, D, __
E5V, J10Q, O15L, T20G, Y25B___

3.
4.

ND, IY, DT, YO, TJ,__


abb _ baa _ a _ bab _ aba
(a) abba
(b) abab
(c) ccac
(d) aabb

Practice Questions
1.
2.
3.
4.
5.
6.
7.

3, 8, 11, 19, 30, 49, __


0, 2, 3, 10, 15, 26, 35, 50, 63, __
2, 7, 20, 61, 182, 285__
RQP, ONM, LKJ, __ , FED
-3, -2, 5, 24, 61, 122, ____
36, 9, 27, 6, 21, 3, ___
0, 0, 4 , 18, 48, 100, 180, __

LOGICAL REASONING CLASS ASSIGNMENT - 2

Page 32

Data Interpretation - I
Directions for Q1 to Q3 : Study the data of 19 companies and answer the questions (all figures in Rs. Crore)

1.

How many companies have the same


rank by sales, and by profit (if arranged
in their decreasing order) in the last year
for which data is available?
(a) None
(b) 1
(c) 2
(d) 3

2.

If the companies are arranged on the


basis of total assets in decreasing order
in 2000-01 and similar lists are prepared
based on sales and profit parameters
also for the same year, how many
companies will be among top ten in all
the three lists?
(a) 6
(b) 7
(c) 8
(d) 9

3.

Which of the following statements is not


true?
(a) All the companies which have decline
in sales in 2001-02 have experienced
decline in the profit also.
(b) Top two companies by avg mkt cap
(AMC) have combined AMC
exceeding 25 times combined AMC
of bottom two companies by this
parameter.
(c) In 2001-02, 13 companies saw their
sales as well as profit going up.
(d) There are 5 companies with total
assets exceeding 1000 lakh in 200001.

LOGICAL REASONING CLASS ASSIGNMENT - 2

Page 33

Post Class Assignment


Question 1-10
In each question, three (or two) statements
are followed by four conclusions I, II, III, IV.
You have to take the given statements to be
true even if they appear to be at variance
with commonly known facts, and then
decide which of the conclusions logically
follow(s) from the given statements. For
each question, mark out an appropriate
answer choice that you think is correct.
1.

2.

Statements
1. All small are books.
2. Some books are windows.
3. No windows is pencil.
Conclusions:
I. Some windows are small.
II. Some books are pencils.
III. No window is small.
IV. Some books are not pencils.
Chose the correct option
(a) I and either II or IV follow
(b) I, III and IV follow
(c) I, II and III follow
(d) IV follows.
Statements:
1. Some blacks are Canadians.
2. Some Indian are black.
3. No Indian is white.
Conclusions:
I. Some Canadians are Indian.
II. Some whites are blacks.
III. Some whites are not black.
IV. Some blacks are not whites.

2. Some goats are flowers.


3. No chocolates are boxes.
Conclusions:
I. Some flowers are chocolates.
II. No goats are chocolates.
III. Some flowers are boxes.
IV. Some flowers are not chocolates.
Chose the correct option
(a) II, III and IV follow
(b) I, III and IV follow.
(c) Only II and III follow
(d) Only III and either I or IV follow
4.

Statements:
1. Some cookies are papers.
2. All papers are chocos.
3. Some chocos are trams.
Conclusions:
I. Some cookies are trams.
II. Some papers are trams.
III. Some trams are papers.
IV. Some chocos are cookies.
Chose the correct option
(a) Only II and III follow
(b) I and IV follow
(c) Only IV follows
(d) II, III and IV follow

5.

Statements:
1 No foods are mangoes.
2. No oranges are potatoes.
3. All mangoes are oranges.
Conclusion:
I. Some foods are not oranges.
II. Some oranges are not foods.
III. No mangoes are potatoes.
IV. Some oranges are foods.
Chose the correct option
(a) Only III and I follow
(b) Only II and IV follow
(c) Either II or IV follow
(d) Only II and III follow.

6.

Statements:
1. All coats are paints.

Chose the correct option


(a) I and IV follow.
(b) II and III follow.
(c) Only III follow.
(d) Only IV follow.
3.

Statements:
1. All goats are boxes.

LOGICAL REASONING CLASS ASSIGNMENT - 2

Page 34

2. No paints are shirts.


3. Some shirts are vests.
Conclusions:
I. Some vests are not shirts.
II. Some coats are shirts.
III. No coat is a shirt.
IV. Some vests are not coats.
Chose the correct option
(a) I, II and III follow
(b) I and IV follow.
(c) I and III follow
(d) III and IV follow.
7.

1. Some stars are birds.


2. No birds are elephants.
Conclusions:
I. Some stars are not elephants.
II. All stars are elephants.
III. Some elephant are not stars.
IV. All elephants are stars.
Chose the correct option
(a) Either I or II and either III or IV follow
(b) Only I follows
(c) Either III or IV follows
(d) I and III follow
(e) I and either III or IV follow

Statements:
1. All boxes are tables.
2. No desks are tables.
3. Some desks are curtains.
Conclusions:
I. No boxes are desks.
II. Some boxes are desks.
III. Some curtains are not boxes.
IV. Some curtains are boxes.
Chose the correct option
(a) III and either I or II follow
(b) I and either III or IV follow
(c) Either I or II and either III or IV follow
(d) I and III follow
(e) None of the above

8.

Statements:
1. Some trees are sharks.
2. Some sharks are balloons.
Conclusions:
I. All balloons are sharks.
II. All balloons are trees.
III. Some trees are balloons.
IV. No tree is a balloon.
Chose the correct option
(a) Only I follows
(b) Only II and III follow
(c) Only either I or III follows
(d) Only either I or IV follows
(e) Only either III or IV follows

9.

Statements:

10. Statements:
1. Some copies are desks.
2. No desks are pens.
Conclusions:
I. Some copies are pens.
II. Some copies are not pens.
III. Some pens are desks.
IV. Some pens are not desks.
Chose the correct option
(a) Only II follows
(b) II and III follow
(c) Either I or II follows
(d) II and IV follow
(e) Either III or IV follows

LOGICAL REASONING CLASS ASSIGNMENT - 2

Page 35

LOGICAL REASONING CLASS ASSIGNMENT 3


Analytical Reasoning, Data Interpretation - II

Analytical Reasoning
Directions: Read the text and the statements
carefully and answer the questions.
Four people of different nationalities live on
the same side of a street in four houses, each of
different colour. Each person has a different
favourite drink. The following additional
information is also known:
(i) The Englishman lives in the red house.
(ii) The Italian drinks tea.
(iii) The Norwegian lives in the first house on the left
(iv) In the second house from the right, they drink
milk
(v) The Norwegian lives adjacent to the blue house.
(vi) The Spaniard drinks fruit juice
(vii) Tea is drunk in the blue house.
(viii) The white house is to the right of the red house.

5.

Q is Ps
(a) Father
(c) Son

(b) Mother
(d) Grandmother

S is Ps
(a) Brother
(c) Mother

(b) Father
(d) Sister

R is Qs
(a) Daughter
(c) Grandson

(b) Son
(d) Daughter- in- Law

R is Ss
(a) Wife
(c) Husband

(b) Son
(d) Daughter

A.

1.

2.

3.

Milk is drunk by:


(a) Norwegian
(c) Italian

(b) Englishman
(d) None of these

The Norwegian drinks:


(a) Milk
(c) Tea

(b) Cocoa
(d) Fruit Juice

Which of the following is not true?


(a) Milk is drunk in the red house
(b) Italian lives in the blue house
(c) The Spaniard lives in a corner house
(d) The Italian lives next to the Spaniard

B. P, Q, R, S live together in a house


(i) P lives with his/her parents
(ii) Q lives with at least 3 persons younger than
him/her
(iii) S lives with his mother, and is older than at least
2 people living with him
(iv) R lives with his/her son and is not older than S
4.

Total number of persons in the house is


(a) 3
(b) 4
(c) 5
(d) 6

6.

7.

8.

C.

Six scientists A, B, C, D, E and F are to present a


paper each at a one-day conference. Three of
them will present their paper in the morning
session before the lunch break while the other
three would present it in the afternoon session.
The lectures have to be scheduled in such a
way that they comply with the following
restrictions
(i) B should present his paper immediately before
Cs presentation; their presentations cannot be
separated by the lunch break
(ii) D must be either the first or the last scientist to
present his paper

9.

In case C is the fifth scientist to present his


paper then B must be
(a) First
(b) Second
(c)Third
(d) Fourth

10. B could be place in any of the places in the


order of presenters EXCEPT
(a) First
(b) Second
(c) Third
(d) Fourth
11. In case F is to present his paper immediately
after D presents his paper, then C could be

LOGICAL REASONING CLASS ASSIGNMENT - 3

Page 36

scheduled for which of the following places in


the order
(a) Second
(b) Third
(c) Fourth
(d) Fifth
12. In case F and E are fifth and sixth presenters
respectively then which of the following must
be true?
(a) A is first in the order
(b) A is third in the order
(c) A is fourth in the order
(d) B is first in the order
D.

A whole seller dealing in readymade garments


sent a consignment of two brands of shirts to a
retailer by goods train. However, the
consignment was tampered in transit and the
retailer refused to accept it. Meanwhile, whole
seller also lost the details of consignment in a
fire in his accounting office. Later on only
following data about number of T-shirts sent
could be recovered:

A frenetic search for more data revealed the


following facts about consignment:
*60% of the shirts were of size Large or Extra
Large
*Brand B shirts formed 45% of the consignment
*50% of Brand A shirts were of Medium size
13. What percentage of the Brand B shirts are of
size L?
(a) 37%
(b) 25%
(c) 50%
(d) 33%
14. What percentage of Large size shirts belong to
Brand A?
(a) 49%
(b) 42%
(c) 52%
(d) 30%
15. How many Brand A shirts are of Large size?
(a) 91
(b) 94
(c) 63
(d) None of these

SELECTION BASED ON GIVEN CONDITIONS


E.

Direction: Shahrukh Khan in Chak De!! wants to


build a composite team of six members for next
Olympics. He has with him six men (Dhanraj,
Deepak, Deevesh, Ramandeep, Mohd. Riaz, and
Barla) and five girls (Bindia Naik, Nethra Reddy,
Preeti Sabharwal, Komal Chautala and Vidya
Sharma).
Following things need to be taken care of:
1. Dhanraj and Ramandeep have to be
together.
2. Deevesh will not like to be in a team with
Komal Chautala.
3. Komal Chautala and Vidya Sharma have to be
together.
4. Deepak does not like to be in team with
Mohd. Riaz.
5. Ramandeep will not like to go with Bindia
Naik.
6. Deepak and Preeti Sabharwal have to be
together.
7. Deevesh and Nethra Reddy have to be
together.

16. If Sharukh Khan chooses five males in team,


then which fortunate lady got into the team?
(a) Bindia Naik
(b) Nethra Reddy
(c) Preeti Sabharwal
(d) Komal Chautala
(e) None of these
17. If including Bindia Naik, the team has three
ladies, then how is the team composed?
(a) Deepak, Deevesh, Barla, Nethra Reddy &
Preeti Sabharwal
(b) Dhanraj, Ramandeep, Mohd. Riaz, Komal
Chautala, Vidya Sharma
(c) Dhanraj, Ramandeep, Deepak, Komal
Chautala, Vidya Sharma
(d) Deepak, Barla, Preeti Sabharwal, Komal
Chautala, Vidya Sharma
(e) None of these
18. If including Deevesh the team has four male
members, then what are the other members of
the team?
(a) Dhanraj, Ramandeep, Deepak, Nethra Reddy,
Preeti Sabharwal
(b) Dhanraj, Ramandeep, Mohd. Riaz, Bindia
Naik, Nethra Reddy

LOGICAL REASONING CLASS ASSIGNMENT - 3

Page 37

(c) Barla, Ramandeep, Mohd. Riaz, Dhanraj,


Nethra Reddy
(d) Deepak, Mohd. Riaz, Barla, Preeti Sabharwal,
Nethra Reddy
(e) None of these
19. If four members of the team have to be girls,
then the members of the team are?
(a) Dhanraj, Barla, Komal Chautala, Bindia Naik,
Nethra Reddy, Preeti Sabharwal
(b) Deepak, Bindia Naik, Ramandeep, Nethra
Reddy, Komal Chautala, Vidya Sharma
(c) Deepak, Komal Chautala, Vidya Sharma,
Deevesh, Nethra Reddy, Preeti Sabharwal
(d) Deepak, Barla, Komal Chautala, Bindia Naik,
Vidya Sharma, Preeti Sabharwal
(e) None of these

F.

COMPARISON TYPE QUESTIONS


Direction: Five brands- Tata, Maruti, Honda,
Hyundai and General Motors are to be rated.
1. Tata is rated higher than Maruti but rated
lower than General Motors.
2. Honda is rated lowest.

3. Hyundai is rated higher than Maruti and little


lower than Tata.
20. Which brand is rated the highest?
(a) Tata
(b) Maruti
(c) Honda
(d) Hyundai
(e) General Motors
21. Which brand was rated in the middle?
(a) Tata
(b) Maruti
(c) Honda
(d) Hyundai
(e) General Motors
22. Which brand is rated second?
(a) Tata
(b) Maruti
(c) Honda
(d) Hyundai
(e) General Motors
23. Which brand is rated at the fourth place?
(a) Tata
(b) Maruti
(c) Honda
(d) Hyundai
(e) General Motors

LOGICAL REASONING CLASS ASSIGNMENT - 3

Page 38

Data Interpretation - II
The following pie-chart shows the percentage distribution of the expenditure incurred in publishing a
book. Study the pie-chart and the answer the questions based on it. Various Expenditures (in
percentage)

1. If for a certain quantity of books, the


publisher has to pay Rs. 30,600 as printing cost,
then what will be amount of royalty to be paid
for these books?
(a) Rs. 19,450
(b) Rs. 21,200
(d) Rs. 22,950
(d) Rs. 26,150

3. The price of the book is marked 20% above


the C.P. If the marked price of the book is Rs.
180, then what is the cost of the paper used in
a single copy of the book?
(a) Rs. 36
(b) Rs. 37.50
(c) Rs. 42
(d) Rs. 44.25

2. What is the central angle of the sector


corresponding to the expenditure incurred on
Royalty?

4. If 5500 copies are published and the


transportation cost on them amounts to Rs.
82500, then what should be the selling price of
the book so that the publisher can earn a profit
of 25%?
(a) Rs. 187.50
(b) Rs. 191.50
(c) Rs. 175.00
(d) Rs. 180.00

(a) 15
(c) 54

(b) 24
(d) 48

LOGICAL REASONING CLASS ASSIGNMENT - 3

Page 39

LOGICAL REASONING CLASS ASSIGNMENT 4


Data Sufficiency, Family Relations, Coding-Decoding, DI III

Data sufficiency
Direction for Q. 1-10: Each of the following
consists of a question and two statements
numbered I and II given below it. You have to
decide whether the data provided in the
statements are sufficient to answer the
question. Read both the statements and give
the answer.
(a) Statement (I) alone is sufficient, but
statement (II) alone is not sufficient.
(b) Statement (II) alone is sufficient, but
statement (I) alone is not sufficient.
(c) Both statements together are sufficient, but
neither statement alone is sufficient.
(d) Each statement alone is sufficient.
(e) Both statements are insufficient.
1.

2.

At Delhi Metro Rail Corporation, the average


age of an executive employee is 50 years and
the average age of non-executive employee is
30 yrs. What is the average age of the
employees at Delhi Metro Rail Corporation?
(I) There are 100 executive employees.
(II) The number of non-executive employees is
20 times the number of executive
employees.
Among five boys A, B, C, D, E who is the
youngest?
(I) D is younger to both A and C
(II) B is elder to E but younger to D

3.

If Sanyas age is exactly twice of Peters age,


what is Sanyas age?
(I) Four years ago, Sanyas age was exactly three
times Peters age.
(II) Eight years from now, Sanyas age will be
exactly 1.5 times Peters age.

4.

On a straight road, car X and Y are traveling at


different constant rates. If car X is now 2 km
ahead of car Y, how much time for now will car
X be 4 km ahead of car Y?

(I) Car X is travelling at 45 km/hr and car Y is


traveling at 30 km/hr
(II) 6 minutes ago, car X was km ahead of car
Y.
5.

What are the values of X and Y


(I) Y is an even integer, X is an odd integer and X
is greater than Y
(II) Product of X and Y is 30

6.

Is x<0?
(I) 2 > 0
(II) 3 < 0

7.

The symbol represents one of the following


operations:
addition,
subtraction,
multiplication, or division. What is the value of
3 2?
(I) 0 1=1
(II) 1 0=1

8.

How much did a certain telephone call cost?


(I) The call lasted 53 minutes.
(II) The cost for the first 3 minutes was 5 times
the cost for each additional minute.

9.

If n is a positive integer; is odd?


(I) 3 is odd
(II) + 3 is even

10. A total of 9 women and 12 men reside in the 21


apartments that are in a certain apartment
building, one person to each apartment. If a
poll taker is to select one of the apartments at
random, what is the probability that the
resident of the apartment selected will be a
woman who is a student?
(I) Of the women, 4 are students.
(II) Of the women, 5 are not students

LOGICAL REASONING CLASS ASSIGNMENT - 4

Page 40

Family Relationship
Type 1

Type 2

1.

X introduces Y saying, he is the husband of the


granddaughter of the father of my father. How
is Y related to X.
(a) Husband
(b) Brother
(c) Brother in law
(d) Data insufficient

2.

Deepak said to Nitin, That boy playing the


football is the younger of the two brothers of
the daughter of my fathers wife. How is the
boy playing football related to Deepak?
(a) Brother
(b) Cousin
(c) Brother in Law
(d) data insufficient

Read the following information carefully and


answer the questions that follow:
P & Q means P is the father of Q.
P * Q means P is the sister of Q.
P + Q means P is the brother of Q.
P - Q means P is the mother of Q.
P % Q means P is the son of Q.
P = Q means P is the daughter of Q.

3.

Ravi introduced X as X is the grandson of the


paternal grandfather of my sons son. How is X
related to Ravi?
(a) Grandson
(b) Some other distant relation
(c) Neither a nor b
(d) a or b

1.

Which of the following choices mean that X is


the maternal uncle of Z?
(a) X % Y Z
(b) X = Y % Z
(c) X + Y Z
(d) X * Y + Z

2.

Which of the following means that X is the


nephew of Z?
(a) X Y % Z
(b) X + Y * Z
(c) X % Y Z
(d) X % Y * Z

Coding- Decoding
1.

The code TABLESTESF stands for the word


BELONGINGS how will you code the following:
(1) LONG
(2) ON
(3) GIN
(4) SONG
(5) NO

2. If POSTED is coded as DETSOP, how will be


word SPEED be coded?
3. KAPIL is coded as IFMXH, then PATHAN is
coded as?
4. RETAISH is coded as 80, then how is OXFORD
coded
5. If tera mera sakba means lets enjoy school,
jaisa mera waisa means enjoy before work,
kuch is tera means school only sunday then
what does sabka denote?

LOGICAL REASONING CLASS ASSIGNMENT - 4

Page 41

Data Interpretation - III


1. Convert the following percentage into
fraction
(a) 12.5% - ______
(b) 37.5% - ______
(c) 67.5% - ______
(d) 16.67% ______
(e) 55.55% - _____
(f) 54.54% -______
(g) 09.09% - ______
(h) 90.90%- _____
(i) 3.125% -______
(j) 7.69230% -____
Refer to the table below and answer the
questions that follow
No. of students registered for
JET 2010
Subject

1st year

Science
History
Maths
Economics

20
30
40
50

2nd
year
40
60
80
70

3rd
year
80
120
160
100

2. No. of students registered for History for


all the three years is less than the number
of students registered for Maths for all
the three years by
(a) 12.5%
(b) 17.5%
(c) 25%
(d)22.5%
3. The total registration fees collected from
the students of 1st year for Science,
History, Maths and economics is in the
ratio 3:2:3:1. If the total registration fee
collected is Rs 135000, then what is the
registration fee for Economics (in Rs)?
(a) 350
(b) 300
(c) 250
(d) 287.5
4. Assuming the data for Q3, what is the
total registration fee collected for Maths
in 1st year?

(a) 45000
(c) 50500

(b) 47500
(d) 50725

5. If maximum 40 students can sit in a room


then what is the minimum number of
classrooms required to run the classes
effectively?
(a) 25
(b) 24
(c) 22
(d) 23
Directions for Q6-Q9: The proportion of male
students and the proportion of vegetarian
students in a school are given below. The
school has a total of 800 students, 80% of
whom are in the secondary section and rest
equally divided in class 11 and 12. JET 2011
Male
Vegetarian
Class 12
0.60
_____
Class 11
0.55
0.5
Secondary
_____
0.55
section
Total
0.475
0.53
6. What percentage of male students are in
the secondary section?
(a) 40
(b) 45
(c) 50
(d) 55
7. What is the percentage of vegetarian
students in class 12?
(a) 60
(b) 55
(c) 50
(d) 40
8. In the secondary section, 50% of the male
are vegetarian. Which of the following
statements is correct?
(a) Except vegetarian males, all other
groups have same number of students
(b) Except non-vegetarian males, all other
groups have same number of students

LOGICAL REASONING CLASS ASSIGNMENT - 4

Page 42

(c) Except vegetarian females, all other


groups have same number of students
(d) Except non-vegetarian females, all
other groups have same number of
students
9.

In class 12, 25% of the vegetarians are


male. What is the difference between
the number of female vegetarians and
number of male non-vegetarians?
(a) 16
(b) 14
(c) 12
(d) 10

Directions for Q10 to Q13: Answer the


questions based on the following
hypothetical information
BMS 2013
Profile of Parliamentarians in year 20XX
Lok Sabha
Rajya
Sabha
Party Support
UPA
371
155
supporters
NDA Suporters 181
95
552
250
Gender
Male
530
250
Female
22
0
Age (in years)
Youngest
34
40
Oldest
84
89
Average
48
55
Religion
Hindu
323
173
Muslim
136
30
Sikhs
23
13
Christian
5
8
Others
65
26

Profession
Lawyer
Businessman

Lok Sabha

Rajya
sabha

273
103

157
38

Educator
Farmer
Scientist
Doctors
Unskilled
Social Workers
Govt. Officials
Others
Nominated
Anglo-Indian
Experts

57
18
0
3
8
30
28
32

15
15
5
0
0
10
0
10

2
0

0
12

10. In the Rajya Sabha, if 28 Hindu members


were replaced by 28 Christian members,
the approx. ratio of Hindu members to
Christian members would be
(a) 4:1
(b) 5:1
(c) 3:2
(d) 9:2
11. If 18 members of Rajya Sabha are
Muslim supporting UPA, how many
members of Rajya Sabha are neither
Muslim nor supporters of UPA?
(a) 95
(b) 167
(c) 83
(d) 65
12. Approximately what percentage of the
members of Parliament are lawyers?
(a) 49%
(b) 54%
(c) 63%
(d) 56%
13. Which of the following can be inferred
from the information given in the Table?
(I) The percent of members who are
Educators is greater for the Rajya
Sabha than for the Lok sabha
(II) More than 2/3rd of men in Parliament
are members of Lok Sabha
(a) I only
(b) II only
(c) both I and II
(d) None of these

LOGICAL REASONING CLASS ASSIGNMENT - 4

Page 43

LOGICAL REASONING CLASS ASSIGNMENT 5


Miscellaneous Reasoning, Calendars, Cubes, Clocks

Direction Test
1.

2.

A starts from his office and walks 3 km


towards north. He then turns right and
walks 2 km and then turns right and walks
5 km. He then turns right and walks 2 km
and then again turns right and walks 2
km. In which direction is he from the
starting point?
(a) South
(b) North-east
(c) South-east
(d) he is at the starting point
(e) None of these

anticlockwise direction. Which direction


is
he
facing
now?
(a) North
(b) East
(c) North-East
(d) North-West
(e) None of these
3.

A man is facing north. He turns 45 in the


clockwise direction and then 90 in the

Anurag travels 20 km to the north, turns


left and travels 8 km and then again turns
right and covers another 10 km and then
turns right and travels another 8 km. How
far is he from the starting point?
(a) 8 km
(b) 40 km
(c) 30 km
(d) 20 km
(e) None of these

Nonverbal Reasoning
From the given options 1, 2, 3, 4, & 5, find the option that best completes the given sequence
logically.

1.
A

2.

LOGICAL REASONING CLASS ASSIGNMENT - 5

Page 44

Input / Output
Direction for Q 1 to 4: Observe the given pattern and solve the following questions.
Input: go now 53 39 18 for again 66
Step 1: 66 go now 53 39 18 for again
Step 2: 66 again go now 53 39 18 for
Step 3: 66 again 53 go now 39 18 for
Step 4: 66 again 53 for go now 39 18
Step 5: 66 again 53 for 39 go now 18
Step 6: 66 again 53 for 39 go now 18
Step 7: 66 again 53 for 39 go 18 now
Output: 66 again 53 for 39 go 18 now

1. Input: trees 18 27 desk are 91 hour zero 31 16 chairs.


Which of the following will be step 4?
(a) 91 are 31 trees 18 27 desk hour zero 16 chairs
(b) 91 trees 18 27 desk are hour zero 31 16 chairs
(c) 91 are 31 chairs trees 18 27 desk hour zero 16
(d) 91 are 31 chairs 27 desk 18 trees hour zero 16
(e) None of these

2. Step 2 of an input is: 94 car 86 window shut 52 31 house


Which of the following is definitely the input?
(a) 94 car window 86 shut 52 31 house
(c) car shut window 86 52 31 house 94

(b) 86 window 94 car shut 52 31 house


(d) Cannot be determined

3. Input: show 51 37 now for 82 49 goot


Which of the following steps will be the last but one?
(a) Step 7
(b) Step 8
(c) Step 6

(d) Step 5

4. Step 4 of an input is: 58 box 47 dew 15 21 town pot


Which of the steps will be the last?
(a) Step 7
(b) Step 6

(c) Step 8

(d) Step 9

Direction for Q 5 to 7: A word and number arrangement machine when given an input line of words
and numbers rearranges them following a particular rule in each step. The following is an illustration
of input and rearrangement. (All the numbers are two-digit numbers.)
Input: tall 48 13 rise alt 99 76 32 wise jar high 28 56 barn
Step I: 13 tall 48 rise 99 76 32 wise jar high 28 56 bam alt
Step II: 28 13 tall 48 rise 99 76 32 wise jar high 56 alt barn
Step III: 32 28 13 tall 48 rise 99 76 wise jar 56 alt barn high

LOGICAL REASONING CLASS ASSIGNMENT - 5

Page 45

Step IV: 48 32 28 13 tall rise 99 76 wise 56 alt barn high jar


Step V: 56 48 32 28 13 tall 99 76 wise alt barn high jar rise
Step VI: 76 56 48 32 28 13 99 wise alt barn high jar rise tall
Step VII: 99 76 56 48 32 28 13 alt barn high jar rise tall wise
And Step VII is the last step of the above input, as the desired arrangement is obtained.
As per the rules followed in the above steps, find out in each of the following questions the appropriate
step for the given input.
Input: 84 why sit 14 32 not best ink feet 51 27 vain 68 92
5. Which step number is the following output?
32 27 14 84 why sit not 51 vain 92 68 feet best ink
(a) Step V
(b) Step VI
(c) Step IV
(e) There is no such step.
6. Which word/number would be at 5th position from the right in Step V?
(a) 14
(b) 92
(c) feet
(e) why

(d) Step III

(d) best

7. How many elements (words or numbers) are there between 'feet' and '32' as they appear in the
last step of the output?
(a) One
(b) Three
(c) Four
(d) Five

Cubes
1.
A solid cube is painted Red on all sides, and is then cut into 1000 identical smaller cubes
(a) What is the minimum number of cuts required to do so?
(b) How many of the smaller cubes have their three sides painted
(c) How many of the smaller cubes have their two sides painted
(d) How many of the smaller cubes have their one sides painted
(e) How many of the smaller cubes have their no side painted

Calendars
1.

What was the day on 19th July 2001?

2.

If 3rd October, 2013 was Monday, then what will be the day on:
(a) 3rd October 2031
(b) 3rd October 1991

LOGICAL REASONING CLASS ASSIGNMENT - 5

Page 46

Clocks
1. What is the angle between the hour hand
and the minute hand of an analog clock at
4:35 pm?

2. There are two digital clocks. One is two


minutes faster than the other one. If both the
clocks show the same time at 12:00 PM today,
when will they show the same time together
again for the first time?

LOGICAL REASONING CLASS ASSIGNMENT - 5

Page 47

VERBAL ABILITY CLASS ASSIGNMENT - 1


Subject-Verb Agreement, Phrasal Verbs, Vocabulary

Subject Verb Agreement


DIRECTIONS for questions 1 to 5: Identify the
errors in each sentence and correct them. If there
is no error, then write 'no error'.
1. I and you are going to enjoy this outing.
2. Either of these roads leads to the theatre.
3. She sent word that she would be arriving soon.
4. The minister as well as his team are arriving
tomorrow.
5. Fourteen rupees are a large sum for the migrant
worker.
DIRECTIONS for questions 6 to 12: Fill in the
blanks with the proper form of verb.
1. Bread and butter ____________ his favourite
food. (are, is)
2. French, as well as German, ____________ taught
in the institute. (was, were)
3. There ____________ neither food nor water in
the house. (were, was)
4. Politics ____________ always discussed during his
lecture. (is, are)
5. The cattle ____________ grazing in the meadow
till late evening. (was, were)
6. Neither of my children ____________ to
school.(go, goes)
7. Neither Ram nor his parents ____________
present in the party. (was, were)

Phrasal Verbs
Directions for questions 1 to 5: Fill in the blank
with the appropriate option
1. Please ____________ a card from the box that is
before you.
(a) pick on
(b) pick out

4. The machine is ____________ the needs of share


brokers and their clients.
(a) adapted to
(b) adapted for
5. One can ____________ from the paragraph That
birds are further categorized into various types of
species
(a) infer
(b) imply
Directions for questions 1 to 5: Fill in the
blanks with the most appropriate set of words
from the options.
1. The __________ policies of the new government
seem to throw the economy of the country
____________.
(a) backward, backwards
(b) backwards, backward
2. Did the manager resort to ____________ ways in
order to ____________ such statements from his
secretary?
(a) illicit, elicit
(b) elicit, illicit
3. I have an idea that the employees ____________
the developments in the ____________ appraisal
criteria.
(a) recent, resent (b) resent, recent
4. The ____________ forecast will tell us
____________ to step out of the house today or
not.
(a) whether, weather
(b) weather, whether
5. The ____________ forbids everyone from climbing
the ____________.
(a) statue, statute
(b) statute, statue

2. Why are you ____________ your food? Arent you


hungry?
(a) picking at
(b) picking up
3. If you are yet to think of an excuse, you are ____
(a) done for
(b) done in

Verbal Ability Class Assignment - 1

Page 48

VOCABULARY
Words

Definitions

Aspire (v):

To strive for a goal; to want or desire something excellent or far beyond the
current state.
Having two sides
Honesty or straightforwardness, especially being honest about something
unpleasant.
To cheat; to gain something dishonestly, as through a lie, deception, or
confidence game.
A god or goddess.
To escape from someone or something giving chase or searching; especially to
use clever tactics to escape from pursuers.
A permanency; a person or thing remaining fixed, in the same position.
The character or style of a persons writing by hand; cursive writing.
Chaos or large-scale destruction.
To drink a liquid; to absorb or take in something.
A joint or seam, where two parts meet.
In a plant, a grain or seed; often edible and found inside the plant
The act of falling from a righteous or just position; a slight or temporary moral
failure.
A disease, illness, or medical condition, especially a serious or chronic condition.
Many; a large number; countless.
To make something necessary.
To lay or be enclosed comfortably; to snuggle.
To destroy; to break down into nothing; to eliminate.
To break something down into its parts in order to explain or understand it;
especially to break apart a sentence into words and phrases, describing each
part.
A line; a group of people waiting in order.
Extremely hungry; filled with hunger that cannot be satisfied.
To sink a ship by creating a hole or holes in the ship beneath the water line; also
to destroy or wreck.
A government ruled by the leaders of a religion or by the power of a god or gods.
A world beneath or below the normal world, especially a criminal world such as
mafia society or a world of the dead such as Hades.
To move uncomfortably; to twist around, especially to express pain

Bilateral (adj)
Candor (n):
Defraud (v)
Deity (n)
Elude (v)
Fixture (n)
Handwriting (n)
Havoc (n)
Imbibe (v)
Juncture (n)
Kernel (n)
Lapse (n)
Malady (n)
Myriad (n)
Necessitate (v)
Nestle (v)
Obliterate (v)
Parse (v)

Queue (n)
Ravenous (adj)
Scuttle (v)
Thearchy (n)
Underworld (n)
Writhe (v)

1.
2.
3.
4.
5.

SYNONYMS
In the blank space, write the word from the list
above that means the same (about the same) as
the word given.
Squirm
____________
Starving
____________
Demolish
____________
Countless
`____________
Convergence
____________

6.
7.
8.
9.
10.

Directness
Realize
Require
Devastation
Nether Region

Verbal Ability Class Assignment - 1

____________
____________
____________
____________
____________

Page 49

1.
2.
3.
4.
5.

ANTONYMS
In the space provided, write the word from the
list above that means opposite of the word
given.
Float
___________
One-Sided
___________
Confront
___________
Mortal
___________
Steadfastness
___________

ANALOGIES:
Choose the word from the list that best
completes the analogy, creating the same
relationship between the second set of words as
exists between the first set of words.
1. store: shop ::________: wait
2. hallucination: unreal ::_______: unmoving
3. processor : computer ::______: plant
4. courageous : cowardly ::______: few
5. relax : bath ::_______: blanket
6. monarchy : king ::_______: deity
7. burn : log ::_______: vessel
8. lie : recant ::______: repay
9. infestation : exterminate ::______: cure
10. dissect : animal ::______: sentence

1.

FILL IN THE BLANKS:


In the space provided, write the word from the
list above that best completes each given
sentence.
After awakening from hibernation, a bear is
_________________ for food.

6.
7.
8.
9.
10.

Printing
Heaven
Drift
Health
Expel

___________
___________
___________
___________
___________

7.

Miltons Paradise Lost portrays Heaven as a


true______________, ruled by God.

8.

Car accidents _____________notifying the state


Department of Motor Vehicles.

9.

_____________ talks between the two warring


factions did not result in compromise.

10. Criminals
______________
unsuspecting
consumers by stealing their online passwords.
11. Kittens _______________ against their mother
for comfort and warmth.
12. For some reason, I cannot _________ that
sentence correctly; what are you trying to say?
13. To _______________ the medicine was
unpleasant, but the patient dutifully drank the
mixture.

2.

The patient was relieved to hear that her


_____________ was not serious.

14. All my children ______________ to great things;


one wants to be president, another a lawyer, and
the third a teacher.

3.

The store owner wanted to ________________


the memory of the robbery from her mind.

15. The puppy wreaked ___________ on our living


room, tearing up all the furniture.

4.

The mans date was offended by his


________________ in saying she was not
beautiful.

5.

Because of her shaky hands, the womans


______________ was difficult to read.

6.

The ship was obsolete, so the navy decided to


________________it.

Verbal Ability Class Assignment - 1

Page 50

VERBAL ABILITY CLASS ASSIGNMENT - 2


Parallelism, Tenses, Vocabulary
(c) A and B
(e) None of these

Parallelism
A.

Directions for questions 1 to 5: Identify the


INCORRECT sentence or sentences.

1.
A. I am very fond of my dog, my cat and my canary.
B. I am very fond of my dog, cat and canary.
C. I am only fond of my dog but not of my cat or
my canary.
D. I am only fond of my dog but not of my cat nor
my canary.
E. I am neither fond of my dog, nor of my cat or
canary.
(a) A and C
(b) B and E
(c) C and D
(d) A and B
(e) D and E
2.
A. Not only has the Director impressed the
workers, but has motivated them.
B. The Director has impressed the board with his
inspiring speeches that have improved
employee morale.
C. If the Director has made a good impression on
the workers, then he has succeeded in
motivating them as well.
D. The workers were motivated and the Directors
of the company were impressed with their
hard-work.
E. The board has been impressed by the Director
as he has singlehandedly motivated the
workers.
(a) A only
(b) B and D
(c) C only
(d) D only
(e) None of these.
3.
A. You can play neither indoors nor outdoors.
B. You can play either indoors or outdoors.
C. Being able to play outdoors is more important
than playing indoors.
D. To be able to play indoors is only a substitute for
playing outdoors.
E. Playing indoors is vastly preferred to playing
outdoors.
(a) C only
(b) D and E

(d) E only

4.
A. Caffeine in coffee has been attributed to quicken
the aging process and increasing the risk of cancer.
B. He always is in a hurry and often forgets his keys
and wallet.
C. The power station provides electricity to around
one million people, and is situated on the outskirts
of the township.
D. The boy played with his blocks, his toy cars and
crayons.
E. The creek begins the drop by falling down a 20 feet
cascade, then impacts a large, egg-shaped dome
and spreads into a wide fan, dropping 400 feet.
(a) A, C and D
(b) B, C and D
(c) A, B and D
(d) C only
(e) C, D and E
B.
1.
2.
3.

Identify the error(s) in the given sentences


It was both a long journey and very tiring.
I enjoy swimming, running and to jog.
The revamping of the office is neither simple not it
will be cheap.
4. When I was in London, I learned piano and how to
play the guitar.
5. Father supports my view because, first, its
simplicity and second, it is unique.
6. We were either asked to change our flight or to
take a train.
7. One cannot always be worried or terrified of the
difficulties in life.
8. The manager taught his client how to stand, how
to cry and how talk with fans.
9. I like to officiate basketball than playing basketball.
10. Clara likes singing carols and to play the mandolin.

Tenses
A.
1.
2.
3.

Choose the correct verb form from those given in


brackets.
He (has, is having) a house in Bombay.
I shall meet you when he (comes, will come) back.
It started raining while we (played, were playing)
football.

Verbal Ability Class Assignment - 2

Page 51

4.

Can I have some milk before I (go, am going) to


bed?
5. He (fell, had fallen) asleep while he was driving.
6. I am sure I (met, had met) him at the station
yesterday.
7. He (is living, has been living) in this house (since,
for) ten years.
8. He thanked me for what I (have done, had done)
for him.
9. If you (start, started) at once, you will reach
there by this evening.
10. He (went, has gone) out five minutes ago.
B.

Fill in the blanks with the correct form of the


verb in any aspect of the past tense.
Eric and Ilsa are brother and sister. They (grow)
_______ (1) up together in the city that used to
be known as West Berlin, in the former West
Germany. Eric (move) _______ (2) to the United
States decades ago, before the eastern and
western parts of both Berlin and Germany were
reunited in 1990.Ilsa and her family (visit)
________ (3) Eric and his family last year. llsa's
Family (fly) __________ (4) from Berlin to
Detroit for the visit. Although the children
(never, meet) _______ (5) before, except

through e-mail, the families (have) __________ (6)


a great time together. Every day for a week, the
adults and the children (play) _____ (7), talking,
and eating together. One day, they (cook) _______
(8) some German recipes that (be) _______ (9) in
the family for generations. For years, Ilsa (save)
____ (10) them and treasuring them in a box their
mother (give) _______ (11) her just before she
(die) _______ (12). One of their mother's favorites
(be) ________ (13) a dessert called Apple Kuchen.
One night after everyone else (already, go)
________ (14) to bed, Eric and Ilsa (quietly, sit)
________(15) and talking.
"What have you (miss) ___________ (16) most
about Berlin?" Ilsa wanted to know.
Eric (pause) __________ (17), then answered,
"Mostly, I miss living in a city with such wonderful
landmarks. The kids and I (look) _________ (18) at
some books about Berlin and Germany for a while
before you and your family (arrive) __________
(19). We (discuss) _________ (20) the
Brandenburg Gate when Franz interrupted to ask
why its image (use) _________ (21) as a design on
some of the coins back when Germany had
converted its money to euros."

REVISION OF PRACTICE TEST BOOK -1


Mark the Synonyms of the words in Capital
1. DETRACT
(a) Borrow
(b) Diminish
(c) Dampen
(d) Ruin
2.

3.

4.

5.

PROLOGUE
(a) Event
(c) Preface
TRIMMING
(a) Skimming
(c) Planning
DISPARITY
(a) Disseminate
(c) Discord

6.

He tried to CAJOLE her, but it was in vain.


(a) Rejoice
(b) Inspire
(c) Enjoin
(d) Coax

7.

CONSTERNATION
(a) Group of stars
(c) Large display

(b) Humble service


(d) Great amazement

COVERT
(a) Patent
(c) Ditch

(b) Secret
(d) Greedy

TRIVIAL
(a) Crucial
(c) Vital

(b) Significant
(d) Ordinary

10. POMPOUS
(a) Pretentious
(c) Demanding

(b) Supportive
(d) Flashy

(b) Index
(d) Caption
8.
(b) Arranging
(d) Cutting
9.
(b) Difference
(d) Difficult

On account of the PAUCITY of the funds


plan remained incomplete,
(a) Lack
(b) Scarcity
(c) Abundance
(d) Excess

Verbal Ability Class Assignment - 2

Page 52

VOCABULARY
REVISION OF CLASS ASSIGNMENT-1
Write the Synonyms
1. Aspire
2. Bilateral
3. Candor
4. Defraud
5. Deity
6. Elude
7. Fixture
8. Handwriting
9. Havoc
10. Imbibe
11. Juncture
12. Kernel
13. Lapse
14. Malady
15. Myriad

______________
______________
______________
______________
______________
______________
______________
______________
______________
______________
______________
______________
______________
______________
______________

4.

______________:
to
break
something down into its parts in
order to explain or understand it.
5 ______________: a group of
people waiting in order
6. ______________:
filled
with
hunger that cannot be satisfied
7. ___________: to sink a ship by
creating a hole or holes in the ship
beneath the water line
8. ____________:a government ruled
by the leaders of a religion or by
the power of a god or gods
9. ______________: a world beneath
or below the normal world,
especially a criminal world such as
mafia society or a world of the
dead such as Hades
10. ______________
to
move
uncomfortably; to twist around,
especially to express pain

Fill in the blanks


1.
______________: to make something
necessary.
2.
_____________: to lay or be enclosed
comfortably.
3.
________:to break down into nothing.

1.

Greek and Latin Roots


Exercise: Circle the correct meaning of the
italicized and underlined word(s) in each sentence.
The Latin root am means love. An amiable person
is
(a) Talkative
(b) Truthful
(c) Well educated (d) Friendly, good natured

(c) Illogical.
(d) One that blames others.
4.

The Latin root qui means quiet. A QUIESCENT place


is
(a) Very isolated.
(b) Very chaotic.
(c) Very dangerous
(d) Very still and restful

2.

The Latin root plac means to please. A complacent


person is one who
(a) Makes frequent mistakes
(b) Is argumentative
(c) Is self-satisfied
(d) Is known to tell frequent lies

5.

The Latin root loc/loq/loqu means word, speech.


Something that is ELOQUENT is
(a) Dull and trite
(b) Expressed in an effective way
(c) Very old-fashioned
(d) Equally divided into parts

3.

The Latin root luc/lum/lus means light. A LUCID


argument is
(a) Very clear and understandable.
(b) Loosely held together.

6.

The Greek root auto means self. To have


AUTONOMY means to
(a) Have a lot of money (b) Be independent
(c) Have courage
(d) Have strong opinions

Verbal Ability Class Assignment - 2

Page 53

3.
7.

8.

9.

The Greek root pas/pat/path means feeling,


suffering, disease. To have EMPATHY is to
(a) Give to others.
(b) Have a love for others.
(c) Identify with the feelings of others.
(d) Be similar to others.
The Greek root pseudo means false, fake. The root
nom/nym means name. A PSEUDONYM is
(a) A false name.
(b) An ancient god or deity.
(c) A harsh sound.
(d) A long and boring speech.

One from Many


Read the clues. The LATIN / GREEK root is hidden
within the clues. Look for the word parts that are
the same in the BOLDFACE words.
1.

all the people born around the same time. To


generate means to produce. You are the progeny
of your parents.
Greek root for birth or race: _ _ _
4.

Latin root for mother. Maternal means related to

Greek root for life. A biography is the story of a


persons life. An autobiography is the story of the
writers life. Biology is the science of living things.
Greek root for life: _ _ _

5.

Latin root for see or look. A spectator is a member


of an audience. A spectacle is something to see.

The Greek root dog/dox means opinion. The suffix


-ic means having the quality of. A person who is
DOGMATIC is
(a) Not in touch with reality
(b) Intolerant of other opinions
(c) One who asserts opinions in an arrogant way
(d) Secretive and ungenerous

10. The Greek root phil means love and the root
anthro/andro means human. PHILANTHROPY is
(a) The love of humankind
(b) A preference for something in particular
(c) Using force to control others
(d) Spreading unkind rumors

Greek root meaning birth or race. A generation is

To inspect is to look carefully.


Latin root for see or look: _ _ _ _

For each sentence, below, the Greek/ Latin root


and meaning are provided. The sentences use a
variation of the root word. Fill in the blanks from
the word bank for each sentence.
1.

Verto = turn

a.

Create a(n)_____ so the teacher doesnt see my


surprise!
The old man ____ to his former ways and began
smoking again.
The quiet student is a(n) ____; he never speaks to
anyone.
The Chevy ____ drove in the rain with the top
down.
I have a(n) ___ to insects; I stay away from them.

b.
c.
d.
e.

motherhood, kind, caring, in a protective,


Aversion,
Diversion

motherly way. Maternity means the condition of

Introvert,

Reverted,

Convertible,

being a mother. Your alma mater is a school you


graduated from.
Latin root for mother: _ _ _ _ _
2.

Latin root for water. An aquarium is a water tank


for fish. Aqua and aquamarine are two watery blue
colors. Aquarius is the sign of the Zodiac pictured
as a man pouring water.
Latin root for water: _ _ _ _

2.

Scio = know

a.

His ____ wouldnt let him lie; he confessed about


the crime.
In ____, we are studying biology, electricity, and
cells.
The worker was ____ after his fall from the tower.
People say dreams are thoughts we have in our
____minds.
Theater goers should be ___ and throw away their
trash in the appropriate receptacles.

b.
c.
d.
e.

Verbal Ability Class Assignment - 2

Page 54

Activate,
Unconscious, Conscience, Conscientious, Science,
Subconscious

Fill in the blank with the root word that completes


the sentence.

3.

Phobos = fear

Canto, Culpa, Genus, Jungo, Porto, Scio, Sophos,


Totus, Tractum,

a.
b.

If you are afraid of something, you have a(n) ___.


Locked in a closet as a child, she has fear of close
spaces, ___.
The boy was afraid of anyone from other
countries; hes a(n)___.
A fear of open spaces, such as golf courses, is
called ___.
She has an aversion to water; this is called____.

c.
d.
e.

Agoraphobia, Hydrophobia, Xenophobe, Phobia,


Claustrophobia
Caput = head

a.

He is the ___ of the ship; In charge of everyone


here.
We are not supposed to wear baseball _____in
school.
The head of government for a state is located in
the city that is the_____.
The heading of a chapter, newspaper, or picture is
called the ____.
A tax levied per head, or a fee for each individual,
is called ____.

c.
d.
e.

The root word that means carry is: ______.

b.

Drawn or pulled is the meaning of the root


word: ____.

c.

If someone wants to discuss blame or guilt, they


need the root word: ____.

d.

The root word that means wise is: ___.

e.

This word means know and has many words


referring to the mind: ____.

4.

b.

a.

f.

A choir might perform for an audience using this


root word: ____.

g.

If we talk about the kind or origin of something,


the word we need is: _________.

h.

The word that means join is: ____.

Capitation , Capital, Caps, Captain, Caption

5.

Acta = things done

a.

What was your ____ to the movie we saw in


history?
The Supreme Court took ___in the Brown
Boardcase.
The drivers created a(n)_____of the crash.
The woman deposited her money into vault,
completing the ___.
She could not __ her credit card until she called it
in.

b.
c.
d.
e.

Reaction,

Reenactment,

Transaction,

Action,

Verbal Ability Class Assignment - 2

Page 55

VERBAL ABILITY CLASS ASSIGNMENT - 3


Sentence Correction, Para Jumbles, Vocabulary

Sentence Correction
Direction: Each of the questions consists of a
certain number of sentences. Some sentences
are grammatically incorrect or inappropriate.
Select the option that indicates the
grammatically incorrect or inappropriate
sentence(s).
1
A.
B.
C.
D.

2
A.
B.
C.
D.
E.

It is a perfect plan to execute.


Jane looks good.
A conical old Ming vase was on the table.
It is terribly hot during the day. E. The cat smells
badly.
(a) C and E
(b) B, C and D
(c) B, C and E
(d) A and C
(e) C only
The CEO spoke out surely.
She plays hockey good.
Jack entered the bar deadly drunk.
A snake slithered past our tent.
John always spoke to her kindly.
(a) A and B
(b) B, C and D
(c) B and C
(d) C, D and E

(e) C and D

3.
A. They signed the oath in blood.
B. His writings offer a radical point of view on the
origin of ancient cultural rituals and traditions
play prime roles.
C. The criminal must go on trial for what he has
done.
D. I got the job because I had the necessary
experience.
E. He was educated at a local high school.
(a) E only
(b) C and D
(c) B and C
(d) A only
(e) B only
4.
A. He was a fiction writer who liked to dabble in
mythology.
B. It is often difficult to speak with foreigners in their
mother tongue.
C. It made me angry to have to listen to such lies.
D. The antidote must be administered two times in
a day.
E. I was angered by their constant interruptions.
(a) A and E
(b) C and E
(c) B only
(d) D only
(e) B and D

B. The economy is based on tourism, food industry,


agriculture and on trading and harbour activities.
C. It is the more colourful and wealthy district of the
city threaded by narrow lanes and its economy
centres around the tourist industry.
D. It is the capital of the new democratic country
and historically it was an important city as it was
here that the treaty was signed.
E. Less than two minutes had elapsed since
departing the dock.
(a) A and C
(b) B and E
(c) B and C
(d) B only
(e) A, D and E
6.
A I am probably the worlds worst cook.
B I am fine when it comes to appreciating this
cuisines of great chefs but, left to myself, I
couldnt even fry an egg.
C It is not that I dont know how to do all this.
D I can tell you how a kakori kabab is made, how the
flavor of a coq au vin is directly related to the
wine you should use.
(a) A only
(b) B only
(c) B and D
(d) C only
(e) No Error
7.
A I just read in a news magazine that TV comprise
67 percent of the Indian entertainment industry.
B Films and other things comprise the other 23
percent.
C Finally, television is at par with film industry.
D A TV actor is a bonafide star now and TV is not a
poor cousin of Bollywood anymore.
(a) A and C
(b) B only
(c) A, B and C
(d) A and D
(e) C only
8.
A A print design project communicates a message.
B Contrast is a good way to create a noticeable
design and layout.
C Contrast can be size, color, direction, format or
the expected versus the unexpected.
D Size contrast can involve graphics or type.
(a) B
(b) B and C
(c) C
(d) A, B and C
(e) A and C

5.
A. The busiest air route from the airport in terms of
passengers is Mumbai airport.
Verbal Ability Class Assignment - 3

Page 56

Para jumbles
Direction: Each question consists of a number of
sentences that need to be properly sequenced,
to form a meaningful and coherent
paragraph/sentences. Choose the most logical
order of sentences from the choices given
below.
1
I.
II.
III.
IV.
V.
VI.

2
I.
II.
III.
IV.
V.

knowledge, even when it is exact


because we tend to forget what we know
if we do not pay attention
forget how to process it properly
does not often lead to appropriate actions
even when we are experts
(a) I, V, II, IV, III, VI
(b) I, V, III, IV, II, VI
(c) VI, V, II, I, III, IV
(d) III, VI, I, II, V, IV
in the classroom
once they are let out on streets
statisticians, it has been shown
and engage in most trivial inferential errors
tend to leave their brains
(a) III, V, I, IV, II
(b) II, III, V, I, IV
(c) I, V, II, III, IV
(d) III, I, V, IV, II

3.
I.
II.
III.
IV.
V.

sure enough failed


I have tested myself and
by carefully setting a wide range
even while consciously trying to be humble
as we will see the core of my professional
activities
VI. and yet such underestimation happens to be
(a) I, II, V, III, IV, VI
(b) IV, II, I, III, V, VI
(c) V, II, I, IV, III, VI
(d) II, I, IV, III, VI, V
4.
I. except that people got all excited
II. so I would not have cared
III. and talked quite a bit about
IV. the least about them
V. pouring verbal sauce around the forecasts
VI. at these figures were going to mean
(a) I, II, IV, V, III, VI
(b) I, III, II, IV, V, VI
(c) II, IV, I, III, VI, V
(d) V, II, IV, VI, I, III
Direction: A number of sentences are given
below, which when properly sequenced, form a
coherent paragraph. Choose the most logical
order of sentences from the choice given to
construct a coherent paragraph.

C. Have you ever seen a play or motion picture that


was so charming that you felt sheer delight as you
watched?
D. I have not spoken of books that grip you
emotionally, of plays and movies that keep you
on the edge of your seat in surprise, or of food
that satisfies a ravenous hunger.
E. Or perhaps you have had a portion of pumpkin
pie, light and airy and mildly flavoured, and with
a flaky, delicious crust, that was the last word in
gustatory enjoyment?
(a) AECDB
(b) CEBDA
(c) DBACE
(d) ACEBD
2.
A. The rapid pace of developments is probably a
strong answer to the rival National Cricket league
challenging its supremacy over cricket in the
country.
B. And that seems to be the strategy for success for
the BCI, especially with cricket lovers moving
towards the faster version of the game.
C. Already the board has signed up thirty
international cricketers and has already invited
bids for the teams.
D. The board is busy signing up players for its new
cricket league as if there is no tomorrow.
(a) DABC
(b) BACD
(c) DBCA
(d) ADBC
(e) DBAC
3.
A. He believed that honesty is the best policy.
B. He would otherwise be forced to resign from his
position, which he could not afford.
C. However, the situation demanded that he bend
the rules according to the wish of his boss.
D. He was facing a great dilemma and could not
decide what should be done.
(a) ADCB
(b) DBAC
(c) ACBD
(d) DACB
(e) BCAD
4.
A. You cant miss a huge blue sign displayed near the
exit.
B. Beware! it announces in bold red lettering as
you start thinking where you have reached.
C. As soon as you land at the new Mumbai airport
and enter the terminal, you feel like you can get
lost in the crowd.
D. Do not leave your baggage unattended. Watch
out for pickpockets says the fine print.
E. You stop and read to find out more.
(a) CABDE
(b) BCADE
(c) ABCED
(d) CBAED
(e) CABED

1.
A. Have you ever gone through a book that was so
good you kept hugging yourself mentally as you
read?
B. Now, notice the examples I have used
Verbal Ability Class Assignment - 3

Page 57

REVISION TEST OF PRACTICE TEST BOOK - 2


Mark the synonyms of the following
words
(b) Sorrowful
(d) Laborious

6. PERDITION
(a) Firmness
(c) Damnation

(b) Bounty
(d) Objection

2. DOLOROUS
(a) Sorrowful
(c) Sacred

(b) Financial
(d) Parsimonious

7. TARDY
(a) Casual
(c) Late

(b) Pliable
(d) Favourable

3. DOWDY
(a) Corpulent
(c) Elegant

(b) Rakish
(d) Unstylish

8. MAMMOTH
(a) Straight
(c) Wild

(b) Huge
(d) Greedy

(b) Prohibition
(d) Surety

9. EULOGY
(a) Apology
(c) Speech

(b) Address
(d) Praise

1. CUMBERSOME
(a) Heavy
(c) Unwieldy

4. INSURRECTION
(a) Blessing
(c) Uprising
5. ERR
(a) Hasten
(c) Despise

(b) Scrawl
(d) Be wrong

10. ORDERLY
(a) Semitic
(c) Democratic

(b) Colic
(d) Chaotic

VOCABULARY
REVISION OF CLASS ASSIGNMENT - 2
Fill in the blanks with correct words:
1. Lucid
______________
2. Eloquent
______________
3. Progeny
______________
4. Reverted
______________
5. Agoraphobia
______________
6. Capitation
______________

7.
8.
9.
10.
11.
12.

Scio
Caput
Canto
Genus
Ravenous
Nestle

______________
______________
______________
______________
______________
______________

Abdicate (v)
Affectation (n)

To decline or reject a powerful position, such as a kingship


An exaggerated or deliberate mannerism

Arbitrary (adj)
Beseech (v)

Done or chosen without a reason; random


To beg or plead

Censorious (adj)
Consensus (n)

Finding fault; judging harshly


An opinion that is agreed upon by all members of a group of people
Verbal Ability Class Assignment - 3

Page 58

Cudgel (n)
Despite (prep)

A short, thick blunt weapon; a club


In spite of

Effrontery (n)
Fealty (n)
Hesitant (adj)

oldness; audacity
Loyalty; fidelity
Lacking decisiveness; uncertain or unwilling to make a decision

Inaudible (adj)
Lithe (adj)

Unable to be heard
Moving or bending easily; graceful and fluid in motion, supple

Metamorphosis (n)

A change in form or shape, often from one stage of development to another, as a


caterpillar turning into a butterfly

Nocturnal (adj)

Belonging to, occurring in, or related to the night time

Opulence (n)

Riches or affluence, especially as shown by expensive or ostentatious possessions or


lifestyle

Partition (n)
Phonetic (adj)

Something which separates two parts or areas


Of, relating to, or representing the sounds of speech

Prolong (v)
Reckless (adj)

To make something last longer; to extend a period of time


Careless or unwise; unheeding of danger

Rigor (n)
Somnolent (adj)

A harsh or stern rigidity; unyielding


Drowsy or inclined to sleep

Tenacious (adj)
Umbrage (n)
Veto (n)

Stubborn; holding fast to a belief or goal


Resentment based on a feeling of having been injured
A vote that blocks a decision, the order or right to prohibit

SYNONYMS:
In the blank space, write the word from the list
above that means the same (about the same) as
the word given.
1. Offense
____________
2. Allegiance
____________
3. Pretension
____________
4. Divider
____________
5. Persistent
____________
6. Judgmental
____________
7. Sleepy
____________
8. Accord
____________
9. Transformation
____________
10. Implore
____________

ANTONYMS:
In the space provided, write the word from the list
above that means opposite of the word given.
1. Cautious
____________
2. Wakeful
____________
3. Timidity
____________
4. Purposeful
____________
5. Flexibility
____________
6. Awkward
____________
7. Disagreement
____________
8. Poverty
____________
9. Shorten
____________
10. Certain
____________

ANALOGIES:
Choose the word from the list that best
completes the analogy, creating the same
relationship between the second set of words as
exists between the first set of words.

1.
2.
3.
4.
5.

invisible : see ::
: hear
resign : job ::
: throne
molecular : matter ::
: speech
exclaim : tell :: : ask
pistol : shot ::
: blow

Verbal Ability Class Assignment - 3

Page 59

6.
7.
8.
9.
10.

diurnal : sun :: : moon


stretch : space ::
: time
into : out of ::
: because of
rejection : application ::
: law
spendthrift : poverty ::
: danger

In the space provided, write the word from the list


above that best completes each given sentence.
1. His ________________ of raising one
eyebrow to express disdain was pretentious.
2.

The committee could not come to a


________________ about which offer to
choose; there were three members who
disagreed with the majority.

3.

Because of her __________ in upholding the


rules, the students thought she was cruel.

4.

The victim was hit over the head with a blunt


object, like a _______________.

5.

Many schools use a ___________ method to


teach young children to read, by teaching
what sounds letters or combinations of
letters represent.

6.

The president may ___________ a bill, but


congress can still pass it if there is enough
support.

7.

In order to ______________ their interview,


he thought of as many things to talk about as
he could.

8.

She considered it _______________ for the


young man to ask her age.

9.

The _______________of a caterpillar into a


butterfly takes place inside a cocoon.

12. The little girl continued to ________ her


mother to buy her a pony.
13. The ________________ dancer
gracefully complete even the
complicated movements.

could
most

14. Josh's parents' ______________ attitude


made him miserable; they were always
criticizing him!
15. The
music
was
practically
_____________because he had the volume
on its lowest setting.

10. He was _________ in trying to get the street


renamed; he just wouldn't give up.
11. The _______________ of the palace was
amazing to her; she had never seen such
wealth before.

Verbal Ability Class Assignment - 3

Page 60

VERBAL ABILITY CLASS ASSIGNMENT - 4


Critical Reasoning, Implicit Statement, Statement Conclusion, Strong & Weak Argument

Critical Reasoning
Directions: Read the following passage and
answer the questions that follow.
Virtually all health experts agree that secondhand smoke poses a serious health risk. After the
publication of yet another research paper
explicating the link between exposure to
secondhand smoke and a shorter life span, some
members of the State House of Representatives
proposed a ban on smoking in most public places
in an attempt to promote quality of life and
length of lifespan.
1. Which of the following, if true, provides the
most support for the actions of the State
Representatives?
(A) The amount of damaging chemicals and fumes
released into the air by cigarette smoke is far less
than the amount released from automobiles,
especially from older models.
(B) Banning smoking in most public places will not
considerably reduce the percent of the
population in the state in question that smokes.
(C) The state whose legislators are proposing the
tough smoking legislation has a relatively high
percent of its population that smoke.
(D) Another state that enacted a similar law a decade
ago saw a statistically significant drop in lung
cancer rates among non-smokers.

(C) The study was only conducted in one city.


(D) The participants in the study who ate beets were
more likely to exercise regularly than those who
did not eat beets.
Directions: Read the following passage and
answer the questions that follow.
Automation, the trademark of a modern
economy, is essential to maximizing a country's
economic production while minimizing its costs.
Health care executive want to increase revenues
while reducing costs. Consequently, they propose
significantly greater automation of health care.
Yet, this should be rejected. Radical automation
of health care would cause patients to lose trust
in the system as the health care they would
receive would lack the in-person care that studies
show patients desire.
3. Which of the following expresses the main point
of the argument?
(A) Health care should not be heavily automated
(B) Patients desire customized in-person care
(C) Trends in the general economy do not apply to
the health care industry
(D) Health care executives are becoming too greedy

Eating beets significantly lowers the risk of


cancer, according to an article in a nutritional
magazine. The article refers to a study that found
that people who consumed one or more beets per
day were half as likely to be diagnosed with the
disease as people who did not.

Directions: Read the following passage and answer


the questions that follow.
E-businesses are starting to gradually take over
traditional "bricks-and-mortar" businesses. Last
year, companies listed on the NASDAQ, a stock exchange index for new technological companies,
rose on average twice as fast as the fastest-rising
stocks listed on the Dow Jones index.

2. Which of the following, if true, most weakens


the argument in the magazine article?
(A) Another study found that people who consumed
one tablespoon of flax seed oil per day were more
than four times less likely to be diagnosed with
cancer as those who did not.
(B) Participants in the study reported consuming no
vegetables other than beets.

4. The argument about e-businesses would be


strengthened the most by which of the
following?
(A) A report by a financial expert stating that share
prices are set to rise in the future.
(B) An end-of-year statement by a company listed on
the NASDAQ announcing record profits for the
past year.

Verbal Ability Class Assignment - 4

Page 61

(C) An article in the Financial Times indicating that a


larger proportion of traditional firms (that is, non
e-business firms) have gone bankrupt than new
e-businesses.
(D) A novel by a company director about an ebusiness that ends up on the bestsellers list.
Directions: In each of the following questions,
two statements numbered I and II are given.
There may be cause and effect relationship
between the two statements. These two
statements may be the effect of same cause or
independent causes. These statements may be
independent causes without any relationship.
Read the statements and answer using the
options that follow.
5. Statements
I. The prices of petrol and diesel in the domestic
market have remain unchanged for the past few
months.
II The crude oil prices in the international market
have gone up substantially in the last few months.
(a) Statement I is the cause and statement II is the
effect
(b) Statement II is the cause and statement I is the
effect
(c) Both the statements are independent causes
(d) Both the statements are independent effects
of the independent causes
6. Statements
I. The private medical colleges have increased the
tuition fees in the current year by 200% over the
last years fee to meet the expenses.
II.The Government medical colleges have not
increased their fees in spite of the price escalation
(a) Statement I is the cause and statement II is the
effect
(b) Statement II is the cause and statement I is the
effect
(c) Both the statements are independent causes
(d) Both the statements are independent effects
of the independent causes

IMPLICIT STATEMENT
In each question below is given a statement
followed by two assumptions numbered I and II.
Consider the statement and assumptions and
decide which of the assumptions is implicit in
the statement
BMS 2013
1. Statement:
A notice in the train compartment reads-To
stop the train, pull this chain. Fine for improper
use is RS 500.
Assumptions
I) Some people misuse chain pulling
II) Sometimes people want to stop a running
train
a) Only I is implicit
b) Only II is implicit
c) Either I or II is implicit
d) Both I and II are implicit
2. Statement:
The patients condition would not improve even
after operation.
Assumptions
I) The patient cannot be operated upon in this
condition
II) The patient can be operated upon in this
condition
a) Only I is implicit
b) Only II is implicit
c) Either I or II is implicit
d)Both I and II are implicit
3. Statement:
In spite of poor services, the commuters have not
complained against it.
Assumptions
I) Generally people do not tolerate poor
services.
II) Complaints help in improving services.
a) Only I is implicit
b) Only II is implicit
c) Either I or II is implicit
d) Both I and II are implicit
4. Statement:
This book is so prepared that even a layman can
study Geology in the absence of a teacher.

Verbal Ability Class Assignment - 4

Page 62

Assumptions
I) A teacher may not always be available to teach
Geology
II) A layman generally finds it difficult to learn
Geology on its own.
a) Only I is implicit
b) Only II is implicit
c) Either I or II is implicit
d) Both I and II are implicit

STATEMENT AND CONCLUSION


Direction: Study the statements and the two
conclusions and give answer
(a) If only conclusion I follows
(b) If only conclusion II follows
(c) If both conclusions I and II follow
(d) If neither I nor II follow
1. Statement:
The most polluting units are those engaged in
electroplating metals. These units generate
highly toxic substances. Such industries are
concentrated in the walled city.
Conclusions:
(I) Electroplating industries must be shut down.
(II) In the walled city there is greater pollution.
2. Statement:
Imports have led to a difficult foreign exchange
position. Exports have not been stepped up. So
industry has suffered.
Conclusions:
(I) Industries need foreign exchange.
(II) We spend foreign exchange for importing goods.
3. Statement:
Maintenance of peace is essential for progress.
As most of the Asian countries are passing
through a developing stage, they should honestly
work for peace.
Conclusion:
(I) Asian countries must make progress.
(II) Asian countries are not working for progress.

STRONG AND WEAK


ARGUMENTS
Direction: An interrogative sentence is followed
by two arguments-one beginning with Yes and
the other with No. You are to pick one of the
following answer choices which is applicable to
each of the following questions:
(a) If only argument I is strong
(b) If only argument II is strong
(c) If both argument I and II are strong
(d) If neither I nor Ii is strong
1. Has India been consistently non-aligned?
(I) Yes, because India has not sided with any big
power.
(II) No, because India did not condemn Russian
occupation of Afghanistan.
2. Should education be job oriented?
(I) Yes, because the purpose of education is to
prepare us for earning.
(II) No, because education should be for the sake of
education.
3. Should sports be made compulsory for all able
bodied students?
(I) Yes, because it will help them to keep healthy.
(II) No, because sports will divert the attention of the
students from studies.
4. Should India remain secular?
(I) Yes, because that is the best way of harmony in a
diverse culture like India.
(II) No, because the different religious groups do not
see eye to eye with one another.
5. Should the press in India be given full freedom?
(I) Yes, because only, then people will become
politically enlightened.
(II) No, because such a press will create problems.
6. Should the public sector in India be abolished?
(I) Yes, because it is entirely inefficient
(II) No, because it has always given us high quality
goods.

Verbal Ability Class Assignment - 4

Page 63

REVISION TEST OF PRACTICE TEST BOOK - 3


1.

2.

3.

4.

5.

ERRAND
(a) Energy
(c) Mistake

(b) Task
(d) Blunder

NONCHALANT
(a) Imaginary
(c) Neutral

(b) Casual
(d) Formal

DULCET
(a) Melodious
(c) Pastry

(b) Squadron leader


(d) Battleground

EDIFY
(a) Amuse
(c) Satisfy

(b) Consume
(d) Instruct

YEARN
(a) Confront
(c) Deny

(b) Crave
(d) Accept

6.

7.

8.

9.

IMPECCABLE
(a) Upright
(c) Inoffensive

(b) Harmless
(d) Flawless

CAMOUFLAGE
(a) Demonstrate
(c) Disguise

(b) Fabric
(d) Cover

PENSIVE
(a) Contemplative
(c) Reluctant

(b) Precise
(d) Unhappy

FILIAL
(a) Parent
(c) Son

(b) Duty
(d) Wise man

10. AFFLUENT
(a) Volatile
(c) Vanishing

(b) Wealthy
(d) Ill

VOCABULARY
REVISION OF CLASS ASSIGNMENT
Fill in the blanks with correct words
1.
2.
3.
4.
5.

Lucid
_________________
Eloquent _________________
Offense
_________________
Allegiance _________________
Agoraphobia________________

6. Capitation _________________
7. Scio
_________________
COMPLETE THE GIVEN ANALOGIES
8. diurnal : sun ::
: moon
9. stretch : space ::
: time
10. rejection: application :: : law

Word List with Definitions


Aberration (n)
A variation or change from what is normal or right; something unnatural
Aghast (adj)
Shocked; terrified or amazed
Appellation (n)
A name or title; a word used to identify a person or thing
Bewilder (v)
To perplex or confuse; to disorder a person's thinking
Chattel (n)
A thing that is owned; a possession
Contemptuous (adj) Thinking poorly of; scornful or disdainful
Debonair (adj)
Charming; having a sophisticated and attractive manner
Detest (v)
To have a profound dislike or abhorrence for; to hate

Verbal Ability Class Assignment - 4

Page 64

Elasticity (n)

The physical property of matter wherein matter returns to its original form after
being reshaped, as an elastic band returns to its original shape after Being stretched

Fickle (adj)

Capriciously changeable in affection or belief; not remaining faithful to one idea or


feeling

Hindrance (n)
Inconceivable (adj)
Litigious (adj)
Minimize (v)
Notorious (adj)
Ordeal (n)
Ostracize (v)

An obstruction or impediment; something that blocks progress


Unimaginable or unthinkable
Willing and likely to pursue a lawsuit
To make as small as possible
Widely known but viewed negatively or critically
An experience that is extremely difficult or trying
To banish from or force out of a group or situation

Peccadillo (n)
Placid (adj)
Prosaic (adj)
Rectitude (n)
Salient (adj)
Spurious (adj)
Tincture (n)
Vigilance (n)

A minor sin or fault; a trifling wrongdoing


Tranquil and calm; at rest
Matter-of-fact; without spirit, style, or imagination
Correctness or uprightness; unerring in following principles
Noticeable or important; standing out
Inauthentic; not bona fide
A typically alcoholic mixture containing a medicine or remedy
Alertness; unbreaking attention to a duty

SYNONYMS:

ANTONYMS:

In the blank space, write the word from the


list above that means the same (about the
same) as the word given.
1. Unbelievable
____________
2. Peaceful
____________
3. Obstacle
____________
4. False
____________
5. Horrified
____________
6. Abnormality
____________
7. Righteousness
____________
8. Infamous
____________
9. Loathe
____________
10. Exclude
____________

In the space provided, write the word from


the list above that means opposite of the
word given.
1. Rude
____________
2. Inconspicuous
____________
3. Inattentiveness
____________
4. Plasticity
____________
5. Help
____________
6. Steadfast
____________
7. Lauded
____________
8. Enlighten
____________
9. Creative
____________
10. Maximize
____________

ANALOGIES:
Choose the word from the list that best completes the analogy, creating the same relationship between
the second set of words as exists between the first set of words.
1. degree : B.A. :: _______ : Mr.
6. conundrum : problem:: _______: situation
2. hunted : sought :: __________ : famous
7. explain : confusion : :___________: clarity
3. awestruck : admire :: __________: scorn
8. flirtation : love :: ________: cardinal sin
4. product : buy : :____________ : own
9. gorgeous : looks :: _________ : manners
5. saline : salt :: _____________ : medicine
10. ambitious : power ::__________: lawsuits

Verbal Ability Class Assignment - 4

Page 65

1.
2.

3.
4.
5.
6.
7.
8.
9.
10.
11.
12.
13.
14.
15.
16.
17.

18.
19.
20.

In the space provided, write the word from the word list that best completes each given sentence.
The nun lived her life with _______and propriety.
References to slaves as "________" from pre-Civil
War times make it clear that slaves were viewed
more as property than as people.
Jenny felt that her little sister was more of a
__________ than a help in baking cookies.
In Oedipus Rex, the gods _______ Oedipus from
Thebes, and he becomes a wanderer.
The artist's work is _________,lacking originality.
The conclusion that is most _______ to me in
this report is that we must raise our prices.
The monster created by Dr. Frankenstein was an
________, an unnatural beast.
Mr. Siddle was _____________ at the horrifying
sight that met his eyes.
Josie learned a recipe to make a homemade
_____________ of medicinal herbs and alcohol.
The waters were so ____________ that it seemed
impossible to believe a storm was brewing.
Like many teenagers, Omar was _______, with a
new crush each week.
Antonia accused her mother of being _________
when she filed her third lawsuit.
Cary Grant was very _______, and his suave,
refined persona made him popular.
She tried to ________ the number of trips she
had to make each week to save gas.
Joshua was ______________ of his elder brother,
who seemed ignorant and crude.
Arnold Schwarzenegger became known by the
informal ______ "The Governator."
Margaret felt that being late for curfew was just
a _________, but her parents thought it a serious
transgression.
The little girl began to __________ the taste of
spinach, and she refused to eat it.
Rubber bands have high _________, which
constricts them after they've been stretched.
It is ____________ to me that aliens have visited
our planet.

Verbal Ability Class Assignment - 4

Page 66

VERBAL ABILITY CLASS ASSIGNMENT - 5


Tone of Passage/Summary, Reading Comprehension

Tone of Passage/Summary
Directions for question: From among the
options, choose the summary of the passage
that is written in the same style as that of the
passage.

(d) TILA is definitely working for the


betterment of the society as its goals are
aligned with those of human progress.
(e) The rights of customers have to be
protected. TILA seeks to standardize
disclosure of costs to aid the consumer to
shop but it does offer protection from
certain high- cost mortgage loans.

1) The purpose of TILA is to promote the


informed use of consumer credit by requiring
disclosures about its terms, cost to
standardize the manner in which costs
associated with borrowing are calculated and
disclosed. TILA also gives consumers the right
to cancel certain credit transactions that
involve a lien on a consumer's principal
dwelling, regulates certain credit card
practices, and provides a means for fair and
timely resolution of credit billing disputes.
With the exception of certain high- cost
mortgage loans, TILA does not regulate the
charges that may be imposed for consumer
credit. Rather, it requires uniform or
standardized disclosure of costs and charges
so that consumers can shop.
(a) I have realized that TILA is a law which
protects the rights of consumer. As a
consumer I have noticed that TILA seeks to
standardize disclosure of costs to aid the
consumer to shop.
(b) TILA protects the rights of consumers by
regulating the calculation and disclosure of
the costs associated with borrowing. TILA
seeks a standardized disclosure of costs to
aid the consumer to shop.
(c) Whether TILA will prove to be useful
remains to be seen. It aims at protecting
the rights of consumers. At present, TILA
seeks to standardize disclosure of costs to
aid the consumer to shop.

2) Democracy doesn't mean much if people


have to confront concentrated systems of
economic power as isolated individuals.
Democracy means something if people can
organize to gain information, to have
thoughts for that matter, to make plans, to
enter into the political system in some active
way, to put forth programs and so on. If
organizations of that kind exist, then
democracy can exist too. Otherwise it's a
matter of pushing a lever every couple of
years; it's like having the choice between
Coca-Cola and Pepsi- Cola.
(a) Democracy means something only if there
exist organizations that can help people to
mobilize themselves into performing
some concrete action.
(b) Democracy is a form of government that
symbolizes concentrated systems of
economic power and the inability of
individuals to try and bring about any
meaningful change.
(c) Democracy means something only if
people can organize to gain information,
to have thoughts for that matter, to make
plans, to enter into the political system
and to put forth programs and so on.
(d) While democracy may be the best and
most civilized form of governance, the fact
remains that unless organizations exist
that can enable people to perform some
concrete actions, like entering into the

Verbal Ability Class Assignment - 5

Page 67

political system, putting forth programs


and so on; it means very little in real terms.
(e) I feel that democracy only means
something if organizations exist that can
enable people to gain information, to put
forth programs and so on. Otherwise it
means very little in real terms.
Directions for questions 3 and 4: Answer the
questions based on the passage given below.
In the colonies the most striking feature of
Spanish economic policy was its wastefulness.
After the conquest of the New World, it was to
the interest of the Spaniards to gradually wean
the native Indians from barbarism by teaching
them the arts and sciences of Europe, to
encourage such industries as were favored by
the soil, and to furnish the growing colonies
with those articles which they could not
produce themselves, and of which they stood in
need. Only thus could they justify their
monopoly of the markets of Spanish America.
The same test, indeed, may be applied to every
other nation which adopted the exclusivist
system. Queen Isabella wished to carry out this
policy, introduced into the newly-discovered
islands wheat, the olive and the vine, and
acclimatized many of the European domestic
animals. Her efforts, unfortunately, were not
seconded by her successors, nor by the
Spaniards who went to the Indies. In time the
government itself, as well as the colonist, came
to be concerned, not so much with the
agricultural products of the Indies, but with the
return of the precious metals. Natives were
made to work the mines, while many regions
adapted to agriculture, Guiana, Caracas and
Buenos Ayres, were neglected, and the
peopling of the colonies by Europeans was
slow. The emperor, Charles V, did little to stem
this tendency, but drifted along with the tide.
Immigration was restricted to keep the
colonies free from the contamination of heresy
and of foreigners. The Spanish population was
concentrated in cities, and the country divided
into great estates granted by the crown to the
families of the conquistadores or to favourites

at court. The immense areas of Peru, Buenos


Ayres and Mexico were submitted to the most
unjust and arbitrary regulations, with no object
but to stifle growing industry and put them in
absolute dependence upon the metropolis.
3). Identify the tone of the following
sentence.
After the conquest of the New World, it was to
the interest of the Spaniards to gradually wean
the native Indians from barbarism by teaching
them the arts and sciences of Europe, to
encourage such industries as were favored by
the soil, and to furnish the growing colonies
with those articles which they could not
produce themselves, and of which they stood
in need.
(a) Pessimistic
(b) Didactic
(c) Cynical
(d) Concern
(e) Sympathetic
4). Identify the tone of the following
sentence.
The immense areas of Peru, Buenos Aires and
Mexico were submitted to the most unjust and
arbitrary regulations, with no object but to
stifle growing industry and put them in
absolute dependence upon the metropolis.
(a) Cynical
(b) Sympathetic
(c) Concern
(d) Critical
(e) Didactic
Directions for questions Q5 to Q7: Answer the
questions based on the passage given below.
In the United Kingdom, funding the war had a
severe economic cost. From being the world's
largest overseas investor, it became one of its
biggest debtors with interest payments
forming around 40% of all government
spending. Inflation more than doubled
between 1914 and its peak in 1920, while the
value of the Pound Sterling (consumer
expenditure) fell by 61.2%. Reparations in the
form of free German coal depressed the local
industry, precipitating the 1926 General Strike.
British private investments abroad were sold,

Verbal Ability Class Assignment - 5

Page 68

raising 550 million. However, 250 million


new investment also took place during the war.
The net financial loss was therefore
approximately 300 million; less than two
years investment compared to the pre-war
average rate and more than replaced by 1928.
Material loss was "slight": the most significant
being 40% of the British merchant fleet sunk by
German U-boats. Most of this was replaced in
1918 and all immediately after the war. The
military historian Correlli Barnett has argued
that "in objective truth the Great War in no way
inflicted crippling economic damage on
Britain" but that the war "crippled the British
psychologically but in no other way". Less
concrete changes include the growing
assertiveness of Commonwealth nations.
Battles such as Gallipoli for Australia and New
Zealand, and Vimy Ridge for Canada led to
increased national pride and a greater
reluctance to remain subordinate to Britain,
leading to the growth of diplomatic autonomy
in the 1920s. These battles were often
decorated in propaganda in these nations as
symbolic of their power during the war.
Traditionally loyal dominions such as
Newfoundland were deeply disillusioned by
Britain's apparent disregard for their soldiers,
eventually leading to the unification of
Newfoundland into the Confederation of
Canada. Colonies such as India and Nigeria also
became increasingly assertive because of their
participation in the war. The populations in
these countries became increasingly aware of
their own power and Britain's fragility.
5). A suitable title for this passage can be:
(a) The aftermath of World War I on
Britain.
(b) Post war Great Britain.
(c) The futility of war.
(d) The economic depression in Great
Britain.
(e) The Great War A wasted effort.

6). What is the Central Idea of this passage?


(a) The involvement of the British Empire
in the First World War and its after
effects.
(b) The economic losses and the resulting
changes faced by the British after the
war.
(c) The rising reluctance in British colonies
over being ruled as a result of their
active participation in war activities.
(d) The economic and psychological
damages suffered by Britain in the post
war era.
(e) The post war battles and uprisings in
the commonwealth nations after the
First World War.
7) Which of the following options best
summarizes the passage given above?
(a) The United Kingdom faced an economic
crisis after the war in the form of
investment losses, rising inflation and
labour strikes. Apart from this, 40% of
the British fleet was wiped out by the
Germans.
(b) Britain not only suffered huge economic
losses and national crises but also had to
deal with growing assertiveness of their
colonies which happened as a result of
their participation in the First World
War.
(c) The United Kingdom, even though
victorious, suffered greatly due to the
war and faced problems of both
economic as well as of a psychological
nature.
(d) Investment losses worth 300 million
pounds, the consumer expenditure
reducing by 61.4%, the General Strike of
1926, depletion of 40% of their fleet and
tensions rising in their colonies were
some of the problems faced by the
British after the war.
(e) British colonies like Newfoundland,
Australia, New Zealand, India and
Nigeria gained voice because of their
participation in the war and made life

Verbal Ability Class Assignment - 5

Page 69

difficult for the Empire in the post war


era.

Reading Comprehension
Directions: Read the following passage and
answer the questions, choosing the correct
answer from among the choices given.
Passage-1
The Great Debates
The first of the Great Debates, between
Senator John F. Kennedy of Massachusetts and
the incumbent Vice President Richard Nixon on
September 26, 1960, centered on domestic
issues. The topic of the next debate, on October
7, was a clash over U.S. policy regarding two
small islands off the Chinese coast, and on
October 13, this controversy continued. On
October 21, the final debate, the candidates
focused on American/Cuban relations.
Few of the 70 million viewers could have
fathomed what this firstever televised
presidential debate augured, not only for this
specific series of debates, but more importantly
for the preeminent role the fledgling medium
would play in the future of the political arena.
A pallid Nixon arrived at the Chicago CBS
studios after a grueling day of campaigning.
The previous August a knee infection had
sidelined him. He was still twenty pounds
underweight, and he perspired profusely in an
illfitting shirt. Moreover, he declined makeup
to burnish his hospital pallor. The freshly
painted studio backdrop had dried to an ashen
hue that obscured his matching suit.
The Democratic contender by contrast exuded
a robust glow after a month of 15 campaigning
in California. He had spent his day rehearsing
potential questions and relaxing. An aide later
admitted that he supplemented his natural
glow with a smidge of makeup. He was fit, trim,
and confident.
Despite the remarkably similar agendas and
arguments of the Republican and the
Democrat, TV viewers unequivocally believed

Kennedy to be the victor whereas people who


20 had followed the debates on the radio held
the opposite opinion. The age of TV had
arrived, and the subsequent party shuffle
proved the undeniable potency of television.
1) The author is mainly concerned about
(a) The debating styles of John Kennedy and
Richard Nixon during the 1960 Great
Debates
(b) The domestic issues which affected the
result of 1960 Great Debates
(c) The health of Richard Nixon at the time
of the 1960 Great Debates
(d) The number of television viewers who
tuned in to the 1960 Great Debates
(e) The effect of television on the results of
the 1960 Great Debates
2) It can be inferred from the passage that
(a) Kennedy was a better debater than
Nixon
(b) Nixon was the unequivocal winner of the
1960 debates
(c) The Democrat beat the Republican in the
1960 election
(d) Nixon was more prepared for the first
debate than Kennedy
(e) Kennedy and Nixon disagreed strongly
on issues on the home front.
3) According to the passage, which of the
following was true of Richard Nixon?
(a) He had a five oclock shadow during the
first debate.
(b) He wore a brown suit during the first
debate.
(c) He warned of the impending Cuban
crisis.
(d) He limped onstage for the first debate.
(e) He lost his job after the election.
Passage-2
A sanctuary may be defined as a place where
man is passive and the rest of nature active. Till
quite recently nature had her own sanctuaries,
where man either did not go at all or went only
as a tool using animal in comparatively small

Verbal Ability Class Assignment - 5

Page 70

numbers. But now, in this machinery age, there


is no place left where man cannot go with
overwhelming forces at his command. He can
strangle to death all the nobler wild life in the
world today. Tomorrow he certainly will have
done so, unless he exercises due foresight and
self-control in the meantime. There is not the
slightest doubt that birds and mammals are
now being killed off much faster than they can
breed. And it is always the largest and noblest
forms of life that suffer most. The whales and
elephants, lions and eagles, go. The rats and
flies, and all mean parasites, remain. This is
inevitable in certain cases. But it is wanton
killing off that I am speaking of tonight.
Civilized man begins by destroying the very
forms of wild life he learns to appreciate most
when he becomes still more civilized. The
obvious remedy is to begin conservation at an
earlier stage, when it is easier and better in
every way, by enforcing laws for close seasons,
game preserves, the selective protection of
certain species, and sanctuaries. I have just
defined a sanctuary as a place where man is
passive and the rest of nature active. But this
general definition is too absolute for any
special case. The mere fact that man has to
protect a sanctuary does away with his purely
passive attitude. Then, he can be beneficially
active by destroying pests and parasites, like
bot-flies or mosquitoes, and by finding
antidotes for diseases like the epidemic which
periodically kills off the rabbits and thus starves
many of the carnivore to death. But, except in
cases where experiment has proved his
intervention to be beneficial, the less he upsets
the balance of nature the better, even when he
tries to be an earthly providence.
4) The author implies that his first
definition of a sanctuary is
(a) Totally wrong
(b) Somewhat idealistic
(c) Unhelpful
(d) Indefensible
(e) Immutable

5) The authors argument that destroying


bot-flies and mosquitoes would be a
beneficial action is most weakened by all
of the following except
(a) Parasites have an important role to play
in the regulation of populations.
(b) The elimination of any species can have
unpredictable effects on the balance of
nature.
(c) The pests themselves are part of the
food chain.
(d) These insects have been introduced to
the area by human activities.
(e) Elimination of these insects would
require the use of insecticides that kill a
wide range of insects.
6) It can be inferred that the passage is
(a) Part of an article in a scientific journal.
(b) Extracted from the minutes of a nature
club.
(c) Part of a speech delivered to an
educated audiences.
(d) A speech delivered in a court of law.
(e) From a polemical article published in a
magazine.
7) The purpose of the final paragraph is
(a) To sum up the main points of the
authors argument
(b) To urge a solution to an increasingly
pressing problem
(c) To qualify the authors definition of an
important term
(d) To propose a program
(e) To suggest that man should not
intervene in natural environments
Passage-3.
The existence of mammals on the earth can be
traced back to at least the Triassic time. The
rate of development was retarded, till
evolutional change suddenly accelerated in the
oldest Paleocene. This resulted in an increase in
average size, larger mental capacity, and
special adaptations for different modes of life,
during the Eocene time. Further improvement
was seen during the Oligocene Epoch, with the

Verbal Ability Class Assignment - 5

Page 71

appearance of some new lines and extinction of


other of various groups and a continued
approach toward modern characters. It is in
the Miocene time that the mammals reached
their peak with reference to variety and size.
The ability of the mammals to adapt to various
modes of life finds a parallel in the reptiles of
the Mesozoic time, and apart from their
greater intelligence, the mammals apparently
have not done much better than the
corresponding reptilian forms. Undoubtedly
the bat is a better flying animal than the
pterosaur, but at the same time the dolphin
and whale are hardly more fish like than the
ichthyosaur. Quite a few of the swift-running
mammals inhabiting the plains, like the horse
and the antelope, must excel any of the
dinosaurs. Although the tyrannosaur was a
more weighty and robust carnivore than
perhaps any carnivorous mammal, the lion and
the tiger, by virtue of their superior brain are
far more efficient and dangerous beasts of
prey. It is significant to note that various
species of mammals gradually adapted
themselves to various kinds of lifestyles, some
took to grazing on the plains and were able to
run swiftly (horse, deer, bison), others started
living in rivers and swamps (hippopotamus,
beaver), inhabiting trees (sloth, monkey),
burrowing underground (rodent, mole),
feeding on flesh (tiger, wolf), swimming in the
water (dolphin, whale, seal), and flying in the
air (bat). Human beings on account of their
superior brain have been able to harness
mechanical methods to conquer the physical
world and adapt to any set of conditions.
Such adaptation to different conditions leads
to a gradual change in form and structure. This
is a biological characteristic of the youthful,
plastic stage of a group. It is seen that early in
its evolutional cycle animals possess the
capacity for change, but as the animal
progresses in its cycle becoming old and fixed,
this capacity for change disappears. The
generalized types of organisms retain longest
that ability to make adjustments when
required, and it is from them that new, fecund
stocks take origin certainly not from any

specialized end products. With reference to


mammals, we see their birth, plastic spread in
many directions, increased specialization, and
in some cases, extinction; this is a characteristic
of the evolution of life, which can be seen in the
geologic record of life.
JET-2012
8) From the following, choose the most
appropriate title for the above passage.
(a) From Dinosaur to Man
(b) Adaptation and Extinction
(c) The Superior Mammals
(d) The Geologic Life Span
9) From the above passage, we can infer that,
the pterosaur:
(a) Resembled the Bat
(b) Was a Mesozoic mammal
(c) Was a flying reptile
(d) Inhabited the seas
10) As inferred from the passage, the largest
numbers of mammals were found in
which of the following periods?
(a) Eocene period
(b) Oligocene epoch
(c) Pliocene period
(d) Miocene period
11) It is clear from the passage, that the
evidence used to discuss the life of past
time periods:
(a) Was developed by Charles Darwin
(b) Has been negated by more recent
evidence
(c) Was never truly established
(d) Is based on Oligocene Epoch
Passage-4
The terms intelligence augmentation and
intelligence amplification evoke images of
human beings with computer chips embedded
in their skulls or bizarre accoutrements
attached to their heads. However, according to
an article entitled Get Smart by Jamais Cascio,
human beings ability to augment their
intelligence is precisely the prowess which has

Verbal Ability Class Assignment - 5

Page 72

empowered us to survive a series of convulsive


glacial events evinced by the last ice age.
Neurophysiologist William Calvin asserts that
the human species continues to evolve
cognitively and to create its own cognitive
evolution in two basic ways: external and
internal.
Cascio states that humans have been
externally augmenting their intelligence for
millennia. By developing written language, we
boosted our capacity to share information over
space and time. Other advancements, such as
agricultural and industrial technologies,
reduced the exigencies of manual labor.
Current external digital systems augment
human intelligence by allowing us to perform
tasks that would be unfeasible with recourse
only to the rational skills of a singular human
brain. Cascio cites as examples the powerful
simulations and massive data sets (which)
allow physicists to visualize, understand, and
debate models of an 11dimension universe,
realtime data from satellites, global
environmental databases, and highresolution
models (which) allow geophysicists to
recognize the subtle signs of longterm
changes to the planet, and similar manmade
interactions which have the functional effect of
augmenting human intelligence. Conceivable
potential software could incorporate individual
attention filters or focus assistants which
would discern and highlight your individual
preferences in a computer display, permitting
you to focus and direct your computer searches
more efficiently than you do now. It could
incorporate individualized planning and
foresight systems which could allow people to
play whatif with their life choices. Such
systems could coevolve with people to
produce intimate technologies which would
become something akin to collaborative
intuition, through webbased information
systems with personalized components,
according to Cascio.
Somewhat more problematic in social terms
might be pharmacological intelligence
augmentation, evoking Brave New World
nightmares pharmaceutically placated

people
tranquilized
to
zombielike
subservience to the collective and a central
bureaucracy dedicated to its own continued
survival. However, as with external cognitive
augmentation, the future has arrived in the
form of, for example, ADD drugs,
pharmaceutical agents which mitigate sleep
disorders, and antidepressants, all of which
enhance human problemsolving ability and
cognitive efficiency. According to Cascio,
people who dont know about (such drugs) or
dont want to use them will face stiffer
competition from people who do. From the
perspective of a culture immersed in athletic
doping wars, the use of such drugs may seem
like cheating. From the perspective of those
who find theyre much more productive using
this form of enhancement, its no more
cheating than getting a faster computer or a
better education.
Cognitive amplification, whether by external or
internal means, may constitute evolution, if
Calvins assertion is correct. Some societies
may readily embrace it, while others may shy
away. As science fiction writer William Gibson
observes, The future is already here; its just
unevenly distributed.
12) The author is mainly concerned about
(a) Various dangers of intelligence
augmentation.
(b) The advantages of intelligence
augmentation.
(c) The basic methods of intelligence
augmentation.
(d) Some scientists who are working on
intelligence augmentation.
(e) The differences between external and
internal intelligence augmentation.
13) The authors use of the phrase
somewhat more problematic in social
terms refers to
(a) The difficulty of making cognitive
enhancement widely available.
(b) The difficulty of making
pharmacological enhancement socially
acceptable.

Verbal Ability Class Assignment - 5

Page 73

(c) Equalizing
cognitive
competitive
advantages among social groups.
(d) Bureaucracies which hamper cognitive
enhancement activities.
(e) The relationship between external and
internal intelligence augmentation.
14) According to the passage the italicized
word placated most closely means
(a) Deprived.
(b) Enhanced.
(c) Cured.
(d) Assisted.
(e) Quieted.
15) In the context of the passage, which of
the following best articulates the
authors opinion?
(a) Intelligence amplification by external
means might be more difficult to achieve
than by internal means.
(b) Cognitive augmentation does not really
constitute evolution.
(c) Some people consider intelligence
enhancement to be a form of cheating.
(d) External and internal intelligence
enhancement
might
constitute
evolution in cultures that accept them.
(e) Personalized software could be misused
by a bureaucracy intent on its own
continued survival.
16) The primary purpose of the passage is to
(a) Describe different kinds of intelligence
enhancement.
(b) Discuss societys reactions to
pharmacological cognitive
augmentation.
(c) Examine the differences between
external and internal intelligence
enhancement.
(d) Dispel misgivings about humanitys
attempts at creating its own evolution.
(e) Illustrate the limitations of external
intelligence augmentation.

Verbal Ability Class Assignment - 5

Page 74

VOCABULARY
REVISION OF CLASS ASSIGNMENT - 4
Fill in the blanks with correct words
1.
2.
3.
4.

Elude
Malady
Tractum
Cudgel

5.
6.
7.
8.

_________________
_________________
_________________
_________________

Phrasal verbs
1. Complete with one of these phrasal verbs:
be through, go on, fill in, take off, stay out,
speak up.
A. Could you ........... this application
form, please?
B. I ll never talk to you again. We..........!
C. If you don t .........., we can t hear you.
D. I m tired because I ......... too late last
night.
E. The plane ................late because of the
bad weather.
2.

Complete with one of these phrasal verbs:


go off, put off, see off, take off, turn off.
A. Let s go to the airport to .......... them
B. The plane doesn t .......... till 5 o clock.
C. He was sleeping soundly when the alarm
clock..........
D. The meeting has been .......... till next
month.
E. Don t forget to .......... all the lights when
you leave.

3.

Complete with: carry on, get on, hold on,


put on, try on.
A. Hi! Is Mr. Knight in? .........., I ll call him.
B. Excuse me, could I .................. this dress,
please?
C. How are you ................... at college?
D. Are you still ...................... with your
tennis lessons?
E. It was a bit chilly, so she.................... her
jacket.

4.

Complete with: give up, hang up, look up,


wake up, wash up.

Beseech
Litigious
Tincture
Anthro

__________________
__________________
__________________
__________________

A. The kind of housework I hate most


is............
B. If he rings back, just ..........
C. I didn t know that word, so I .......... it in a
dictionary.
D. Don t ................ You can do it if you try
hard.
E. When I ................... in the middle of the
night, I had some temperature.
5.

Match the phrasal verbs with their


corresponding synonyms.
1. put off
a. cancel
2. call off
b. switch off
3. look up
c. postpone
4. go off
d. continue
5. carry on
e. explode
6. turn off
f. check

Fill in the correct form of one of the phrasal


verbs .
1. I looked for my keys everywhere but I couldnt
_____________ where I put them
2. I _____________ you at the party but I couldnt
see you.
3. Although he tried to eat less he __________
some weight during the holidays
4. I _____________ Karen and her new boyfriend
when I went shopping
5. Over a hundred people ______________ for
the news conference
6. Our plane ________________ an hour late
because of the fog.
7. My son drove me crazy about buying a new
bike, so finally I________________.
8. Erich is very sick at the moment but I am sure

Verbal Ability Class Assignment - 5

Page 75

hell ________________
9. She shouldnt ____________________ him like
that. Youre not his boss.
10. I understand you opinion but I have to
________________ Linda on this question.

Vocabulary for describing


character and personality
The words and expressions in the box can all be
used informally to describe different kinds of
people. Use them to complete sentences 1 20.
Note that many of the words / expressions have
a negative connotation and are not very polite,
so you should be careful how you use them!
Anorak, bigmouth, bunny boiler, busybody,
chatterbox, chinless wonder, clock-watcher,
couch potato, crank, creep, daydreamer, Don,
Juan, eager, beaver, early bird, golden boy,
happy camper, life and soul of the party, moaning
Minnie, pain in the neck, rolling stone, rough
diamond, salt of the earth, Scrooge scrounger,
skiver, slave driver, smart Alec, smart cookie,
stuffed shirt, tear away, troublemaker,
wallflower, wet blanket, wimp, wolf in sheep's
clothing
1. Nobody likes Peter very much because he's so
annoying. He's a right __________!
2. Andy is so boring. Did you know that his idea of
a perfect day is going to the station to collect
train registration numbers? What a / an
__________!
3. I know that you don't like your job very much,
but I wish you would stop complaining about it
all the time. Don't be such a / an __________!
4. Imelda loves working here: she's a real
__________.
5. Alan is an excellent and intelligent manager
who runs the department well and deals
effectively with any problems that come up.
Everyone agrees that he's a / an __________.
6. You've been sitting in front of the television for

almost four hours. Why don't you turn it off


and go for a walk? You're turning into a / an
__________.
7. We were having a wonderful evening until
Anne joined us. Why does she have to be so
negative about everything all the time? She's
such a / an __________!
8. Don't be such a /an __________! If you
concentrated instead of speaking all the time,
you would get more work done.
9. If you want some help, ask Imelda. She's always
happy and willing to help out: she's a real
__________!
10. I hope Rick comes out with us tonight. He's
such good fun, always the __________.
11. Poor Samantha is a bit of a / an __________.
She would have much more fun and would get
to know more people if she had more
confidence.
12. Don is a bit of a / an __________. He never
eats vegetables because he thinks they slow
down your brain!
13. Don't be such a / an __________! You've only
got a small cut on your hand; you haven't lost
a whole arm!
14. All the newspapers are writing about Gordon
Stapleton. He's the new __________ of
English football.
15. When Laurence ended his relationship with
Mandy, she refused to accept it and started
sending him insulting letters. Then one day
she went to his house and threw a brick
through his window! I never realised she was
such a __________!
16. My line manager Mr Burton is a real
__________. Yesterday he made us work for
six hours without a break, and wouldn't let us
leave until 7 o'clock.
17. Maureen is the __________ in this company.
She starts work at 7 o'clock, two hours before
anyone else arrives.
18. I'm afraid my son has become a bit of a / an
__________. He stays out all night with his
friends and he never listens to a word I say.
19. All the girls in the office love Daniel, and he
loves them right back! He's a regular
__________.
20. Mrs. Ranscombe is such a /an __________. I
wish she would stop interfering in my private
life!

Verbal Ability Class Assignment - 5

Page 76

GENERAL AWARENESS CLASS ASSIGNMENT 1


History, Polity, Geography
History
1.

Which was the largest site of Indus Civilization?


(a) Mohenjodaro
(b) Lothal
(c) Chanhudaro
(d) Rakhigarhi

2.

Which is the oldest text in the world?


(a) Yajur Veda
(b) Atharva Veda
(c) Rig Veda
(d) Sama Veda

3.

4.

5.

6.

Which Veda contains cure of diseases?


(a) Sama Veda
(b) Atharva Veda
(c) Rig Veda
(d) Yajur Veda
Ajatasatru was son of:
(a) Bimbisara
(c) Shisunaga

(b) Udayin
(d) None of above

Alexander invaded India in :


(a) 526 B.C
(b) 326 A.D
(c) 326 B.C
(d) 26 B.C.
The famous Indian astronomer Aryabhatta (476
520 AD) lived during the reign of:
(a) Chandragupta Vikramaditya
(b) Ashoka
(c) Harsh Vardhana
(d) Kanishka

10. Delhis Qutab Minar, the tallest (250ft or 72.5 m)


free standing stone tower in the world was
completed by:
(a) Qutab-u-din Aibak
(b) Feroze Tughlaq
(c) Iltutmish
(d) Alauddin Khilji
11. Founder of the Lodhi Dynasty was:
(a) Bahlol Lodhi
(b) Daulat Khan Lodhi
(c) Abraham Lodhi
(d) Ibrahim Lodhi
12. First battle of Panipat was fought between?
(a) Babur & Lodi
(b) Akbar & Hemu
(c) Mughal & British
(d) Akbar & Lodi
13. Grand Trunk Road was built by:
(a) Babar
(b) Sher Shah Suri
(c) Rana Sanga
(d) Birbal
14. When Humayun was succeeded by Akbar, he
was:
(a) 13 years old
(b) 14 years old
(c) 15 years old
(d) 16 years old
15. Second battle of Panipat was fought between:
(a) Babur & Lodi
(b) Akbar & Hemu
(c) Mughal & British
(d) Akbar & Lodi

7.

Arthashastra was written by:


(a) Manu
(b) Panini
(c) Harsha Vardhan
(d) Kautilya

16. Din-e-Ellahi was founded by:


(a) Aurangzeb
(b) Jehangir
(c) Akbar
(d) Shah Jahan

8.

Chandragupta Maurya was succeeded by:


(a) Sannidhata
(b) Ashoka
(c) Hemusara
(d) Bindusara

17. With whose permission did the English set up


their first factory in Surat?
(a) Akbar
(b) Jahangir
(c) Shahjahan
(d) Aurangzeb

9.

To which king belongs the Lion capital at


Sarnath?
(a) Chandragupta
(b) Kanishka
(c) Ashoka
(d) Harsha

18. The Red Fort was built by :


(a) Akbar
(b) Shahjahan
(c) Jahangir
(d) Sher Shah Suri
19. Last Sikh Guru was:
(a) Guru Ramdas

GENERAL AWARENESS CLASS ASSIGNMENT - 1

(b) Guru Govind Singh


Page 77

(c) Guru Ranjit Singh

(d) Guru Angad

(c) Landlordism

20. Last of the Mughal Ruler was:


(a) Bahadur Shah Jafar
(b) Aurangzeb
(c) Shah Alam
(d) Faroq Siyar
21. The Indian National Congress was founded in
1885 by:
(a) A.O. Hume
(b) Motilal Nehru
(c) Mahatma Gandhi
(d) Dadabhai Naoroji
22. The treaty of Versailles humiliated:
(a) Austria
(b) France
(c) Germany
(d) England

(d) Apartheid

24. The Opium wars were fought between:


(a) Britain & China
(b) Britain & India
(c) Britain & Poland
(d) India & China
25. Waterloo is located in:
(a) China
(b) Belgium
(c) France
(d) Germany
26. The Second World War started in the year:
(a) 1939
(b) 1940
(c) 1941
(d) 1941

23. The American Civil War saw the end of:


(a) Slavery
(b) Monarchy

Polity
27. The constituent assembly
constitution on:
(a) 15th August 1947
(b) 26th January, 1946
(c) 26th November 1949
(d) 26th January 1950

adopted

our

28. Who is the custodian of the constitution of


India?
(a) President
(b) Parliament
(c) Vice President
(d) Chief Justice
29. The President of India exercises his power on the
advice of the:
(a) Council of Ministers
(b) Union Cabinet
(c) Vice President
(d) Parliament
30. Article 370 of the Constitution deals with:
(a) Centre-State Relations
(b) Accession of Jammu and Kashmir in India
(c) Quantum of State Authority
(d) Special provision in respect of J & K
31. Who is the custodian of the Fundamental Rights
embodied in the Indian Constitution?
(a) President of India
(b) Prime Minister

(c) Supreme Court of India


(d) State Assemblies
32. The meetings of Lok Sabha are presided over by:
(a) Prime Minister
(b) Speaker
(c) Vice President of India (d) President of India
33. The term of the Rajya Sabha comes to an end
after:
(a) 2 years
(b) 4 years
(c) 5 years
(d) Is not subject to dissolution as it is a
permanent body
34. The fundamental rights in
Constitution are enjoyed by:
(a) Aliens only
(b) Adults only
(c) Indian citizens
(d) All male citizens of India

the

Indian

35. The minimum age for a member of the Lok


Sabha should be:
(a) 18 years
(b) 21 years
(c) 25 years
(d) 30 years

GENERAL AWARENESS CLASS ASSIGNMENT - 1

Page 78

36. The International Court of Justice is located at:


(a) The Hague
(b) Geneva
(c) Amsterdam
(d) Vienna
37. Who appoints the Governors in India?
(a) Prime Minister
(b) Union Cabinet
(c) President
(d) Vice President
38. A candidate for the office of the president of
India should not be less than:
(a) 35 years
(b) 30 years
(c) 25 years
(d) 20 years
39. In the Indian constitution who has no power
over money bill (Budget)?
(a) President
(b) Lok Sabha
(c) Rajya Sabha

(d) Neither the Lok Sabha nor the Rajya Sabha


40. Which of the following is a Directive Principle of
State Policy?
(a) Equitable distribution of wealth or the
socialistic pattern of society
(b) Provision for Uniform Civil Code
(c) Free and compulsory education for all
children below the 14 years of age
(d) All of the above
41. Chairman of Constituent Assembly was:
(a) Rajendra Prasad
(b) Jawaharlal Nehru
(c) Mahatma Gandhi
(d) B.R. Ambedkar

GEOGRAPHY
42. Which of the following is true regarding
International Date Line?
(a) Longitude
(b) It is a straight Line
(c) It is a big circle
(d) It is a curved line beyond earth
43. On which of the following planets is the water
cycle available?
(a) Venus
(b) Earth
(c) Jupiter
(d) Mars
44. Ozone Hole in the atmosphere is largely caused
by the presence of:
(a) Oxygen
(b) Hydrogen
(c) Chloro-floro-carbon
(d) Radio Active Waste
45. The deepest ocean of the world is:
(a) Atlantic Ocean
(b) Arctic Ocean
(c) Indian Ocean
(d) Pacific Ocean

47. The largest ocean of the world is:


(a) Atlantic
(b) Indian
(c) Arctic
(d) Pacific
48. Which of the following is the hottest planet?
(a) Venus
(b) Mercury
(c) Jupiter
(d) Mars
49. In the context of time, GMT means:
(a) General Merdian Time
(b) Greenwich Mean Time
(c) Global Mean Time
(d) None of these
50. The Himalayan Mountain Range is an example
of:
(a) Volcanic mountains
(b) Residual Mountains
(c) Block Mountains
(d) Folded Mountains

46. The light from the Sun reaches the earth in


about:
(a) 8 seconds
(b) 8 minutes
(c) 10 seconds
(d) 10 minutes

GENERAL AWARENESS CLASS ASSIGNMENT - 1

Page 79

CURRENT AFFAIRS - I
51. Which US-based solar power company won a bid
to sell solar power in India?
(a) SunEdison
(b) SunSol
(c) SunPow
(d) SolPow
52. Which railway station became the first visuallychallenged-friendly station in India?
(a) Mysuru
(b) Itarsi
(c) Vizag
(d) Nagpur
53. Googles project to provide internet
connectivity using large balloons is..
(a) Project Balloon
(b) Project Loon
(c) Project Internet
(d) Project Data Free
54. Which automobile Car Company has roped in
Argentinian football legend Lionel Messi as the
new global brand ambassador?
(a) Mahindra & Mahindra
(b) Toyota
(c) Tata Motors
(d) Chevrolet

(c) South Africa


(d)France
58. The APEC Summit for 2015 was held in which
city?
(a) Manila
(b) Dubai
(c) Tokyo
(d) Toronto
59. The Magic of the Lost Temple, which is the
name of a newly released childrens book, is the
25th book of which noted author?
(a) Jhumpa Lahiri
(b) Sudha Murty
(c) Sheetal Singh
(d) Rashmi Vajpayee
60. What is the name of the Advanced Torpedo
Defence System developed by the DRDO?
(a) Mareech
(b) Khamb
(c) Sheen
(d) Jaitsh

55. What is Indias position in the recently released


annual report on worlds most valuable nation
brands compiled by Brand Finance?
(a) 6th
(b) 7th
(c) 8th
(d) 9th
56. Mens Hockey World League final 2015 was held
at:
(a) Raipur
(b) Amritsar
(c) Mohali
(d) Jaipur
57. The 2015 Rugby World Cup (Men) was won by
which country on 31 October 2015?
(a) Australia
(b) New Zealand

GENERAL AWARENESS CLASS ASSIGNMENT - 1

Page 80

GENERA AWARENESS CLASS ASSIGNMENT 2


BRANDS AND TAGLINES
Revision of Practice Test Book 1
1.

Supreme Court Judges retire upon attaining the


age of:
(a) 65 years
(b) 62 years
(c) 68 years
(d) 70 years

2.

What was the main feature of Indus valley


civilization?
(a) Rural Planning
(b) Town Planning
(c) Art and Architecture
(d) Administrative System

3.

The second atom bomb was dropped on


Nagasaki on which date?
(a) Aug. 6, 1914
(b) Aug. 9, 1945
(c) Aug. 6, 1943
(d) Aug. 9, 1943

4.

ISO-9000 is a :
(a) Quality Standard Mark (b) Space Project
(c) Trade Technique
(d) Loan Security

5.

Big Cinemas an entertainment company


belongs to which of the following business
groups of India?
(a) Tatas
(b) Reliance Group
(c) Mahindra and Mahindra (d) Dalmia Group

6.

7.

8.

Who is the brand ambassador for National


Tobacco Control Campaign?
(a) Rahul Dravid
(b) Kapil Dev
(c) Sachin Tendulkar
(d) Anil Kumble
Who invented Dynamite?
(a) Sir Alexander Graham Bell
(b) Benjamin Franklin
(c) Thomas Alva Edison
(d) Alfred B. Nobel
On which date is United Nations Day observed?
(a) 4 October
(b) 12 October
(c) 19 October
(d) 24 October

9.

On which date is Human Rights Day observed?


(a) 4 December
(b) 10 December
(c) 17 December
(d) 23 December

10. Who is the recipient of Rajiv Gandhi Khel Ratna


Award 2015?
(a) P.V. Sindhu
(b) Virat Kohli
(c) Sania Mirza
(d) Abhijeet Gupta
11. The boundary line b/w India and Pakistan is
called as:
(a)Line of control
(b) Radcliffe line
(c) 38th Parallel
(d) Durand Line
12. Match the columns:
State
Capital
(A) Uttaranchal
1. Ranchi
(B) Chhattisgarh
2. Dispur
(C) Jharkhand
3. Dehradun
(D) Assam
4. Raipur
(a) A - 3, B - 4, C - 1, D - 2
(b) A - 4, B - 3, C - 1, D - 2
(c) A - 3, B - 4, C - 2, D - 1
(d) A - 4, B - 3, C - 2, D 1
13. Security of Delhi Metro is handled by?
(a) Delhi Police
(b) CISF
(c) Paramilitary forces
(d) SFF
14. When a bank returns a cheque unpaid, it is
called:
(a) Payment of the cheque
(b) Drawing of the cheque
(c) Cancelling of the cheque
(d) Dishonour of the cheque
15. The compulsory Education Act ensures
education to the children up to the age of?
(a) 10 years
(b) 12 years
(c) 8 years
(d) none of these

GENERAL AWARENESS CLASS ASSIGNMENT - 2

Page 81

BRANDS AND TAGLINES


Write the parent company of the following brands
Fair and Lovely
Kissan
Ariel
Vicks
Gillette Dove
Kwality Walls
Head & Shoulders
Duracell
Olay
Lakme
Oral-B
Pears
Surf Excel
Brooke Bond Taj
Mahal Wella
Pantene
Clinic Plus
Tide
Knorr
Bru
Pure it
Brooke Bond Red
Label
AXE deodorant

Pepsodent
Lifebuoy
Rin Detergent
Pampers
Vaseline
Close up Toothpaste
Ambi pur
Sunsilk
Vim
Tresemme

Given below are famous celebrities. List down all the possible brands that you can think of that are
promoted by these actors.

__________________________________________
__________________________________________
__________________________________________
__________________________________________
__________________________________________
__________________________________________
__________________________________________
__________________________________________
__________________________________________
__________________________________________
__________________________________________
__________________________________________
__________________________________________
__________________________________________
__________________________________________
__________________________________________
__________________________________________
__________________________________________

GENERAL AWARENESS CLASS ASSIGNMENT - 2

Page 82

__________________________________________
__________________________________________
__________________________________________
__________________________________________
__________________________________________
__________________________________________
__________________________________________
__________________________________________
__________________________________________
__________________________________________
__________________________________________
__________________________________________
__________________________________________
__________________________________________
__________________________________________
__________________________________________
__________________________________________
__________________________________________
__________________________________________
__________________________________________
__________________________________________
__________________________________________
__________________________________________
__________________________________________
__________________________________________
__________________________________________
__________________________________________

__________________________________________
__________________________________________
__________________________________________
__________________________________________
__________________________________________
__________________________________________
__________________________________________
__________________________________________
__________________________________________

GENERAL AWARENESS CLASS ASSIGNMENT - 2

Page 83

__________________________________________
__________________________________________
__________________________________________
__________________________________________
__________________________________________
__________________________________________
__________________________________________
__________________________________________
__________________________________________

Name the current brand ambassadors of the following brands/companies.


Anti-tobacco campaign
Titan Raga
_________________
Bitia Bachao campaign
Whirlpool Refrigerators
_________________
Polio campaign
Boost
_________________
Clear Shampoo
_________________
BSF
_________________
Identify the Company from the given tagline
Fresh N Juicy
_________________
Taste The Feeling
_________________
The taste of India
_________________
Dimag ki Batti Jala de
_________________
Tedha hai Par Mera Hai
_________________
Desh ka namak
_________________
India's International Bank _________________
Daag Acche hain
_________________
Har Ghar Kuchch Kahta hai _________________
The best a man can get
_________________
Impossible is nothing
_________________
Just do it
_________________

The Complete Man


Tyres with Muscle
Do you have it in you
Bajaate raho
Born Tough
Bond with the best

Identify the manufacturer of the following two wheelers


Discover _____________
Stunner _____________
Pulsar _____________
Access _____________
Hayate _____________
Platina _____________
Fazer
_____________
Activa _____________
Aviator _____________
Crux
_____________
Unicorn _____________
Gladiator_____________
Dio
_____________
Glamour _____________

Identify the International corporates form its tagline


Soooo!! GOOOOD!!!!!
_________________
Im lovin it
_________________
Pizzas and much more
_________________
Yeh hai Rishton ka time
_________________
Applying thought
_________________

_________________
_________________
_________________

_________________
_________________
_________________
_________________
_________________
_________________

Karizma _____________
Splendor _____________
Passion _____________
Apache _____________
Wego _____________
Jupiter _____________

Think different
Everything is possible
Innovation and you
Wires that dont catch fire
Inspire the next

GENERAL AWARENESS CLASS ASSIGNMENT - 2

_________________
_________________
_________________
_________________
_________________
Page 84

BRANDS AND TAGLINES QUIZ


1.

Who among the following is not a co-founder


of Facebook?
(a) Mark Zuckerberg
(b) Dustin Moskovitz
(c) Steve Wozniak
(d) Chris Hughes

7.

Which of the following is the largest employer


of India?
(a) Indian Army
(b) Indian Oil
(c) India Post
(d) Indian Railways

2.

Nothing like anything is the tagline of:


(a) Nokia
(b) Samsung
(c) Micromax
(d) Spice

8.

Which of the following does not belong to the


Bennet and Coleman group?
(a) Times of India
(b) Economic Times
(c) Femina
(d) Outlook

3.

Which of the following is the tagline of the


sports company Nike?
(a) All in
(b) Just do it
(c) Impossible is nothing
(d) Just Imagine

9.

Who heads the Tata Group at present?


(a) Ratan Tata
(b) Russi Mody
(c) Cyrus Mistry
(d) None of these

4.

Find the odd one out:


(a) Mountain Dew
(c) 7UP

(b) Slice
(d) Maaza

5.

'Kitchens of India' ready to eat food, is the


product of:
(a) Hamdard
(b) ITC Limited
(c) Britannia
(d) MTR

6.

Dettol is a product of:


(a) Procter and Gamble
(c) Johnson & Johnson

10. Consider the following statements:


1. WhatsApp has been acquired by Facebook.
2. Motorola has been acquired by Lenovo.
Which of the statements given is/are correct?
(a) 2 only
(b) 1 only
(c) Both 1 & 2
(d) Neither 1 nor 2

(b) HUL
(d) Rickitt-Benckiser

CURRENT AFFAIRS II
11. What is the name of the first indigenouslydesigned and built torpedo launch and recovery
vessel that was commissioned into the Indian
Navy?

(a) INS Akash


(b) INS Vyom
(c) INS Ashtradharini
(d) INS Gagan

14. In which city, U.N. Climate Change Summit held


in December 2015?

(a) New York


(c) London

12. Who was nominated as the new head of the


Intergovernmental Panel on Climate Change
(IPCC)?

(a) Hoesung Lee


(c) Lime Koex

(b) Shashank Manohar


(c) Kiran More
(d) Chetan Chauhan

(b) Kim Xiang


(d) Sue Nali

(b) Madrid
(d) Paris

15. Who has been selected for prestigious 2015


Nobel Prize in Literature?

(a) Svetlana Alexievich (b) Maria Upton


(c) Roes White
(d) Karina Benitez

13. Who is the new president of BCCI?

(a) Rajeev Shukla


GENERAL AWARENESS CLASS ASSIGNMENT - 2

Page 85

GENERAL AWARENESS CLASS ASSIGNMENT 3


Books/Authors, Important Days, Invention, Management Communication-I

Revision of Practice Test Book 2


1.

The Britishers came to India as traders and


formed company named:
(a) Indo-British Company
(b) The Great Britain Company
(c) Eastern India Company
(d) East India Company

2.

This form of writing was used by the Egyptians:


(a) Pictograms
(b) Hieroglyphics
(c) Ebonics
(d) Cuneiform

3.

The members of Lok Sabha hold office for a


term of__________.
(a) 4 years (b) 5 years (c) 6 years (d) 3 years

4.

Who is the author of the book A SuitableBoy?


(a) Husain Haqqani
(b) Vikram Seth
(c) Ishrat Husain
(d) M.J. Akbar

5.

Match the following books with their authors:


(A) Maulana Abul Kalam Azad
(B) Jawaharlal Nehru
(C) Rajendra Prasad
(D) Bhagat Singh

(i) India divided


(ii) Discovery of India
(iii) India Wins Freedom
(iv) Why I am an Atheist

(a) A i, B ii, C iii, D iv


(b) A iii, B ii, C I, D iv
(c) A I, B ii, C iv, D iii
(d) A iii, B iv, C I, D ii
6.

What did J. B. Dunlop invented?


(a) Pneumatic rubber tire
(b) Automobile wheel rim
(c) Rubber boot
(d) Model airplanes

7.

Which scientist discovered the radio active


element Radium?
(a) Isaac Newton
(b) Albert Einstein
(c) Benjamin Franklin
(d) Marie Curie

8.

Fields Medal is given in which subject?

(a) Physics
(c) Botany
9.

(b) Chemistry
(d) Mathematics

What is Mendeleevs contribution to the


modern day science?
(a) He invented the Atom Bomb
(b) He created the first ever clone
(c) He created the periodic table
(d) He developed the microwave oven

10. The World's largest river is:


(a) Brahmaputra
(b) Amazon
(c) Nile
(d) Mississippi
11. Which country is the highest producer of cars in
the world in as per latest data release?
(a) U.S.A
(b) Germany
(c) Japan
(d) China
12. World Bank is also known as:
(a) International Bank for Reconstruction and
Development
(b) International Bank for Rehabilitation and
Development
(c) International Bank for Refinance and
Development
(d) International Bank for Research and
Development
13. Park Avenue, Parx, Colorplus, Manzoni etc. are
the brands of which company?
(a) Future Group
(b) Levi Strauss
(c) Raymond
(d) Arvind Mills
14. Who has been appointed as the Brand
Ambassador of Border Security Force?
(a) M S Dhoni
(b) Virat Kohli
(c) Rohit Sharma
(d) Shikhar Dhawan
15. Pradhan Mantri Jan Dhan Yojna is associated
with:

GENERAL AWARENESS CLASS ASSIGNMENT - 3

Page 86

(a) Ration
(c) Voting

18. How many rings are present on the Olympic


flag?
(a) Two (b) Three (c) Four (d) Five

(b) Banking
(d) Identity

16. Who is called the Iron Man of India?


(a) Mahatma Gandhi
(b) Bhagat Singh
(c) Vallabh Bhai Patel
(d) Udham Singh
17. The UNO was formed in:
(a) 1935 (b) 1945 (c) 1955 (d) 1965

19. Which country is called the sugar bowl of the


world?
(a) China (b) Cuba (c) India (d) Pakistan
20. Pali is a:
(a) Computer Program
(c) Computer Virus

(b) Language
(d) None of these

Books/Authors
1.

Playing it my way is the autobiography of


which of the following sports person?
(a) M.C. Mary Kom
(b) Yuvraj Singh
(c) Sachin Tendulkar
(d) Milkha Singh

2.

As I See is a book written by:


(a) Kiran Bedi
(b) Arvind Kejriwal
(c) Narendra Modi
(d) Manmohan Singh

3.

Asthadhyayi is a book written by:


(a) Valmiki
(b) Ved Vyas
(c) Panini
(d) Tulsi Daas

4.

The book Things a little bird told me is a book


from the co-founder of Twitter. The name of
the co-founder is:
(a) Biz Stone
(b) Mark Zuckerberg
(c) Steve jobs
(d) None of these

5.

6.

7.

What is the name of the autobiography of


Naseeruddin Shah
(a) Two Days Later
(b) Fir ek Sham Huyi
(c) And then One day
(d) Meri Daastaan
What is the name of the book written the
current RBI governor Raghuram Rajan?
(a) Fault Lines
(b) Indian Decline
(c) Economic Nightmare
(d) Reforms
Who is also known as the Indian Shakespeare?
(a) Chetan Bhagat
(b) Kalidaas
(c) R.K. Narayanan
(d) Kabir Daas

8.

Who wrote the India National Anthem?


(a) B.C. Chatterjee
(b) S.C. Naidu
(c) Rabindra Nath Tagore (d) Javed Akhtar

9.

Who wrote the Panchtantra?


(a) Valmiki
(b) Ved Vyas
(c) Vishnu Sharma
(d) Tulsi Daas

10. The White Tiger is a famous book from?


(a) Khushwant Singh
(b) Arvind Adiga
(c) Dadabhai Naoroji
(d) Nathuram Godse
11. Who is the Man Booker Prize winner of 2014?
(a) Eleanor Catton
(b) Shashi Tharoor
(c) V.S. Naipaul
(d) None of these
12. Sir Arthur Conan Doyle is a famous author. His
most famous creation is the character:
(a) Jams Bond
(b) Sherlock Holmes
(c) Mowgli
(d) Scrooge
13. Which one of the following literary works was
not written by R.K. Narayan?
(a) Malgudi Days
(b) Guide
(c) Swami and friends
(d) Gardner
14. Who is the author of Discovery of India and
Glimpses of World History?
(a) Mahatma Gandhi
(b) Dadabhai Naoroji
(c) Jawahar Lal Nehru
(d) Sardar Patel

GENERAL AWARENESS CLASS ASSIGNMENT - 3

Page 87

Important Days
1.

2.

When is Air Force Day celebrated in India?


(a) 4th October
(b) 9th November
th
(c) 15 August
(d) None of these

6.

When is National Hindi Diwas celebrated?


(a) 14th September
(b) 15th September
th
(c) 16 September
(d) None of these

When is sparrow day observed in India?


(a) 20th March
(b) 20th October
(c) 20th November
(d) 20th Fanuary

7.

Mandela Day is celebrated in honor of Nelson


Mandela
(the
famous
South
African
revolutionary, politician and philanthropist), on
his birthday every year. On which day is
Mandela Day celebrated?
(a) 17th July
(b) 18th July
th
(c) 19 July
(d) 20th July

8.

When is international Day of Non Violence


celebrated?
(a) 15th August
(b) 2nd October
th
(c) 26 January
(d) None of these

3.

International Womens Day is observed on:


(a) 8th March
(b) 8th April
th
(c) 8 May
(d) 8th June

4.

World Water Day is observed on:


(a) 24th May
(b) 24th June
th
(c) 24 July
(d) 22nd March

5.

Labour Day is observed on:


(a) 1st May
(b) 1st July
(c) 1st December
(d) 1st June

Inventions
1.

The first telescope was built by:


(a) Galileo
(b) Newton
(c) Hans Lippersheyt
(d) David Young

2.

Match the following:


I. Baird
II. Otis
III. Colt
IV. Tesla
V. Galileo

3.

i. Revolver
ii. Elevator
iii. Thermometer
iv. Television
v. Induction Coil

Write the names of the inventors of:


(a) Light Bulb
________________
(b) Computer
________________
(c) Telephone
________________
(d) Braille Language
________________
(e) Penicillin
________________
(f) Pasteurization
________________
(g) AK-47
________________
(h) Jet Engine
________________

GENERAL AWARENESS CLASS ASSIGNMENT - 3

Page 88

Management Communication-I
1.

2.

3.

Which of the following strategies is an example


of promotion for a new brand of hair oil?
(a) Packing it in eco-friendly bottles
(b) Creating a brand name
(c) Advertising it as adding shine to hair and
helping prevent hair fall
(d) Making it available only in barber shops and
beauty saloons
The OTC Exchange of India was modelled on
the lines of which stock exchange?
(a) NYSE
(b) NASDAQ
(c) Euronext
(d) SWX Swiss Exchange
Which of the following actions involves a senior
manager to perform a decisional role?
(a) Identifying trends that affect the company
(b) Allocating funds for developing new products
(c) Holding group meetings with his junior
colleagues
(d) Attending conferences about increasing corecompetency

4.

Which partners association with the company


is unknown to the general public?
(a) Secret partner
(b) Active partner
(c) Nominal partner
(d) Sleeping partner

5.

If two rival companies join hands in order to


secure a major contract, then this action can
best be described by which of the following
terms?
(a) Outsourcing
(b) Strategic alliance
(c) Networking
(d) Industry vertical

6.

Which of the following contracts is not


applicable in life insurance?
(a) Unilateral contract
(b) Indemnity contract
(c) Conditional contract
(d) None of these

7.

Employees who inform authorities of wrong


doings of their companies or co-workers are
known as:
(a) Wikileakers
(b) Whistle blowers
(c) Spokespersons
(d) Black sheep

8.

Under which of the following business


structures is there separation of ownership and
management?
(a) Company
(b) Sole proprietorship
(c) Partnership
(d) All business organizations

9.

Which of the following roles is performed by a


merchant bank?
(a) Providing loans, mainly to companies
(b) Disciplining companies over unethical
business practices
(c) Monitor activities of publicly listed companies
(d) Providing licenses to brokers for trading in the
stock market

10. Which of the following practices best defines


hedging?
(a) Reducing credit risks on international
transactions
(b) Minimizing the effects of global business cycle
(c) Using transfer pricing between subsidiaries
(d) Protecting against fluctuating exchange rates
11. Which is the highest level of need in the
Maslow theory of motivation:
(a) Physiological needs
(b) Social needs
(c) Self-actualization needs
(d) Safety needs
12. What is the full form of USP?
(a) Unit Sale Pivot
(b) Unique Selling Proposition
(c) Unique Selling Presumption
(d) Unit Sales Production

GENERAL AWARENESS CLASS ASSIGNMENT - 3

Page 89

Current Affairs-III
1.

Which of the following organizations published


the World Economic Situation and Prospects
(WESP) 2016 report?
(a) World Bank
(b) OECD
(c) United Nations
(d) WTO

2.

Which of the following countries has won the


first Blind T20 Asia Cricket Cup?
(a) Bangladesh
(b) Sri Lanka
(c) India
(d) Pakistan

3.

The first Ministerial Meeting of the Arab-India


Cooperation Forum took place in which
country?
(a) Kuwait
(b) Bahrain
(c) Oman
(d) Yemen

4.

5.

Which of the following is the first railway station


in India to have free high-speed Wi-Fi Internet
facility?
(a) Chennai Central station
(b) Mumbai Central station
(c) New Delhi station
(d) Kolkata station
Syed Mushtaq Ali trophy is related to which
sports?
(a) Hockey
(b) Badminton
(c) Volleyball
(d) Cricket

6.

Which of the following teams has won the 2016


Premier Badminton League (PBL)?
(a) Hyderabad Hunters
(b) Delhi Acers
(c) Mumbai Rockets
(d) Awade Warriors

7.

Which company has launched Indias first social


mobile wallet Udio?
(a) Oxigen
(b) Twitter
(c) TranServ
(d) Facebook

8.

Who has been named as the new CEO of Tata


Motors?
(a) Keyur Joshi
(b) Guenter Butschek
(c) John Leahy
(d) Tom Williams

9.

Who won the Nobel Prize in Economics for 2015


as announced on 12 October 2015?
(a) Angus Deaton
(b) Satrick Carl
(c) Mat Seamons
(d) Cathy Dennis

10. What is the name of the book written by former


Pakistani foreign minister Khurshid Mahmud
Kasuri?
(a) To the Brink & Back
(b) Neither a Hawk nor a Dove
(c) The Pakistan Paradox
(d) The Silk Roads

GENERAL AWARENESS CLASS ASSIGNMENT - 3

Page 90

GENERAL AWARENESS CLASS ASSIGNMENT 4


Awards, Sports, Science, Banking Awareness, World Bodies, PSUs

Revision of Practice Test Book 3


1.

Who was known as the Punjab Kesari (Lion of


the Punjab)?
(a) Lala Lajpat Rai
(b) Bhagat Singh
(c) Jatin Das
(d) Bipin Chandra Pal

2.

Who is popularly known as 'Father of Indian


Constitution'?
(a) Jawaharlal Nehru
(b) B.R. Ambedkar
(c) Mahatma Gandhi
(d) Bal Gangadhar Tilak

3.

What is the maximum number of seats in Lower


House of Parliament?
(a) 545
(b) 565
(c) 530
(d) 552

4.

What is the maximum number of seats in Upper


House of Parliament?
(a) 250
(b) 210
(c) 215
(d) 284

5.

Which is the second highest mountain peak in


the world?
(a) Kangchenjunga
(b) Mount Everest
(c) K2
(d) Lhotse

6.

What was the name of the first artificial satellite


that was launched by Russia?
(a) Sputnik 1
(b) Explorer 1
(c) HS-333
(d) MILSTAR

7.

Innovation and You is the tagline of which


company?
(a) Toshiba
(b) Sony
(c) Philips
(d) Panasonic

8.

The Banker to Every Indian is the tagline of


which bank?
(a) State Bank of India
(b) Union Bank
(c) Deutsche Bank
(d) Yes Bank

9.

The headquarters of Pepsico is located at:


(a) Redmond, Washington
(b) Detroit
(c) Purchase, New York
(d) Palo Alto, California

10. ____________ acquired Tropicana in 1998 and


Quaker Oats in 2001.
(a) Fanta
(b) Coca Cola
(c) Limca
(d) PepsiCo
11. Which FMCG major owns Real a beverage
brand?
(a) Nestle
(b) PepsiCo
(b) Dabur
(d) Coca-Cola
12. Which of the following is the correct
description of the Capital Market?
(a) Stock Markets and Bond Markets
(b) Banks and Insurance Companies
(c) RBI and Nationalized Banks
(d) Stock Markets and Banks
(e) None of these
13. Which of the following is not a mode of foreign
capital inflow in India?
(a) FDI
(b) NRI deposits
(c) FII
(d) None of these
14. Which of the following terms is used in the field
of finance and banking?
(a) Single Bond
(b) Quantum number
(c) Adjustment Credit
(d) Absolute zero
15. Many times we read a term CBS used in banking
operations. What is the full form of the letter 'C
in the term 'CBS'?
(a) Complete
(b) Continuous
(d) Core
(d) None of these

GENERAL AWARENESS CLASS ASSIGNMENT - 4

Page 91

16. Where would you find the Great Barrier Reef?


(a) India
(b) USA
(c) Russia
(d) Australia
17. The present Indian monetary system is based
on
(a) Gold Reserve System
(b) Proportional Reserve System
(c) Convertible Currency System
(d) Minimum Reserve System
18. Who is the author of the book The Test of My
Life?
(a) Yuvraj Singh
(b) Rahul Dravid

(c) Sachin Tendulkar

(d) Adam Gilchrist

19. Which of the following is the tallest building of


the world?
(a) Burj Khalifa
(b) Taipei 101
(c) Sears Towers
(d) Pentagon
20. What does URL stand for?
(a) Universal Remote Location
(b) Unidentified Resource Lookup
(c) Uniform Resource Locator
(d) None of these

Awards, Sports & Science


1.

The highest civilian award in India is:


(a) Bharat heera (b) Bharat Ratna
(c) Padma Shree (d) Padma Bhushan

2.

Who among the following has not won the


Bharat ratna Award?
(a) Lal Krishna Advani
(b) Morarji Desai
(c) Jawaharlal Nehru
(d) A.P.J. Abdul Kalam

3.

Which is Indias highest military honour?


(a) Param Vir Chakra
(b) Vir Chakra
(c) Maha Vir Chakra
(d) Bharat Ratna

4.

Who gets Dronacharya Award?


(a) Best Wrestler
(b) Best Batsman
(c) Best Tennis Player
(d) Best Coach

5.

On whose name are the Nobel Prizes named?


(a) Sir William Nobel
(b) Sir Arthur Nobel
(c) Sir Alfred Nobel
(d) Sir Anthony

6.

Who was the first foreigner to win the Bharat


Ratna?
(a) Benazir Bhutto
(b) Winston Churchill
(c) Nelson Madela
(d) Khan Abdul Gaffar Khan

7.

Which of the following awards has not been


conferred to Amitabh Bachchan?
(a) Padma Bhushan
(b) Padma Vibhushan
(c) Leign of Honour
(d) Unesco Pyramide con Marni

8.

Who was the first person to win Rajiv Gandhi


Khel Ratna Award?
(a) Geet Sethi
(b) Karnam Malleswari
(c) Kapil Dev
(d) Vishwanathan Anand

9.

What is the venue of Olympics 2016?


(a) Greece
(b) Rio (Brazil)
(c) Delhi (India)
(d) Tokyo (Japan)

10. Which of the following international tennis


tournaments is held on grass court?
(a) Australian Open
(b) Wimbledon
(c) French Open
(d) US Open
11. Which one among the following is the coral
group of islands of India?
(a) Andaman
(b) Lakshadweep
(c) Minicoy
(d) Nicobar
12. Who is the current World Chess Champion?

GENERAL AWARENESS CLASS ASSIGNMENT - 4

Page 92

(a) Vishwanathan Anand


(c) Garry Kasprov

(b) Magnus Carlsen


(d) None of these

13. For which game Jeev Milkha Singh is famous?


(a) Basketball
(b) Football
(c) Golf
(d) Pole-vault
14. Who among the following is credited with the
invention of light-bulb?
(a) Einstein
(b) Edison
(c) Faraday
(d) Tesla
15. Deficiency of which vitamin in body causes the
disease Beri-Beri?
(a) Vitamin-A
(b) Vitamin-B
(c) Vitamin-C
(d) None of these
16. The disease Goitre affects which part of the
human body?
(a) Eyes
(b) Nose
(c) Ears
(d) Neck
17. Where is the National Centre for Good
Governance setup by Indian government?
(a) Jaipur
(b) Delhi
(c) Bangalore
(d) Mussoorie
18. Which of the following is used in pencils?
(a) Graphite
(b) Lead
(c) Silicon
(d) Charcoal
19. Washing soda is the common name for:
(a) Sodium Carbonate

(b) Calcium Bicarbonate


(c) Sodium Bicarbonate
(d)Calcium Carbonate
20. Which of the following gas is not a Green House
Gas?
(a) Methane
(b) Nitrous Oxide
(c) Carbon Dioxide
(d) Hydrogen
21. Which of the following is used as a lubricant?
(a) Graphite
(b) Silica
(c) Iron Oxide
(d) Charcoal
22. The group of metals Fe, Co, Ni may best called
as:
(a) Alkali Metals
(b) Alkaline Earth Metals
(c) Transition Metals
(d) Rare Metals
23. Heavy water is:
(a) Deuterium Oxide
(b) PH7
(c) Tritium Oxide
(d) Addition of a substance
24. The element common to all acids is:
(a) Hydrogen
(b) Oxygen
(c) Carbon
(d) Sulphur
25. Find the Odd one out:
(a) Diamond
(c) Carbon

(b) Graphite
(d) Plati

Banking Awareness
1.

2.

Which of the following currency is the costliest?


(a) US Dollar
(b) Bahraini Dinar
(c) Euro
(d) British Pound Sterling
Issuing of credit cards is a component of:
(a) Investment Banking
(b) Retail Banking

(c) Rural Banking


(d) None of these
3.

Which of the following is not a type of cheque?


(a) Bearer Cheque
(b) Account Payee Cheque
(c) Cancelled Cheque
(d) Running cheque

GENERAL AWARENESS CLASS ASSIGNMENT - 4

Page 93

4.

5.

6.

Which of the following countries has not


accepted Euro as its main currency?
(a) Germany
(b) United Kingdom
(c) France
(d) Italy
Which of the following bank was called the
Imperial Bank of India?
(a) State Bank of India
(b) Bank of India
(c) Central Bank of India
(d) Reserve bank of India
Which of the following is a function of RBI?

1. Issue of Bank Notes


2. It is the banker to the government of India
3. It is the Custodian of cash reserves of banks.
(a) Only 1 and 2
(b) Only 2 & 3
(c) All of the above
(d) None of these
7.

Who among the following is the 23rd RBI


Governor?
(a) D Subbarao
(b) Raghuram Rajan
(c) Bimal Jalan
(d) Y.V. Reddy

8.

Find the odd one out:


(a) SLR
(c) Repo Rate

9.

(b) CRR
(d) IPO

Which of the following is the tagline of State


bank of India:
(a) The banker to every Indian
(b) The nation you can Bank upon
(c) Hum hain na
(d) None of these

10. Which of the following are terms used in stock


market?

(a) Bear
(c) Long

(b) Bull
(d) IPO

11. Which of the following is Indias oldest stock


exchange?
(a) Amsterdam Stock Exchange
(b) New York Stock Exchange
(c) Bombay Stock Exchange
(d) National Stock Exchange
12. Which among the following committee
suggested to establish Insurance Regulatory
Authority in India?
(a) Rangrajan Committee
(b) Malhotra Committee
(c) Dandekar Committee
(d) Usha Committee
13. Which of the following institutions provides
short term loans to its member countries to
stabilize their fluctuating economies?
(a) World bank
(b) International Monetary Fund
(c) Reserve Bank of India
(d) Federal Bank
14. Who among the following is the current
president of World Bank?
(a) Jim Yong Kim (b) Tony Blair
(c) Mark Anthony (d) Kofi Annan
15. Select the correct match:
i. India
I. NASDAQ/NYSE
ii. China
II. EURONEXT
iii. France
III. SSE
iv. USA
IV. NSE/BSE
(a) i-I, ii-II, iii-III, iv-IV
(b) i-IV, ii-II, iii-I, iv-III
(c) i-IV, ii-III, iii-II, iv-I
(d) i-II, ii-III, iii-IV, iv-I

World Bodies & PSUs


1.

Which of the following was the first to be


established?
(a) United Nations
(b) NAM
(c) League of Nations
(d) BRICS

2.

Where is the headquarters of United Nations?


(a) Geneva
(b) Washington D.C.
(c) New York
(d) None of these

GENERAL AWARENESS CLASS ASSIGNMENT - 4

Page 94

3.

4.

5.

Which of the
Organization?
(a) UNESCO
(c) ILO

following

is

cultural

(b) WHO
(d) FAO

Which of the following is not associated with the


UNO?
(a) ILO
(b) WHO
(c) ASEAN
(d) All of the above
Which of the following was formed by the
amalgamation of 5 financial institutes (namely
IBRD, IFC, IDA, MIGA & ICSID)?
(a) IMF
(b) World Bank
(c) RBI
(d) SBI

6.

Which country among the following does not


hold the Veto power in the Security Council of
UN
(a) USA
(b) France
(c) China
(d) India

7.

What is the expansion of E in OPEC?


(a) Energy
(b) Economy
(c) Expenses
(d) Exporting

8.

The Red Cross Society has its headquarter in:


(a) New Delhi
(b) Geneva
(c) Hague
(d) Rome

9.

Which of the following country was admitted


into G7, converting it to G8?
(a) Ukraine
(b) Russia
(c) Kazakhstan
(d) Canada

10. INTERPOL means


(a) International Political Conference
(b) International Criminal Police Organisation
(c) International Association of Police Officers
(d) None of these
11. What is BRICS?
(a) It is a type of material used to build buildings
(b) It is a fashion brand
(c) It is an association of five major emerging
national economies

(d) It is a group of cricket playing countries


12. The next G-20 summit will be held at:
(a) Russia
(b) USA
(c) France
(d) None of these
13. India is not a part of which of the following
associations?
(a) BRICS
(b) UN
(c) SAARC
(d) ASEAN
14. Where is the headquarters of ADB (Asian
Development Bank) located?
(a) Philippines
(b) Manila
(c) Geneva
(d) Hague
15. Which of the following countries is not a
member of ASEAN?
(a) Cambodia
(b) Indonesia
(c) Canada
(d) Malaysia
16. Which of the following is NOT a Maharatna
Company?
(a) Gas Authority of India Limited
(b) Coal India Limited
(c) Steel Authority of India Limited
(d) Airport Authority of India
17. Today there are
companies in India.
(a) 5
(c) 7

________

Maharatna

(b) 6
(d) 8

18. Which is Indias largest oil production company?


(a) ONGC
(b) IOC
(c) OIL
(d) GAIL
19. PSU companies are divided into ________
categories.
(a) 1
(b) 2
(c) 3
(d) 4
20. Which company is associated with distribution
of Aluminium products?
(a) SAIL
(b) HAL
(c) NALCO
(d) AAI

GENERAL AWARENESS CLASS ASSIGNMENT - 4

Page 95

Current Affairs -IV


1.

Dena Bank ties up with ________ for Group


Life Insurance to its Housing Loan customers
under Dena Grihaswami Suraksha Yojana.
(a) ICICI
(b) Axis
(c) HDFC
(d) SBI Life

5.

The title of Hero World Challenge has been


conferred upon which of the following
professional golfer?
(a) Anirban Lahiri
(b) Ricky Fowler
(c) Patrick Reed
(d) Bubba Watson

2.

Who was awarded with four world record


certificates scoring five goals in nine
minutes?
(a) Cristiano Ronaldo
(b) Lionel Messi
(c) Marco Reus
(d) Robert Lewandowski

6.

Which city Police has been conferred with


Express
IT excellence
award in
acknowledgment with IT initiatives launched
by them ?
(a) Chattisgarh Police
(b) Karnataka Police
(c) Hyderabad Police
(d) Lucknow Police

3.

Which bank launches countrys first display


variant debit card?
(a) ICICI Bank
(b) Yes Bank
(c) Axis Bank
(d) HDFC BanK

7.

International Monetary Fund (IMF) has block


its financial aid program to which country?
(a) Zimbabwe
(b) Russia
(c) Ukraine
(d) Indonesia

4.

Which country has passed Anti-Terrorism


law to strip dual nationals of their citizenship
if they are convicted of terrorism offences?
(a) Japan
(b) Australia
(c) Russia
(d) Austria

8.

Who is named as the Sports Illustrateds


2015 Sportsperson of the Year?
(a) Sania Mirza
(b) Lyndrea Price
(c) Serena Williams
(d) Venus Williams

GENERAL AWARENESS CLASS ASSIGNMENT - 4

Page 96

GENERAL AWARENESS CLASS ASSIGNMENT 5


Economics, Finance, Govt. Policies, Rankings
1.

Which of the following statements is/are not


correct about National Population Register
(NPR):
(a) It is a project of home ministry.
(b) It is a substitute for Aadhar (UID).
(c) It collects identity through three biometrics
photographs, fingerprints and iris impression.
(d) All the residents above five years of age have
to be enrolled mandatorily.

7.

2.

What is the purpose of Make in India scheme?


(a) To make India a global country.
(b) To encourage small industries in India to open
branches
all
over
the
world.
(c) To attract businesses from around the world
to invest and manufacture in India.
(d) To increase HDI of India.

8.

3.

Niti
Aayog
has
commission?
(a) Urban Commission
(b) Finance Commission
(c) Judicial Commission
(d) Planning Commission

Which of following statement is true about the


Economic Growth:
(a) It refers to increase in Gross Domestic
Product (GDP)
(b) It refers to the long and sustained rise in real
Gross Domestic Product (GDP)
(c) It is characterized by fall in unemployment
rate
(d) Rise in GDP on monetary term

9.

Market Based economy means:


(a) All economic decision are taken based on the
demand and supply forces
(b) Some economic decision are taken by
government and other are left to market
forces
(c) Government has full control over the private
sector
(d) Economic decisions are taken after keeping in
view the social welfare

4.

5.

6.

replaced

(a) Only 1 and 2


(b) Only 2 and 3
(c) Only 1 and 3
(d) All of them

which

The scheme by which each MP adopt a village of


his/her constituency and then turn them into a
model village within 2 years is known as?
(a) Deendayal Upadhyaya Gram Jyoti Yojana
(b) Sarva Shiksha Abhiyan (SSA)
(c) Sansad Adarsh Gram Yojna
(d) Pradhan Mantri Jan Dhan Yojana
Which of the following org. is associated with
HRD ministry:
(a) INDEST
(b) National Book Trust
(c) National Bal Bhavan
(d) All of above
Correct chronological order? (older to newer)
(a) Mid Day meal, RTE, SSA
(b) SSA, Mid Day Meal, RTE
(c) RTE, Mid Day meal, SSA
(d) Mid Day Meal, SSA, RTE

Governing council of the Niti Aayog includes


(1) Prime minister
(2) Finance minister
(3) Chief ministers of all states

10. Sustainable Development implies:


(a) Sustained rise in real GDP
(b) Long lasting development without negatively
impacting the environment
(c) Full exploitation of natural resources
(d) Reduction in unemployment and poverty
11. Mixed Economy Implies:
(a) Absence of Public Sector
(b) Absence of private sector
(c) Co-existence of public and private sector
(d) All economic activities are motivated by social
welfare
12. Who operates the monetary policy in India?

GENERAL AWARENESS CLASS ASSIGNMENT - 5

Page 97

(a) Ministry of Finance


(b) Reserve Bank of India (RBI)
(c) Security and Exchange Board of India
(d) All of above
13. Forward Markets Commission (FMC) to be
merged with which organization?
(a) BSE
(b) RBI
(c) SEBI
(d) SBI
14. GAAR Stands for?
(a) Govt. Anti-Avoidance Rules
(b) General Anti-Avoidance Rules
(c) General Arm Avoidance Rules
(d) None of above
15. Which was the first Indian bank to provide
internet banking facility?
(a) HSBC
(b) HDFC bank
(c) SBI
(d) ICICI bank

20. What is Indias rank on global corruption index?


(a) 85th
(b) 76th
(c) 67th
(d) None of above
21. What is Indias rank on global hunger index?
(a) 52nd
(b) 53rd
(c) 55th
(d) None of above
22. Revealed Preference Theory was propounded
by?
(a) Adam Smith
(b) Marshall
(c) P. A. Samuelson
(d) J.S. Mill

16. Which of the following is used for the


measurement of distribution of income?
(a) Laffer Curve
(b) Engels Law
(c) Gini-Lorenz Curve
(d) Philip Curve

23. The annual record for all the monetary


transactions of a country with other countries of
the world is known as
(a) Balance of trade
(b) Balance of monetary receipts
(c) Balance of payments
(d) Balance-Sheet

17. Liquidity refers to:


(a) Wealth available with investors
(b) Ease with which assets can be converted into
the money
(c) Currency convertibility
(d) All of above

24. One of the essential conditions of monopolistic


competition is
(a) Many buyers but one seller
(b) Price discrimination
(c) Product differentiation
(d) Homogeneous product

18. Which of following statement is true about the


Primary deficit?
(a) It is difference between Revenue receipts and
Revenue Expenditure
(b) It is difference between capital receipts and
Interest Payment
(c) It is difference between the Fiscal Deficit and
Interest Payment
(d) It is addition of Fiscal Deficit and Interest
Payment

25. In Economics the terms Utility and usefulness


have
(a) Same meaning
(b) Different meaning
(c) Opposite meaning
(d) None of the above

19. What does the term NSDL stands for?


(a) National Securities Development Limited
(b) National Securities Depository Limited
(c) National Safety Development Limited
(d) Natural Safety Deployment Limited

26. Gross Domestic Product is defined as the value


of all
(a) Goods produced in an economy in a year
(b) Goods and services produced in an economy
in a year
(c) Final goods produced in an economy in a year
(d) Final goods and services produced in an
economy in a year

GENERAL AWARENESS CLASS ASSIGNMENT - 5

Page 98

Management Communication II
1.

The ideal behaviour of an administrator of an


organization is:
(a) Influential and essential
(b) Impartial
(c) Organization welfare oriented
(d) Justified according to the situation

2.

The maximum number of members in a private


company is:
(a) 7
(b) 10
(c) 20
(d) 50

3.

The minimum number of directors in a Public


Company is?
(a) 2
(b) 3
(c) 4
(d) 5

4.

In which type of company there is no restriction


on the transfer of shares:
(a) Public company
(b) Private company
(c) Both of the above
(d) None of the above

5.

6.

The board of director of a Joint Stock Company


is elected by:
(a) General Public
(b) Government Bodies
(c) Shareholders
(d) Employees
A decision is taken to put capital of the company
in different assets. What is this decision called:
(a) Dividend decision

(b) Investment decision


(c) Financing decision
(d) Working capital decision
7.

Which is the highest level of need in the Maslow


theory of motivation:
(a) Physiological needs
(b) Social needs
(c) Self-actualization needs
(d) Safety needs

8.

The psychological barriers to effective


communication does not include which of the
following:
(a) Premature evaluation
(b) Lack of attention
(c) Faulty translations
(d) Distrust

9.

The first step in the process of organizing is:


(a) Departmentalization
(b) Establishing reporting relationships
(c) Assignment of duties
(d) Identification and division of work

10. The first step in the process of organizing is:


(a) Departmentalization
(b) Establishing reporting relationships
(c) Assignment of duties
(d) Identification and division of work

Current Affairs V
1.

2.

Which city has launched the first metro station


in the country to run by only women staff?
(a) Bengaluru
(b) Lucknow
(c) Jaipur
(d) Mumbai
Orange Day campaign has been organised in
which Indian State to end violence against
women as stipulated by the UN General
Assembly?
(a) Karnataka
(b) New Delhi

(c) Kerala
(d) Tamil Nadu
3.

Which two Indian banks are going to list their


bonds on the London Stock Exchange?
(a) Axis and ICICI
(b) Yes bank and SBI
(c) ICICI & SBI
(d) HDFC & YES Bank

4.

To make the rail sector passenger friendly,


Ministry of Railways launched a scheme for
waitlisted passengers named _____.

GENERAL AWARENESS CLASS ASSIGNMENT - 5

Page 99

(a) Nirnay
(b) Sujhav
(c) Vikas
(d) Vikalp
5.

6.

7.

(c) Greece
(d) Iran

Who is the Runner-Up of Mens Wimbledon


2015?
(a) Rafael Nadal
(b) Andy Murray
(c) Roger Federer
(d) Novak Djokovic
What is the name of recently developed worlds
first malaria vaccine?
(a) Mosquip
(b) Mosquin
(c) Mosquirix
(d) Mostinct
The G-20 Meeting of Finance Ministers and
Central Bank Governors was held at?
(a) Syria
(b) Turkey

8.

What is Homonaledi?
(a) Newly Discovered Species of Human Relative
(b) Newly Discovered Plant
(c) Newly Discovered Drug
(d) None of These

9.

2015 Commonwealth Youth Games (CYG)


being held at?
(a) South Africa
(b) Samoa
(c) Somalia
(d) Singapore

10. Book Titled To the Brink and Back: India's


1991 Story is written by?
(a) Digvijay Singh
(b) Kapil Sibal
(c) Manish Tewari
(d) Jairam Ramesh

GENERAL AWARENESS CLASS ASSIGNMENT - 5

Page 100

NOTES

NOTES

Вам также может понравиться